Sunteți pe pagina 1din 133

CONTENTS

DISEASES WITH TRANSMISSIVE


MECHANISM OF TRANSMITTION

TESTS AND CLINICAL CASES FOR 5th YEAR


STUDENS

Arboviral encephalits (301-320)

DISEASES WITH PRIMARY AIRDROPLET


WAY OF TRANSMISSION

Lime-borreliosis (321-329)
Louse-born typhus. Brills disease (330-342)

Influenza. ARVI (1-30)

Malaria (343-361)

Acute tonsillitis. Diphtheria (31-62)

Helminthes (362-368)

Infectious mononucleosis (63-69)


Meningococcal infection (70-102)
Legionellosis (103-131)

INFECTIOUS DISEASES WITH


INOCULATION AND PARENTERAL
MECHANISM OF TRANSMISSION

Measles (132-145)

HIV-infection and AIDS (369-378)


Rabies (379-392)

DISEASES WITH PRIMARY ALIMENTARY


WAY OF TRANSMISSION

Tetanus (393-405)

Typhoid fever. Paratyphoid A and B (146163)


Salmonellosis (164-172)

Erysipelas (406-412)
Leptosprosis (413-444)

Food poisonings (173-185)


Botulism (186-192)

DISEASES WITH MULTIPLE WAYS OF


TRANSMITTION

Pseudotuberculosis (193-204)

Haemorrhagic fevers (445-459)

Intestinal yersiniosis (205-212)


Echerichiosis (213-216)

Haemorragic fever with renal syndrome


(460-474)

Shigellosis (217-232)

Herpesvirus infection (475-500)

Cholera (233-243)

Plaque. Tularemia (501-516)

Rotaviral diarrheas (244-249)

Antrax (517-526)

Viral hepatitis (250-271)

Brucellosis (527-536)

Enteroviral diseases (272-273)

Q-fever (537-541)

Poliomyelitis (274-283)

Toxoplasmosis (542-549)

Protozoal diseases. Amoebiasis (284-293)

Sepsis (550-559)

Balantidiasis. Lambliosis (294-300)

TESTS AND CLINICAL CASES FOR 6th


YEAR STUDENS

Clinical cases

e rhinoviral infection.
6. Choose the drug for
mycoplasma infection:
a Ribavirin;
b Eritromicyn;
c Penicillin;
d Riboxin;
e Oxolin oilment.

Clinical tasks (1-58)


Skin changes in some infectious diseases
(59-64)
Laboratory method of diagnostics (65-108)

123
Urgent states in infectious pathology (109144)
DISEASES WITH PRIMARY AIR-DROPLET
WAY OF TRANSMISSION
INFLUENZA. ARVI
1. What is etiological agent of influenza?
a virus;
b chlamydia;
c bacteria;
d mycoplasmas;
e protozoa.
2. What is the most typical localization of
pathological process of influenza?
a nasopharynx;
b larynx;
c trachea;
d bronchi;
e lung.
3. What type of inflammation of the
respiratory tract is typical for the
uncomplicated influenza?
a catarrhal;
b fibrinogenous;
c purulent;
d fibrinohemorrhagic.
4. Choose the most effective medicine for
etiological treatment of influenza:
a Interferon;
b Aciclovire;
c Remantadine;
d Neovire;
e Zovirax;
5. The patient was admitted to the hospital
on the 2nd day of disease with complaints
on general weakness, T 37.1C, headache,
difficulties with nasal breathing, discharging
of abundant serous secretion from the nose.
O/E: maceration of nostrils skin, moderate
hyperemia of the back wall of the throat.
What is the most likely diagnosis in this
patient?
a influenza;
b parainfluenza;
c adenoviral infection;
d RS-viral infection;

treatment

of

7. Patient was admitted on the 3-rd day


after acute beginning of the disease. The
patient complained on chills, headache,
rhinitis and increasing of temperature from
37,80 up to 38.2C, sore throat, feeling and
irritation of eyes. O/E: dropsical face,
conjunctiva is edematous and hyperemic.
The palatine tonsils, back wall of the throat
are edematous, hyperplasia of lymphatic
follicles of back wall of throat is seen, and
submandibular lymph nodes are enlarged.
What is the most likely diagnosis?
a influenza;
b parainfluenza;
c adenoviral infection;
d RS-viral infection;
e rhinoviral infection
8. Patient was admitted on the 3rd day of the
disease. His temperature was 37.3-37.5C. The
patient complained on weakness, mild rhinitis,
sore throat, dry cough. O/E: hyperemia of the
mucous membrane of back wall of the throat,
hoarse voice, normal lung examination.
What is the most likely diagnosis?
a influenza;
b parainfluenza;
c adenoviral infection;
d RS-viral infection;
e rhinoviral infection
9. What is the most typical manifestation of the
RS-infection?
a tracheatis;
b tonsillitis;
c bronchiolitis;
d laryngitis;
e conjunctivitis.
10. Mail of 27 year-old was admitted to the
hospital during the epidemic of influenza. Onset
of disease was acute, temperature was high up
to 39.0-39.7C. Chills, pain in muscles and
joints, headache and irritation of eyes were
present. Treatment with aspirin, paracetamol
was not effective. On the 2nd day of disease
temperature was 39.8C, loss of orientation,
cramps, psychomotor excitement were noted.
O/E: meningeal symptoms are negative,
hyperemia of the skin and mucous membrane
were present.
What complication of disease took place in this
case?

a
b
c
d
e

a) infectious-toxic shock;
b) edema of the brain;
c) intracranial bleeding;
d) meningitis;
e) encephalitis.
11. What are the 2 most frequent manifestation
of the hypertoxic form of the influenza?
a meningitis;
b edema of the brain;
c myocarditis;
d edema of the lungs;
e croup;
f bronchoobstructive syndrome.
12. What are the methods of laboratory
diagnostics of the influenza and ARVI?
a immunofluorescent test (IFT);
b complement
fixation
test
(CFT) ;
c culturing
on
embrioneted
eggs;
d indirect
hemagglutination
test;
e all mentioned..
13. The influenza virus has tendency to affect
(choose correct answer):
cells of the ciliated epithelium and lymph
tissue;
cells of the ciliated epithelium;
epithelium of the gut;
epithelium of the conjunctive;
ciliated epithelium and epithelium of
conjunctive.
14. Adenoviruses have tendency to affect
(choose correct answer):
a ciliated epithelium of respiratory
tract;
b epithelium of the gut;
c epithelium of the conjunctive;
d lymph tissue;
e all mentioned.
15. What are the 2 most frequent complications
of rhinoviral infection?
a myocarditis;
b sinusitis;
c bronchitis;
d pneumonia;
e otitis;
16. What is the most frequent cause of the
acute respiratory insufficiency as a complication
of severe form of the influenza?
a croup;
b bronchocbstructive syndrome;
c edema of the lung;
d myocarditis;
e fit of spastic cough with apnea;
17. Which 2 drugs may be recommended for
treatment of the lung edema?
a Norfloxacin;
b Mannitol;

c Dexametazone;
d Heparin;
e Aminasin
18. Which 2 drugs are not recommended for
treatment of the brain edema?
a dexametasone;
b strichnin;
c Na oxibutirat;
d lasics;
e promedole.
19. Susceptibility of humans to influenza is:
a high in infants;
b *high in all age groups;
c high in immunocompromised persons;
d high in aged persons;
e high in young persons.
20. Acquired immunity after influenza is:
a Weak, short-living, that is the cause of
repeated epidemics;
b Strong for life, repeated diseases
impossible;
c Strong, but strain-specific, repeated
epidemics are connected with genetic
drift of the virus;
d Repeated epidemics are connected with
short-living immunity response and
genetic drift of the virus.
21. What is the most effective antiinfluenza
medication from mentioned below?
a -Interferon;
b Remantadine;
c Oxolin oilment 0.25%
d Aminocapronic acid 5%;
e Aspirin.
22. What is the most accepted scheme for
treatment of the influenza with remantadine?
a 1st day 300 mg, 2nd 200 mg, from 3rd
150 mg/day;
b 100 mg 3 t. d. for whole period of the
disease;
c 50 mg 4 t.d. for whole period of the
disease;
d *50 mg 3 t.d. for whole period of the
disease;
e Single dose of 300 mg daily.
23. the main factor of the development of the
lung edema in patients with influenza is:
a increased pressure in the pulmonary
system;
b disturbance of K+ and Na+-pump of
alveolar epithelium and transfer of liquid
to the alveolus gap;
c severe disturbances of microcirculation
and
disseminated
intravascular
coagulation, mainly in the lung vessels;
d changing of the oncotic properties of
blood;
e) all mentioned.

24. High frequency of such complication of


influenza as pneumonia is conditioned by all
mentioned below factors except of:
a disturbance of the ciliated epithelium
clearance;
b hyperproduction of the mucus in
bronchi
and
increasing
of
its
viscosity;
c viral affection of alveolar epithelium;
d T-cell immunity decreasing;
e *All mentioned..
25. What white cell count is characteristic for
influenza?
a leucopoenia, lymphocytosis, decreasing
of ESR;
b leucopoenia, lymphocytosis, increasing
of ESR;
c leukocytosis, neutrophylosis, high ESR;
d leucocytosis, insignificant lymphocytosis,
increasing of ESR;
e expressed leucopoenia (L 2.0*10 9 /l),
neutropoenia, lymphocytosis, high ESR.
26. Using of antiviral drugs for treatment of
influenza is:
a effective during whole period of the
disease;
b ineffective;
c effective only when it is used in the first
3 days of the disease;
d most effective when it is used after the
4 day of the disease;
e effective after the 7 day of the disease.
27. What complication of influenza is not
connected with secondary bacterial infection?
a pneumonia;
b diencephalitis;
c otitis;
d bronchitis;
e sinusitis.
28. What complications of influenza are not
connected with specific viral affection?
a diencephalitis;
b neuritis of the facial cranial nerve;
c myocarditis;
d sinusitis;
e ganglionitis.
29. The general mechanisms of transmission of
adenoviral infection are all following beside of:
a through the bites of insects;
b fecal-oral;
c air-droplet;
d contact.
30. What peculiarities of blood cell count are
typical for climax period of influenza?
a Leucopoenia, neutropoenia,
eosinopoenia, relative lymphocytosis;
b Normal count or insignificant
leucocytosis, tendency to
neutrophyls increase, drum - stick

c
d

shift to the left, monocytosis, finding


of plasmatic Turks cells;
Leucopoenia, aneosinophylia,
lymphocytosis, thrombocytopoenia,
ESR elevation;
Leucocytosis, neutropoenia, lympho monocytosis, atypical mononuclear
cells.

ACUTE TONSILLITIS AND DIPHTHERIA.


31. To what genus belongs pathogen of
diphtheria?
a Neisseria;
b Corynebacteria;
c Campilobacteria;
d Streptococci;
e Staphylococci.
32. What are the properties of Cl.
Diphtheria?
a spore-forming and Gram-negative
bacteria;
b Gram-negative and nonspore-forming
bacteria;
c nonspore-forming and Gram-positive
bacteria;
d Gram-positive and spore-forming
bacteria;
33. What is the stability of Cl. Diphteria in
environment?
a unstable;
b poorly stable;
c considerably stable;
34. What is a main source of infection with
diphtheria?
a healthy bacteria carriers;
b patients with atypical forms of
disease;
c patients with typical forms of
disease;
d patients with severe forms of
disease.
35. What are the two main way of
transmission of diphtheria?
a *contact;
b *air-droplet;
c transplacenter;
d alimentary;
e through the bites of insects.
36. What is the type of immunity response
acquired after diphtheria?
a active antibacterial;
b active antitoxic;
c passive antibacterial;
d passive antitoxic;
e all mentioned.

37. What is the main factor of the


pathogenesis of diphtheria?
a exotoxinemia;
b endotoxinemia;
c bacteriemia;
38. What is the specimen for bacteriological
investigation in diphtheria?
a blood;
b sputum;
c nasopharyngeal secretion;
d membrane from the tonsils;
e all mentioned.
39. What location of pathological process is
most typical for the modern diphtheria?
a nose;
b oropharyngeal region;
c larynx;
d trachea;
e skin.
40. What kind of inflammation is
characteristic for oropharyngeal form of
diphtheria?
a fibrinous;
b purulent;
c serous;
d productive;
e all mentioned.
41. What is a cause of enlargement of
palatine tonsils in patients with
oropharyngeal form of diphtheria?
a hyperplasia of lymph tissue;
b edema of tonsils;
c productive inflammation;
d hemorrhages;
e all mentioned.
42. What is the principal cause of death on
early stages of diphtheria (2-3 days)?
a myocarditis;
b polyneuritis;
c croup;
d infectious - toxic shock.
e hemorrhagic shock.
43. Which one of the following affections of
nervous system is characteristic for the late
period of diphtheria (2-nd week and later)?
a meningitis;
b polyneuritis;
c encephalitis;
d mental disorders;
e ganglionites.
44. Which one of the following affections of
kidneys is characteristic for severe cases of
diphtheria?
a pyelonephritis;
b glomerulonephritis;
c toxic nephroso-nephritis;
d hydronephrosis;
e all mentioned.

45. What disease should be first


differentiated with membranous form of
oropharyngeal diphtheria?
a lacunar form of tonsillitis;
b chemic and burns of pharynx;
c Simanovsky - Vincent`s tonsillitis;
d mycotic affections of mucous of
pharynx;
e herpetic stomatitis
46. Subtoxic (moderate) form of
oropharygeal diphtheria is most similar to:
a epidemic parotitis;
b purulent parotitis;
c peritonsillitis or paratonsillar
abscess);
d purulent submandibular
lymphadenitis;
e tuberculosis of cervical lymph nodes.
47. What is the basic method of diagnostics
of the diphtheria?
a epidemiological;
b clinical;
c bacteriological;
d serological;
e all mentioned.
48. What is the basic method of therapy of
diphtheria?
a systemic treatment with antibiotics;
b local treatment with antiseptics;
c detoxication specific;
d detoxication nonspecific;
e antiviral drugs.
49. What is the dose of diphtheria antitoxin
for treatment of localized oropharyngeal
form of diphtheria?
a 20 000 IU;
b 40 000 IU;
c 60 000 IU;
d 80 000 IU;
e 100 000 IU.
50. What is the initial dose of horse
hyperimmune antidiphtherial serum for
treatment of severe oropharyngeal form of
diphtheria at first 2 days of disease?
a 40 000 IU ;
b 60 000 IU;
c 80 000 IU;
d 100 000 IU;
e 150 000 IU.
51. Who is the main reservoir of diphtheria?
a ill persons and carriers of bacteria;
b cattle;
c rodents;
d birds;
e insects.
52. Which type of inflammation is most
typical for oropharyngeal form of diphtheria?
a Serous-gemorragic;

a
b
c
d
e

b fibrinous;
c diphtheric;
d productive;
e purulent.
53. Which type of inflammation is most
typical for the diphtheria of larynx and
trachea?
serous-gemorragic;
fibrinous;
diphtheric;
productive;
purulent;
54. What are the first therapeutic measures
in case of diphtheria:
a inhalations of O2;
b diphtheria antitoxin;
c penicillin;
d removing of membranes;
e glucocorticoids.
55. What is one of the most important
clinical indexes of gravity (level of toxicosis):
a spreading of a patches;
b dyspnea;
c enlargement of regional lymph
nodes;
d edema of the subcutaneous layer of
neck;
e tachycardia.
56. What is the most frequent complication
of the combined form of diphtheria
(diphtheria of oropharyngeal region and of
larynges?
a toxic shook;
b acute renal insufficiency;
c croup;
d bronchoobstructive syndrome;
e acute cardio-vascular insufficiency.
57. What is the therapeutic measure for
treatment of croup in patients with
diphtheria?
a Tracheostomy;
b glucocorticoids;
c inhalation of oxygen;
d bemegrid;
e euphillin.
58. Which antibiotic is the drug of choice for
treatment of diphtheria?
a penicillin;
b erythromycin;
c ciprofloxacin;
d amfotericin B.
e all mentioned.
59. What is the mechanism of the
development of polyneuritis complicated
diphtheria?
a reversible dystrophic changes of the
cranial nerves nuclei and the anterior
roots of spinal cord;
b severe irreversible neuropathy;

c
d

demyelinisation of sheaths of nerves;


disturbance
of
acetylcholine
synthesis in synapses.
e No one from mentioned.
60. What is the dose of diphtheria antitoxin
for treatment of cutaneous form of
diphtheria?:
a *10 20000 ME;
b 40 60000 ME;
c 80 10000 ME;
d 80 10000 ME;
e 100 150000 ME;
61. What are the medications for treatment
of diphtheric polyneuritis?
a glucocorticoids;
b potassium orotate;
c nootrops;
d proserin;
e massage.
62. What drug are not recommended for
treatment of diphtheric myocarditis?
a strophantin;
b glucocorticoids;
c rhiboxin;
d strichnin;
e tocopherol.

E
a
s
i
e
r
s
a
i
d
t
h
a
n
d
o
n
e
.

INFECTIOUS MONONUCLEOSIS

neutrophylic leucocytosis, atypical


mononuclears, eosinophylia;
d neutrophylic leucocytosis, drum stick
shift to left, eosinophylia;
e all mentioned.
69. Chose symptoms, typical for icteric form
of infectious mononucleosis:
a fever, lymphoadenopathy, oliguria,
significant serum ALAT elevation;
b lymphoadenopathy and fever are
absent, function of kidneys is
normal, hepatomegaly, significant
serum ALAT elevation;
c fever, generalized
lymphoadenopathy, function of
kidneys is normal, liver and spleen
are enlarged, moderate serum
ALAT elevation;
d generalized lymphoadenopathy;
fever is absent; the function of
kidneys is normal, significant serum
ALAT elevation.
Promise little, but do much.

63. What is the causative agent of the


mononucleosis?
a Hepadnavirus;
b Klebsiella pneumoniae;
c Epstine Barr virus;
d Myxovirus;
e Paramyxovirus.
64. Which diseases from mentioned below
are connected with Epstine-Barr viral
infection (2)?
a Caposi carcinoma;
b Burkitts lymphoma;
c acute lympholeucosis;
d nasopharingeal carcinoma;
e carcinomatosis of the ovaries.
65. What are the two from mentioned below
methods, which are effective for diagnostic
of mononucleosis?
a blood cell count;
b PHAT of patients sera with specific
antigen;
c detection of IgM antibodies to the
capsular antigen of EBV;
d detection of IgG antibodies to the
capsular antigen of EBV;
e detection of IgA antibodies to the
early antigen of EBV;
66. Treatment of mild and moderate cases of
the mononucleosis include all drugs except
of:
a penicillin;
b vitamin C;
c gargling
of
the
throat
with
antiseptics;
d dimedrol;
e glucocorticoids.
67. What are the main clinical signs of the
mononucleosis?
a prolonged fever, diarrhea, abdominal
pain;
b prolonged
fever,
generalized
lymphoadenopathy, liver and spleen
enlargement;
c prolonged
fever,
meningeal
syndrome,
liver
and
spleen
enlargement;
d prolonged
fever,
generalized
lymphoadenopathy, diarrhea.
e generalized
lymphoadenopathy,
petechial rush, kidney affection.
68. What are the typical for the
mononucleosis changing in the blood cell
count?
a neutrophylic
leucocytosis,
eosinophylia, anemia;
b leucocytosis, lymphocytosis, atypical
mononuclears;

MENINGOCOCCAL INFECTION

b lymphogenous;
c per contact;
d per neural;
e all mentioned.
78. What is the most typical causative agent
of primary purulent meningitis?
a Ps. aeruginosa;
b N. meningitidis;
c S. pneumoniael;
d Candida albicans;
e H. influenzae
79. What etiology is most typical for the
primary seasonal meningitis?
a arboviral;
b pneumococcal;
c tubercular;
d brucellous;
80. Which specimens from patients may
contain N. meningitides?
a blood;
b sputum;
c nasal secretion;
d spinal fluid;
e all mentioned.
81. Which characteristics of spinal fluid are
typical for the purulent meningitis?
a
colorless, transparent, 10 cells106/l,
protein 0.12 g/l, all cells lymphocytes;
b white with sediment, muddy; 10000
cells106/l, protein 2.3 g/l, 80% of
cells neutrophyls;
c
transparent,
rust-colored;
15
cells106/l,
protein

1.2
g/l,
lymphocytes and erythrocytes.
d colorless, twinkling; 110 cells106/l,
protein 3.5 g/l, all cells lymphocytes;
e colorless, transparent, 50 cells106/l,
protein 2.5 g/l, all cells lymphocytes;
82. What characteristics of spinal liquor
typical for the serous meningitis?
a colorless, transparent; 10 cells106/l,
protein 0.12 g/l, all cells lymphocytes;
b white with sediment; muddy, 10000
cells106/l, protein 2.3 g/l, 80% of
cells neutrophils;
c
transparent,
rust-colored;
15
cells106/l,
protein

1.2
g/l,
lymphocytes and erythrocytes.
d colorless, twinkling; 110 cells106/l,
protein 3.5 g/l, all cells lymphocytes;
e colorless, transparent, 50 cells106/l,
protein 2.5 g/l, all cells lymphocytes;

70. What is the main source of


meningococcal infection?
a patients with generalized forms of
disease;
b patients with a meningococcal
nasopharyngitis;
c carriers of meningococci;
d convalescents of meningococcal
meningitis
71. What is the site of entrance for the
meningococcal infection?
a nasopharynx;
b conjunctiva;
c damaged skin;
d trachea;
e gastrointestional tract.
72. What kind of exanthema is typical for
meningococcemia?
a maculopapular;
b papular;
c erythematic;
d hemorrhagic;
e All mentioned.
73.On which day of disease the rush
appears in case of meningococcemia?
a 1 2;
b 3 4;
c 5 6;
d 7 8;
e 10 - 12.
74. For which form of meningococcal
infection the exanthema is the most typical
sign?
a meningococcal nasopharyngitis;
b meningococcal meningitis;
c meningococcal meningoencephalitis;
d meningococcemia;
e all mentioned.
75. Which data are the most informative for
diagnostics of meningitis?
a meningeal syndrome;
b symptoms of the general
intoxication;
c inflammatory changes in spinal fluid;
d epidemiological data;
e Brudzinskys sign.
76. Which method is usually used for early
specific diagnostics of meningococcal
infection?
a biological;
b bacteriological;
c serological;
d allergological;
e all mentioned.
77. What is the main way of spreading of
meningococci in human organism?
a hematogenous;

83. What is the most effective antibiotic for


treatment of the meningococci carriers?
a Ampicillin;
b Gentamycin;
c Biseptole;
d Rifampycin;
e Tetracycline.
84. Which two antibiotics are the drugs of
choice for treatment of generalized forms of
meningococcal infection?
a Doxyciclin;
b Penicillin G;
c Streptomycin;
d Ryphampicin;
e Laevomycetin.
85. What is the most serious complication of
the early period of meningitis:
a toxic shock;
b edema of brain;
c ependimatitis;
d myocarditis;
e encephalitis.
86. What is the most serious complication of
the meningococcemia?
a toxic shock;
b Waterhouse-Fridericksens syndrome;
c edema of the brain;
d edema of the lungs;
e endocarditis.
89. Despite antibacterial and pathogenic
therapy
of
the
patient
with
meningococcemia sudden reducing of the
hemodynamic indexes :(BP 40/0 mm Hg,
PS 140 beats/min) registered. Intensive
therapy with high doses of dopamine and
glucocorticoids was ineffective. What the
most likely complications does develop in
this case?
a toxic shock;
b acute cardiovascular insufficiency;
c collapse;
d acute adrenal failure;
e edema of the brain.
90. What are the causes of the development
of ependimatitis in patients with purulent
meningitis?
a late hospitalization;
b ineffective antibacterial therapy;
c it is obligatory stage of the
pathological process;
d immunodeficiency;
e all mentioned.

development
of
convulsive
syndrome;
e all mentioned.
92. Which symptoms are not typical for the
syndrome of wedging the medulla oblongata
into the foramen magnum in patients with
comatose state?
a arterial hypotension;
b midriasis, absence of the pupil reflex;
c bradypnoe;
d arterial hypertension;
e muscular atonia.
93. Which antibiotic is the drug of choice for
initial treatment of severe form of
meningococcemia?
a Penicillin;
b Cephtriaxon;
c Laevomycetin;
d Riphampicin;
e Vibramicin.
94. What is the optimal dose of penicillin for
the treatment of meningococcal meningitis
in adults?
a 250 500.000 units/kg/day;
b 50 100.000 units/kg/day;
c 1 000 000 units /kg/day;
d 600.000 units/kg/day;
e 700.000 units/kg/day.
95. What is the optimal dose of
laevomycetin
for
the
treatment
of
meningococcal meningitis in adults?
a 50 60 mg/kg/day;
b 10 30 mg/kg/day;
c 30 50 mg/kg/day;
d 60 80 mg/kg/day;
e 80 100 mg/kg/day.
96. What is the most frequent form of
meningococcal infection:
a nazopharingitis;
b meningitis;
c meningococcemia;
d pneumonia;
e germ-carrierity.
97. Which serogroups of N. meningitidis are
the most frequent causes of generalized
forms of diseases?
a A, B, C, Y;
b A,X,Y, 29C;
c B, C, X, Z;
d A, B, C, W-135;
e A, B, C, Z.
98. Whish cultural properties are not typical
for N. meningitidis (2)?
a Gram cocci;
b Gram+ cocci;
c beanshaped appearance, frequently
as a dyplococci;
d oval, frequently chainsequenced;
e aerobes;

91. Indicate the most essential symptoms,


characteristic for meningoencephalilis
a presence
of
local
neurological
symptoms;
b comatose state;
c psycho-motor excitement;

99. Which factors are not characteristic for


N. meningitidis (2):
a presence of O-antigen;
b presence of spore;
c presence of pili;
d presence of lipopolysaccharide;
e presence of exotoxin.
100. What methods are used for express
diagnostics of meningococcal infection (2)?
a latexagglutination;
b bacterioscopy of thick drop of blood;
c culture of blood;
d serological test;
e skin allergic test.
101. Which symptoms are not typical for
meningeal syndrome?
a severe headache;
b rigidity of neck;
c vomiting;
d affection of cranial nerves;
e diarrhea.
102. Which symptoms are not typical for
meningeal syndrome?
a rigidity of neck;
b Brudzinskys symptom;
c Bechterevs symptom;
d Kernigs symptom;
e Lessages symptom.
Deeds not words.

10

LEGIONELLOSIS

a
b
c
d
e

111. Typical clinical forms of legionellosis


are the following (2):
a pneumonia;
b systemic form;
c acute infection of upper respiratory
tract;
d gastro-intestinal form;
e meningeal form.
112. Factors, contributing to development of
legionellosis are the following (2):
a depressing of humoral mechanisms
of immune system;
b depressing of cellular part of immune
system;
c decreasing of fagocyts activeness
and neutropoenia;
d persistence of pathogen.
113. Which express methods of laboratory
diagnostics of legionellosis is most effective?
a gram staining;
b hematoxilin-eosin staining;
c direct immunofluorescent test;
d culturing;
e detection of antibodies in urine.
114. What clinical sighs are not typical for
legionellosis (2)?
a high temperature;
b diarrhea;
c ineffective treatment with
cephalosporins;
d ineffective treatment with macrolids;
e development of the disease 10 days
after discharge from hospital.
115. What clinical sighs are not typical for
legionellosis?
a Massive infiltration in lungs;
b Hyponatriumemia (less than 130
mmol/l);
c ineffective tratment with penicillin;
d development of the disease 10 days
after stay in intensive care unit;
e ineffective treatment with
azytromicin.
116. What clinical symptom is not typical for
Pontiac fever?
a high temperature;
b diarrhea;
c catarrhal symptoms
d
meningeal signs;
e artralgia.
117. Incubation period of legionellosis
numrers:
a 24-48 h.
b 2-5 days;
c 2-10 days;
d 2-14 days;
e about 1 month.
118. For extrapulmonary legionellosis is
typical development of:

103. Legionellosis is not connected with:


a L. pneumophilia;
b L. micdadei;
c L. longbeache;
d L. bozemanii;
e L. icterohaemorrhagica.
104. What media are used for cultivation of
legionella?
Agaragar;
Leffler agar;
Yeastcharcoal agar;
Ascite agar;
KittTarocci agar.
105. There are the main antigens of
legionella, which produce antibodies in
human organism:
a superficial antigen;
b lipopolysacharide;
c nucleic acid;
d Oantigen;
e Capsule antigen.
106. What are the main localizations of
legionella in enviorment (2)?
a intestines of animals;
b water reservoirs;
c soil;
d human nasopharinx;
e human intestines.
107. What factors are not facilitate
colonization of legionella?
a t 25 - 42C;
b t 10 - 20C;
c presence of amoeba, green algae;
d presence of sediment in water
systems;
e chlorination of water.
108. What ways of transmission are not
typical for legionellosis (2)?
a air-droplet;
b drinking of unboiled water;
c alimentary;
d during excavations;
e direct human to human contact;
109. What are the factors of risk for
legionellosis (2)?
a immunosupression;
b allergic diseases;
c chronic diseases of stomach;
d young age;
e smoking.
110. What are the factors of risk for
legionellosis (2)?
a local anesthesia;
b endotracheal narcosis;
c allergic diseases;
d conditioners of air;
e hemophilia.

11

a myocarditis;
b pyelonephritis;
c otitis;
d meningitis;
e sinusitis.
119. Typical X-ray signs of legionellosis are
the following (2):
a massive infiltration in lungs;
b small-focal infiltration;
c pleuritis;
d bilateral affection of lungs;
e abscesses;
120. What specimens have be taken from
patient for laboratory diagnostics of
legionellosis (3)?
a sputum;
b blood;
c feces;
d spinal fluid;
e endotracheal aspirate.
121. Witch test from mentioned below does
not prove legionellosis?
a culturing;
b immune enzyme analysis;
c seroconversion with 4 fold increasing
of antibodies titer;
d titer of antibodies 1:64 or more in a
single sample;
e detection of legionella antigen in
urine.
122. Most effective and cheap method of
diagnostics of legionellosis is:
a culturing;
b detection of antigen in tracheal
aspirate with PCR;
c detection of antibodies with IHAT;
d detection of antigen in urine with
direct immunofluorescent test;
e detection of antigen in sputum with
direct immunofluorescent test.
123. What antibiotics from mentioned below
are not effective for treatment of
legionellosis (2)?
a Ciprofloxacin;
b Penicillin;
c TTrimetoprim-sulfametoxazol;
d Ceftriaxone
e Gentamicin
124. What drug is effective for disinfections
of water tanks as prophylactic of
legionellosis?
a Detergents;
b Chloramines;
c Calcium hypochlorid;
d Lysol;
e Potassium permanganate.
125. For what season epidemic outbreaks of
legionellosis are the most typical?
a winter;

b spring;
c summer;
d autumn;
e not connected with season.
126. Which factor is out of risk for
legionellosis?
a young age;
b chronic lung diseases;
c administration of glucocorticoids;
d AIDS;
e alcoholism.
127. Which sign is not included into
criterions of early diagnostics of
legionellosis?
a presiding disease with toxicosis and
hyperthermia during 4-5 days;
b cough;
c diarrhea;
d limphomonocytosis;
e elevation of serum KFK and LDG
levels.
128. Laboratory indices, typical for
legionellosis are all following except of:
a hyponatriumemia;
b 4-5 fold AlaT eleveation in blood;
c hypoalbuminemia;
d elevation of creatinine and urea
levels in blood;
e ESR 50 mm/h.
129. Which complication from mentioned
below is not typical for legionellosis?
a toxic shock;
b acute renal failure;
c edema of lungs;
d hepatic coma;
e edema of brain.
130. Which epidemiological factor from
mentioned below can be connected with
legionellosis:
a haymaking;
b excavations;
c cafeteria feeding;
d contact with animals;
e drinking of unboiled water.
131. Which antibiotic is most effective for
treatment of legionellosis?
a Oxacillin;
b Cepfriaxon;
c Vancomicin;
d Meticillin;
e Rovamicin.

12

Hope is the good breakfast, but a


bad supper.

13

MEASLES

skin;
d gastrointestinal tract;
e all mentioned.
141. What are the Belsky-Filatov-Koplik
spots?
a rash on skin;
b spots
scattered
over
mucous
membrane of a mouth in the initial
stage of measles;
c rash on the conjunctiva;
d all mentioned.
142. When do the Belsky-Filatov-Koplik spots
disappear?
a in first days after initial catarrhal
period;
b in the eruptive stage;
c during convalescence;
143. What is duration of incubation period of
measles?
a from 1 to 5 days;
b from 10 to 45 days;
c from 9 to 10 days
d from 10 days to 1 month.
e 1 year.
144. Where are the first elements of the
measles rash usually localized?
a at the conjunctiva;
b
*at the skin of neck behind the
ears and in center of face;
c at the skin of chest;
d at the skin of extremities;
e all mentioned.
145.
What
are
the
most
common
complications of measles (2)?
a
*measles croup;
b serous meningitis;
c
*pneumonia;
d gangrenous stomatitis or noma;
e encephalitis.

132. What is the ethiological agent of the


measles?
a Candida,
b Fusospirilles,
c Myxovirus;
d Rickketsia.
e Bacillus.
133. Is the causative agent of measles
pathogenic for monkeys?
a yes;
b no.
134. Who is the reservoir of measles
infection?
a monkeys;
b humans;
c cattle;
d rodents;
e all mentioned.
135. What is the route of transmission of
measles?
a alimentary;
b air-droplet;
c contact with rodent excreta;
d sexually transmitted disease;
e through the mosquito bites.
136. The susceptibility of humans to
measles is:
a not high;
b absolute;
c very high (susceptibility index is
0.96).
137. Post-infective immunity after measles
is:
a steady, life - long;
b steady, during one year;
c absent.
138. What is the portal of entry for agent of
measles?
a skin;
b mucous membrane of the upper
respiratory tract and possibly the
conjunctiva;
c tonsils;
d gastrointestinal tract;
e all mentioned.
139. What is the most severe complication
of measles?
a pneumonia;
b gangrenous stomatitis;
c encephalitis;
d bronchitis;
e purulent otitis.
140. Where are the principal pathological
changes of measles localized?
a liver;
b cardiovascular system;
c nasopharynx, respiratory tract and

In the evening one may praise the


day.

14

DISEASES WITH PRIMARY ALIMENTARY


WAY OF TRANSMISSION

c constant;
d undulating;
e no one of mentioned;
154. Specific complication of typhoid and
paratyphoids A and B is:
a intestinal bleeding;
b arthritis;
c otitis;
d intestinal obstruction;
e hypovolemic shock;
155. Specific complication of typhoid and
paratyphoids A and B is:
a intestinal obstruction;
b perforation of a thin intestine;
c pulmonary bleeding;
d encephalitis;
e polyradiculoneuritis
156. Which antibiotic is the drug of choose
for treatment of typhoid and paratyphoids A
and B?
a Penicillin;
b Metronidazole;
c Furazolidone;
d Laevomycetin;
e Sulfadimesine;
157. At patient H. on the 20th day from
beginning of typhoid doctor has suspected
the intestinal bleeding. What is further
medical tactics?
a urgent operative measures;
b hemotransfusion and infusion of the
blood substitutes;
c increasing in a daily dose of
antibiotics;
d prescription of glucocorticoids
(prednisolone, etc.);
e no one mentioned.
158. What is the most typical cause of death
in patients with the typhoid and
paratyphoids A and B during the first week
of disease?
a intestinal bleeding;
b hypovolemic shock;
c infectious - toxic shock;
d peritonitis;
e encephalitis;
159. Typical onset of clinical manifestations
of typhoid in nonimmune persons is:
a acute;
b subacute;
c gradual;
d all mentioned;
e no one mentioned.
160. The early symptom of the typhoid and
paratyphoids A and B is:
a exanthema;
b enlargement of spleen;
c relative bradycardia;

TYPHOID FIVER, PARATYPHOID A AND B


146. Pathogens of typhoid and paratyphoids
A and B belong to the genus:
a Shigella;
b Salmonella;
c Escherichia;
d Klebsiella;
e Enterobacteria;
147. Pathogens of typhoid and paratyphoids
A and B are:
a Gram-negative rods;
b Gram-positive rods;
c Gram-negative cocci;
d Gram-positive cocci;
e All mentioned;
148. Which one of following is the reservoir
of typhoid infection?
a live-stock;
b rodents;
c humans;
d wild carnivores;
e all mentioned;
149. The most common way of transmission
of typhoid infection, paratyphoids A and B
is:
a contact through the injured skin;
b air-droplet;
c transmissible;
d sexual;
e *
alimentary.
150. Toxins, produced by pathogens of
typhoid and paratyphoids A and B belong to:
a exotoxins;
b endotoxins;
c exo- and endotoxins;
d No one from mentioned.
151. Immunity after typhoid fever is:
a antibacterial, short term;
b antibacterial,;
c antitoxic, short term;
d antitoxic long-term,
e all mentioned;
152. Which systems are most affected in
case of typhoid and paratyphoids A and B
(2):
a respiratory;
b cardiovascular;
c digestive;
d lymphatic;
e nervous.
153. What is the fever pattern,
characteristic for typhoid in climax of
disease?
a remittent;
b intermittent;

15

d diarrhea;
e fever;
161.What is most typical exanthema in the
typhoid fever?
a urticaria;
b petechial;
c roseolar;
d erythematous;
e vesicular;
162. The specimen for early bacteriological
investigation of typhoid and paratyphoids A
and B is:
a blood;
b urine;
c feces;
d sputum;
e nasopharyngeal secret.
163. What peculiarities of blood cell count
are typical for climax period of typhoid
fever?
a leucopoenia, neutropoenia,
eosinopoenia, relative lymphocytosis;
b normal count or insignificant
leucocytosis, tendency to neutrophils
increasing, drum - stick shift to the
left, monocytosis, plasmatic Turk
cells;
c leucopoenia, aneosinophylia,
lymphocytosis, thrombocytopoenia,
ESR elevation;
d leucocytosis, neutropoenia,
lymphomonocytosis, atypical
mononuclear.

Soon learnt soon forgotten.

16

SALMONELLOSIS
164. The main way of transmission of
salmonelloses:
a alimentary;
b through the using of unboiled water;
c air-droplet;
d contact;
e All mentioned.
165. The specimen from patient with
localized form of the salmonellosis for
bacteriological diagnostics is:
a blood;
b feces;
c urine;
d bile;
e all mentioned.
166. What is the most typical clinical
symptom of generalized forms of
salmonelloses?
a repeated vomiting;
b polylymphoadenopathy;
c erythematous exanthema;
d high and long fever;
e All mentioned
167. What is the main specimen for
bacteriological diagnostics of generalized
form of salmonellosis?
a blood;
b feces;
c urine;
d bile;
e cerebrospinal fluid;
168. What is the most typical clinical sign of
gastro-intestinal form of salmonellosis?
a fever;
b hepatosplenomegaly;
c dyspeptic disorders;
d headache, a giddiness;
e rash.
169. What is the basic method of treatment
of patients with gastrointestinal form of
salmonellesis?
a antibacterial;
b rehydration;
c symptomatic;
d detoxication;
e dehydration.
170. What is the basic method of treatment
of patients with generalized forms of the
salmonellosis:
a diet;
b antibacterial therapy;
c rehydration;
d activating of immune system;
e symptomatic.
171. Which diseases it is necessary to
differentiate gastro-intestinal form of the
salmonellosis with at first?

a intestinal amebiasis;
b pseudotuberculosis;
c food poisonings;
d typhoid fever and paratyphoids;
e no one from mentioned.
172. What is the method of specific
diagnostics of gastrointestinal form of
salmonellosis?
a Culturing of feces;
b Bacterioscopy of feces;
c latex agglutination test;
d rectosygmoidoscopy
e no one from mentioned;

Wealth is nothing without health.

17

FOOD POISONINGS
173. What is duration of the food poisonings
incubation period?
a 1-12 hours;
b 2-3 days;
c 4-6 days;
d 7-14 days;
e 12-24 hours.
174. What complication is characteristic for
food poisonings?
a infectious-toxic shock;
b hypovolemic shock;
c edema of the brain;
d pneumonia;
e all mentioned.
175. The is the main factor of the
pathogenicity of food poisonings?
a activity of microorganism;
b production of toxin;
c allergic.
176.What is the basic method of food
poisonings therapy?
a antibacterial;
b symptomatic
c lavage of stomach;
d diet;
e immunocorrection.
177. The medication for treatment of food
poisonings:
a furazolidone;
b glucosaline;
c tetracycline;
d biseptole;
e all mentioned.
178. The volume of fluid for rehydration in
case of acute intestinal diseases is
determined with:
a intensity of intoxication;
b intensity and duration of fever;
c degree of dehydration;
d frequency of defecations;
e all mentioned.
179. The most effective measure for
treatment of food poisoning is using of:
a
antibiotics;
b
antidiarrheal drugs;
c
lavage of stomach;
d
analgetics;
e
diet.
180. Lavage of stomach is effective when
we use such volume of fluid as:
a
500 ml;
b
1000 ml;
c
1,5 l;
d
3 l;
e
4-5 l;
181. Water for lavage of stomach must be:
a
of room temperature;

b
hot;
c
cold;
d
worm;
e
of any temperature;
182. Clinical manifestation of food poisoning
is characterized with:
a
high temperature and intensive
watery diarrhea;
b
nausea, repeating vomiting,
subfebrile temperature;
c
protracted signs of the disease;
d
admixture of mucus in stool;
e
spastic pain in the lover part of
abdomen;
183. Incubation period of food poisoning is:
a
30 min-6 h;
b
6-12 h;
c
12-24 h;
d
up to 2 days;
e
up to 5 days.
184. For bacteriological investigation of
patients with food poisoning we have use
such specimens as:
a
contents of vomiting, lavage water of
stomach, stool;
b
stool, blood;
c
stool duodenal drainage;
d
contents of vomiting, stool;
e
contents of vomiting, blood.
185. What measures of treatment are not
effective in case of food poisoning?
a antibiotics;
b antidiarrheal drugs;
c lavage of stomack;
d spasmolitics;
e diet.

Deeds not words.

18

BOTULISM
186. The general factor of the pathogenesis
of the botulism is:
a toxemia;
b bacteremia;
c bacteremia and toxemia;
d inflammatiive processes in organs;
e dehydration.
187. The toxin, formed by the pathogen of
botulism, belongs to:
a endotoxin;
b exotoxin;
c exo-and endotoxin;
d no one from mentioned
188. System, that is selectively affected with
toxin of botulism is:
a cardiovascular;
b digestive;
c nervous;
d respiratory;
e urinary.
189. On reception to the doctor the patient
complained on acute development of
disease with repeated vomiting soon
followed with disorders of vision. At physical
examination of the patient mydriasis,
bilateral ptosis, and dryness of mucous of
the mouth are detected; body temperature
was normal and abdomen was slightly
inflated. What is most likely diagnosis?
a poisoning with belladonna;
b viral encephalitis;
c botulism;
d poisoning with methyl alcohol;
e poisoning with mushrooms.
190. What is the medication for specific
detoxication in case of botulism?
a trivalent botulinus antitoxin;
b interferon;
c vaccine;
d reopolyglucine
e Ringer solution.
191. What clinical symptoms are most
important for early diagnostics of botulism:
a dryness of mouth;
b nausea, vomiting, diarrhea;
c progressive weakness and disorder of
vision;
d bulbar symptoms (dysphony,
disorders of swallowing, dysarthria
etc.);
e high fever, intoxication.
192. What is the main method of treatment
of the botulism?
a antibacterial;
b symptomatic;
c antitoxic specific;
d nonspecific detoxication;

19

all mentioned.

PSEUDOTUBERCULOSIS.

d rectal swabs;
e all mentioned;.
200. Which two groups of antibiotics Y.
pseudotuberculosis are generally
susceptible to?
a penicillins;
b *
aminoglycosides;
c * tetracycline;
d nitrofurans;
e carbenicillin.
201. Which two groups of antibiotics Y.
pseudotuberculosis are generally
susceptible to?
a macrolids;
b *
third-generation cephalosporins;
c *
fluoroquinolones;
d first- generation cephalosporins
e ampicillin.
202.Which methods of laboratory
investigation are usually used for
diagnostics of pseudodotuberculosis?
a culturing of sputum;
b bacterioscopy;
c skin allergic test;
d *
serological;
e all mentioned.
203.What diseases from following which
manifest with exanthema are more often
necessary to differentiate with
pseudotuberculosis?
a typhoid fever;
b louse-born typhus;
c medicamentous toxicodermitis;
d scarlet fever;
e herpes zoster.
204. What of the following clinical forms of
pseudotuberculosis are observed more
often?
a articular;
b mixed;
c generalized;
d abdominal;
e icteric;

193. What is the reservoir of


pseudotuberculosis?
a humans;
b ticks;
c birds;
d rodents;
e All mentioned.
194. What is the main factor in pathogenesis
of a pseudotuberculosis?
a bacteremia;
b dehydration;
c immunosupression;
d toxicosis and specific allergy;
e proliferation.
195. What is the main mechanism of rush
development in pseudotuberculosis?
a drift of pathogens in a skin lymph
vessels with development of perifocal
inflammation;
b toxic and allergic affection of the
vessel wall;
c generalized proliferative and
destructive trombovasculitis;
d fixation of immune complexes in the
capillaries of a skin;
e all mentioned.
196. What is the type of rush, which appears
on soles and palms: in patients with
pseudotuberculosis?
a urtical;
b vesicular;
c erythematous;
d roseolar;
e petechial.
197. Clinical form of pseudotuberculosis with
presence of rush is more often necessary to
differentiate with:
a measles;
b chickenpox;
c rubella;
d scarlet fever;
e all mentioned.
198. What is the most typical clinical form of
a pseudotuberculosis?
a arthritic;
b mixed;
c generalized;
d abdominal;
e icteric.
199. What specimens from patients with
pseudotuberculosis may be used for
bacteriological investigation?
a blood;
b stool;
c material obtained at surgical
exploration

First think, than speak.

20

INTESTINAL YERSINIOSIS
205. The most often clinical form of the
intestinal yersiniosis is:
a appendicular;
b gastritic;
c enterocolitic;
d gastroenteritic;
e generalized.
206. Diarrhea in intestinal yersiniosis is
caused by:
a strengthened production of water
and electrolytes in lumen of
intestine;
b inflammation of a mucous of
gastrointestinal tract;
c increasing of osmotic pressure in
lumen of intestine;
d all mentioned;
e no one from mentioned;
207. What is the most typical complication
for transabdominal form of intestinal
yersiniosis?
a appendicitis;
b pyelonephritis;
c meningitis;
d myocarditis;
e all mentioned.
208. What place of the pathogen inoculation
is typical for intestinal yersiniosis?
a lymphoid formations of
oropharyngeal region;
b mucous of a stomach;
c mucous of ileum;
d mucous of rectum and sigmoid colon;
e all mentioned.
209. What is the most frequent clinical form
of intestinal yersiniosis?
a apendicular;
b gastritic;
c enterocolitic;
d gastroenteritic;
e meningeal.
210. What is the cause of diarrhea during
intestinal yersiniosis?
a amplified production of water and
electrolytes into a gut gleam;
b inflammation of the gastro intestinal mucous;
c increase of the osmolar pressure in a
gut contents.
211.What is the most typical complication
for abdominal form of intestinal yersiniosis?
a appendicitis;
b pyelonephritis;
c meningitis;
d myocarditis.
e All mentioned.

212. What is the place of introduction of the


pathogen of intestinal iersiniosis:
a lymph formations of the throat;
b stomach mucosa;
c jejune mucosa;
d rectum and sigmoid colon mucosa;
e all mentioned.
Idleness is the mother of all evils

21

ECHERICHIOSIS
213. What types of Escherichias cause
diseases similar to shigellosis?
a enteroinvasive;
b enterotoxigenic;
c enteropathogenic;
d enterohemorrhagic;
e all mentioned.
214. What types of Escherichias cause
cholera-like diseases?
a enteroinvasive;
b enterotoxigenic;
c enteropathogenic;
d enterohemorrhagic;
e all mentioned.
215. What is the method of specific
laboratory diagnostics of dysentery-like
escherichiosis?
a serological;
b bacteriological
c allergological;
d clinical;
e instrumental.
216. What is the basic method of therapy of
cholera-like escherichiosis?
a detoxication;
b diet;
c rehydration;
d desensitizing therapy;
e all mentioned.

Self-done is well done.

22

SHIGELLOSIS
217. What is the mechanism of transmission
of shigellosis?
a contact;
b air-droplet;
c fecal-oral;
d through the bites of bloodsucking
insects;
e all mentioned.
218. Active production of exotoxins is
possible for shigellas:
a Sonnei;
b Flexneri;
c Boydii;
d Dysenteriae 1;
e Dysenterie2.
219. The typical site of multiplication of the
shigellosis pathogen in human organism is:
a lymphatic formations of a thin
intestine;
b mucosa of large bowel;
c parenchymatous organs;
d blood;
e mesenteric lymph nodes.
220. The most severe course of shigellosis
causes:
a S. Sonnei;
b S. Flexneri 2;
c S. Boydii 6;
d S. Dysenteriae 1;
e all mentioned.
221.What parts of gastrointestinal tract are
most affected in shigellosis?
a stomach;
b duodenum;
c small bowel;
d caecum;
e sigmoid colon and rectum.
222. What is the reservoir of shigellosis?
a domestic birds;
b large and small horned cattle;
c humans;
d rodents;
e cats and dogs;
223. What is the type of immunity acquired
after shigellosis?
a antibacterial, long-time;
b antibacterial, short-time;
c antitoxic, long-time;
d antitoxic, short-time;
e no one mentioned.
224. The most effective antibiotic for
treatment of shigellosis is:
a Levomycetin (Chloramfenicol);
b Furazolidone;
c Metronidazole;
d Ciprofloxacin (Cifran);
e Biseptole;

225. The main method of treatment of


moderate form of shigellosis is:
a antibacterial;
b pathogenetic;
c symptomatic;
d diet;
e no one from mentioned.
226. What is the basic method of specific
laboratory diagnostics of the shigellosis?
a allergological;
b biological;
c bacteriological;
d clinical;
e serological;
227. What is the main factor of the
pathogenesis of shigellosis?
a bacteremia;
b toxicosis;
c specific allergy;
d alteration;
e proliferation.
228. What clinical symptom from following is
characteristic for the shigellosis?
a indomitable vomiting;
b enlargement of liver;
c liquid stool with slime (and a blood);
d pain in the upper parts of abdomen
of girdling character;
e no one mentioned.
229. Affection of what part of
gastrointestinal tract is typical for
shigellosis?
a stomach;
b small bowel;
c proximal part of large bowel;
d distal parts of large bowel.
230. What are the clinical symptoms of
colitic form of shigellosis?
a Gradual onset, absence of
intoxication, moderate spastic pains
in lower part of abdomen, alternation
of constipation and diarrhea,
sometimes slime in feces;
b Acute onset with sharp pain in
abdomen, symptoms of intoxication
on the beginning are absent; stool is
bloody, frequently without feces;
c Acute onset, moderate intoxication,
diffuse pain in abdomen, vomiting,
then liquid stool with admixture of
slime (and sometimes of blood);
d Acute onset, moderate intoxication,
spastic pain in left lower quadrant of
abdomen, poor liquid stool with slime
and blood.
231. What character and frequency of stool
is typical for colitic form of shigellosis?

23

Liquid, plentiful, slime and blood


(jelly-like), frequent;
b Liquid, poor, frequent, with
admixture of slime and blood;
c Liquid, plentiful, frequent, of greenish
color;
d Liquid, plentiful, rice-water-like,
frequent;
e Bloody, without feces, frequent.
232. Which antibiotics are effective for
treatment of shigellosis?
a Erythromycin;
b First-generation cephalosporins;
c Ciprofloxacin;
d Metronidazol (Trichopol);
e Penicillin;
233. Which data from patient history may
not serve as possible source of shigellosis?
a). contact with ill persons;
b). contact with carriers;
c). using of unboiled milk;
d). *
using of eggs;
e). using of unwashed apples.
234. What specimens from patients with
shigellosis you have to obtain for
bacteriological investigation? (2)
a). blood;
b). *
feces;
c). lavage water from stomach;
d). *
rectal swabs;
e). swabs from throat.
235. What characteristic for shigellosis
changing from mucosa of intestine does
rectosigmoidoscopy reveal? (2)
a). *
catarrhal inflammation;
b). slight hyperemia of mucous
membrane and large, deep ulcers;
c). *
catarrhal inflammation,
hemorrhages, not numerous erosions;
d). catarrhal and hemorrhagic
inflammation with numerous erosions and
small ulcers;
e). all mentioned.

24

CHOLERA

catarrhal inflammation of mucous


tunic of intestine;
c electrolytic diarrhea;
d catarrhal inflammation of mucous
tunic of small and large bowels;
e all mentioned.
241. What pathogenic mechanisms of acute
renal failure are typical for cholera?
a Dehydration, hemoconcentration,
blocking of microcirculation,
reduction of renal filtration;
b Toxemia, loss of water and
electrolytes, disturbance of
peripheral microcirculation;
c Dehydration, metabolic acidosis,
decreasing of potassium level in
blood;
d Increase of vessels permeability,
hemoconcentration, blocking of
microcirculation,
242. What pathogenic mechanisms of tonic
cramps are typical for cholera?
a Dehydration, decreasing of arterial
pressure, reduction of renal filtration;
b Toxemia, disturbance of peripheral
microcirculation;
c Dehydration, metabolic acidosis,
decreasing of potassium level in
blood;
d Increase of water and electrolytes
secretion as a result of activation of
adenilatcyclaze and accumulation of
cAMP.
243. What are the character and frequency
of stool typical for moderate gravity of
cholera?
a Liquid, frequent, plentiful, with
admixture of slime and blood (jelly like);
b Liquid, poor, frequent, with
admixture of slime and blood;
c Liquid, frequent, plentiful, with
admixture of "greens";
d Liquid, frequent, plentiful, rice-waterlike,;
e Liquid, plentiful, "foamy", 3-4 times
per day;
.244. What is a role of antibiotics in
treatment of cholera?(2)
a). *
reducing of stool output;
b). detoxication;
c). *
shortening of the period of
excretion of V. cholere;
d). decrease of inflammatiive
processes in intestine;
e). prevention of bacteremia.
245. What is the main specimen for
laboratory diagnostic of cholera?

233. What is the duration of the incubation


period of cholera (in days)?
a 1-5;
b 7-14;
c
10-21;
d 25-30;
e 1-2.
234. Typically cholera begin from:
a vomiting;
b abdominal pains;
c fever;
d watery diarrhea;
e all mentioned.
235. The general cause of death of patients
with the cholera is:
a infectious - toxic shock;
b hypovolemic shock;
c acute renal failure;
d edema and swelling of a brain;
e all mentioned.
236. Feces of patients with cholera usually
contain:
a slime;
b blood;
c slime and blood;
d undigested food
e do not contain neither slime and
blood or undigested food..
237. What is most typical location of V.
cholerae in human organism?
a in lumen of intestine;
b penetrate into enterocytes;
c penetrate into mesenteric lymph
nodes;
d penetrate into bloodstream;
e penetrate through the BBB.
238. Toxins of V. cholerae are:
a endotoxins;
b exotoxins;
c exo- and endotoxins;
239. The main mechanism of diarrhea in
case of cholera is:
a high permeability of intestinal
vessels;
b hyperperistalsis of intestine;
c the increased osmotic pressure in a
lumen of an intestine
(malabsorption);
d breach of function of enzymatic
systems of enterocytes (adenylate
cyclase-cAMP);
e all mentioned.
240. What is the mechanism of cholerogen
action?
a increased peristalsis of an intestine;

25

a blood;
b urine;
c *
stool;
d rectal swabs;
e bile.
246. Which saline solution we have to begin
use first for treatment of cholera?
a Ringer solution;
b Trisole;
c Disole;
d Acesole;
e Isotonic solution.

26

ROTAVIRAL DIARRHEAS

a
b
c
d
e

244. What is the main mechanism of


transmission of diarrheas of rotaviral
etiology?
a through the bites of bloodsucking
insects;
b fecal-oral;
c air-droplet;
d contact;
e all mentioned.
245. Which parts of the gastrointestinal tract
are usually affected in case of rotaviral
infection?
a stomach;
b upper parts of a small bowels;
c upper parts of large bowels;
d sigmoid colon and rectum;
e all mentioned.
246. The main mechanism of the diarrhea in
case of rotaviral infections is:
a breach of an adsorption of a liquid for
the account of malabsorption;
b hyperperistalsis of an intestine;
c increased permeability of
microvessels of an intestine;
d breach of function of adenylate
cyclase-cyclic adenosine
monophosphate (cP) system;
e all mentioned.
247.What kind of feces is typical for rotaviral
diarrhea?
a mucous with blood;
b watery, plentiful, like "rice broth";
c foamy, of yellowish color;
d watery, with "greens"
e all mentioned.
248. Abdominal pain in patients with
rotaviral gastroenteritis usually is:
a diffuse, over all abdomen, not acute;
b spastic, in the lower areas of
abdomen, intensive;
c intensive, in the upper areas of
abdomen, of constant character;
d intensive, in the right iliac area, quite
often constants;
e all mentioned.
249. What are the methods of specific
diagnostics of rotaviral gastroenteritis? (2)
a bacteriological of feces;
b *
detection of antibody titer rise
with ELIZA;
c bacterioscopy of feces;
d *
detection of antigen in feces
with latex agglutination test;
e rectosygmoidscopy
250. Which drugs may be used for
treatment of rotaviral infection?

27

infusions of saline solutions;


sorbents;
ferments;
antidiarrheal drugs;
*
all mentioned.

VIRAL HEPATITIS

257. What is the most typical syndrome for


the preicteric period of the viral hepatitis B?
a dyspeptic;
b *polyarthralgia;
c influenza-like syndrome;
d astheno-vegetative;
e mixed;
258. What is the most frequent clinical form
of viral hepatitis A?
a subclinical;
b *unicteric;
c cholestatic;
d icteric;
e fulminant.
259. What is the typical sign for the climax
period of acute viral hepatitis?
a erythematic exanthema;
b fever;
c *enlargement of liver;
d enlargement of peripheral lymph
nodes.
e all mentioned.
260. What is the biochemical parameter,
which constantly changes in the preicteric
period of acute viral hepatitis?
a *activity of alanine aminotransferase
(ALaT);
b level of a bilirubin;
c thymol test;
d sublimate titer;
e all mentioned.
261. Decreasing of liver size in climax period
of viral hepatitis B is evidence of:
a cholangitis;
b abscess of liver;
c *massive necrosis of liver;
d dyskinesis of bile-excreting ways;
e all mentioned.
262. What symptom is most typical for initial
period of hepatic failure in acute viral
hepatitis B?
a loss of appetite;
b *giddiness, sensation of "failure";
c dermal itch;
d fever;
e all mentioned.
263. What symptoms are most typical for
acute hepatic failure?
a increasing of jaundice and
hepatomegaly;
b enlargement of level of common
bilirubin and cholesterine;
c *increase of jaundice and decrease of
the sizes of a liver;
d erythematic skin rash and fever;
e all mentioned.

250. What is the superficial antigen of the


hepatitis B virus?
a HbcorAg;
b *HBsAg;
c HBeAg;
d HbxAg;
e HbyAg.
251. Which one of the following secrets
contains maximal quantity of the hepatitis A
virus?
a sperm;
b *feces;
c urine;
d saliva;
e all mentioned.
252. Affection of which from mentioned cells
is most typical for hepatitis B virus?
a epithelial cells of cholangioles;
b *hepatocytes;
c Kupffer cells;
d T-helpers;
e macrophages.
253. The long presence of HbeAg in blood
serum is evidence of:
a *chronization of a viral hepatitis B;
b convalescences phase of acute viral
hepatitis B;
c fulminant course of an acute viral
hepatitis B;
d healthy "carriage" of a hepatitis B
virus;
e all mentioned.
254. What steadiness of the hepatitis B virus
on objects of the environment?
a unstable;
b poorly steady;
c *high-steady;
d moderate steady.
255. What is the main source of the
hepatitis B virus?
a patients with icteric forms of an
acute viral hepatitis B;
b *patients with the subclinical form of
a viral hepatitis B;
c patients with the chronic form of a
viral hepatitis B;
d healthy carriers of HBV;
e no one from mentioned.
256. What is the most typical syndrome for
the preicteric period of the viral hepatitis A?
a astheno-vegetative syndrome;
b polyarthralgia;
c dyspeptic syndrome;
d *influenza-like syndrome;
e mixed;

28

264. What type of viral hepatitis more often


cause chronization of disease and formation
of cirrhosis of liver?
a A;
b B;
c *C;
d ;
e D.
265. In what group of patients lethal
outcome of viral hepatitis E are observed
more often?
a *pregnant women;
b homosexuals;
c dragabused;
d recipients of blood
e immunosupressed patients.
266. In what case of viral hepatitis B
formation of the chronic active hepatitis and
cirrhosis of the liver is typical?
a *superinfection with the virus D
(delta);
b coinfection with the virus D (delta);
c superinfection with the virus ;
d coinfection with virus A;
e all mentioned.
267. What is the most often clinical form of
viral hepatitis C?
a *unicteric;
b icteric;
c fulminant;
d hemorrhagic;
e all mentioned.
268. What is the basic method of therapy of
viral hepatitis A?
a symptomatic;
b *pathogenetic;
c immunocorrection;
d antiviral;
e all mentioned.
269. What is the basic method of
confirmation of acute stage of viral hepatitis
B?
a detection with ELISA method of antiHBs IgG in blood serum;
b *detection with ELISA method of antiHBcore IgM in blood serum;
c detection with ELISA method of HBs
Ag in blood serum;
d detection with ELISA method of antiHBe IgG in blood serum;
e all mentioned.
270. Choose symptoms, typical for icteric
form of viral hepatitis A:
a Fever, lymphoadenopathy, oliguria,
significant serum ALAT elevation;
b Lymphoadenopathy and fever are
absent, function of kidneys is normal,
hepatomegaly, significant serum
ALAT elevation;

*Fever, generalized
lymphoadenopathy, function of
kidneys is normal,
hepatosplenomegaly, moderate
serum ALAT elevation;
d Generalized lymphoadenopathy,
fever is absent, the function of
kidneys is normal, significant serum
ALAT elevation.
271. Chose symptoms, typical for icteric
form of viral hepatitis B:
a Gradual onset from nausea, loss of
appetite, increasing itch of skin with
scratches traces; jaundice; turgor of
skin is reduced; hepatomegaly; in
gall-bladder projection elastic
painless formation 33cm is
palpated..
b Acute beginning from nausea, pain in
a right upper quadrant of abdomen,
fever up to 38C; moderate jaundice;
slight hepatomegaly; palpation of
gall-bladder projection is painful.
c *Gradual onset from nausea, pain in
joints, itch of skin, and loss of
appetite; presence of jaundice;
turgor of skin is normal; .moderate
hepatomegaly, palpation of edge of
liver is sensitive.
d Acute onset with chill, fever up to
38,8C, nausea, dizziness, weakness;
moderate jaundice, temperature is
39C;. hepatosplenomegaly and.
anuria within 10 hours.
272. Which one of the following infection
routes is most often involved in the neonatal
transmission of hepatitis B virus (HBV)?
a
b
c
d
e

29

Blood transfusion;
*
Fetal contact with infected
blood during childbirth;
Ingestion of the virus via
maternal breast milk;
Transmission of the virus from
hospital personnel during
childbirth;
Transplacental transmission of the
virus.

ENTEROVIRAL DISEASES

d
cerebrospinal fluid;
e
mesenteric lymph nodes.
280. Poliovirus usually affects all types of
nerve cells beside of:
a
the anterior horn cells of spinal cord;
b
reticular formation cells;
c
vestibular nuclei;
d
deep cerebellar nuclei;
e
cortex of brain.
281. Clinical forms of poliomyelitis are all of
following beside of:
a
asymptomatic form;
b
typhoid-like form;
c
paralytic form;
d
aseptic meningitis;
e
bulbar form.
282. In typical cases of poliomyelitis
temperature pattern is:
a
continual;
b
hectic;
c
wave-like;
d
intermittent;
e
remittent.
283. Treatment of uncomplicated
poliomyelitis includes all measures except
of:
a
Interferon;
b
Ribavirin;
c
Antibiotics;
d
Maintenance of respiration;
e
Massage.

272. The most often etiological factor of


viral diarrheas:
a Reoviruses;
b Enteroviriuses;
c Adenoviruses;
d Rotaviruses;
e Coronaviruses.
273. The general method of therapy of viral
diarrheas:
a therapy with antibiotic;
b symptomatic;
c immunocorrection;
d detoxication;
e antiviral therapy.
POLIOMYELITIS
274. Polioviruses are representatives of
group:
a
Enteroviruses
b
Orthomyxoviruses;
c
Rhinoviruses
d
Adenoviruses;
e
Retroviruses.
275. Polioviruses may be cultured in:
a
hen eggs;
b
monkey kidney cells;
c
lymphocyte culture;
d
ntracerebral inoculation in multinipple mice;
e
respiratory epithelium of mice.
276. Specimens, which are commonly used
for cultivation of polioviruses, are:
a
throat swabs;
b
feeces;
c
cerebrospinal fluid;
d
blood;
e
throat swabs and stool.
277. Reservoir of poliomyelitis infection is:
a
birds;
b
rodents;
c
goats and catte;
d
humans;
e
incects
278. The main route of transmission of
poliomyelitis is:
a
through the parenteral exposure;
b
through the bites of insects;
c
alimentary;
d
air-droplet way;
e
alimentary and air droplet ways.
279. Primary multiplication of poliovirus
takes place in:
a
tonsils and Payer patches of
intestine;
b
mucous membrane of nasopharynx;
c
blood;

30

PROTOZOAN DISEASES

289. The basic method of laboratory


investigation of intestinal amebiasis is:
a parasitoscopy of blood smears;
b detection of specific antibodies;
c parasitoscopy of feces;
d rectosigmoidoscopy;
e culturing of blood..
290. What diseases it is necessary first to
differentiate with intestinal amebiasis?
a cholera;
b intestinal yersiniosis;
c shigellosis
d salmonellosis;
e rotaviral gastroenteritis.
291. What part of gastro-intestinal tract is
affected typically in case of intestinal
amebiasis?
a stomach;
b small intestine;
c proximal part of gut;
d distal parts of large intestine;
e duodenum.
292. What are character and frequency of
stools in chronic form of intestinal
amebiasis?
a Liquid, plentiful, frequent, with slime
and blood (jelly - like),
b Liquid, poor, frequent, with
admixture of slime and blood;
c Liquid, plentiful, frequent, with an
admixture of "greens",;
d Liquid, plentiful, frequent, rice-waterlike;
e Bloody, without feces, frequent.
293. What drag may be used for specific
therapy of intestinal amebiasis?
a Erythromycin;
b Remantadin;
c Chlorochin (Delagil);
d Metronidazol (Trichopol);
e Ciprofloxacin.

AMEBIAISIS
284. What is the pathogen of amebiasis:
a) Balantidium coli;
b) Izospora belli;
c) Entamoeba hartmanni;
d) Entamoeba histolytica;
e) Endolimax nana.
285. What is a reservoir of amebiasis?
a humans;
b livestock;
c pigs;
d rodents;
e insects.
286. The basic method of therapy of
intestinal amebiasis is:
a rehydration;
b detoxication;;
c diet;
d chemotherapy;
e symptomatic..
287. Patient B. complains of presence of
pain in right lower part of abdomen and
jelly-like liquid stool (mucus stained with
blood) up to 4-6 times a day during last 6
months. A day before his condition has
sharply worsened, body temperature has
risen up to 39,5 C, sharp pain in right upper
quadrant, increasing during respiration, has
appeared. Mentioned symptoms is
connected with:
a exacerbation of an intestinal
amebiasis;
b amebic hepatitis;
c amebic abscess of liver;
d amebic pericolitis;
e all mentioned.
288. Symptoms, characteristic for the
intestinal amebiasis are:
a liquid stool with mucus and blood,
spastic pain in the left lower
quadrant of abdomen, temperature
39C;
b liquid stool with mucus and blood as
raspberry jelly, discomfort in
abdomen, subfebrile temperature;
c plentiful watery stool, rumbling in an
abdomen, a normal body
temperature;
d plentiful watery stool with "greens",
pain and rumbling over all abdomen,
a body temperature 39.5C;
e all mentioned.

31

BALANTIDIASIS. LAMBLIASIS
(GARDIASIS)
294. What is the reservoir of the
balantidiasis?
a humans
b cattle
c pigs
d rodents
e all mentioned.
295. What symptom is characteristic for the
lambliasis (gardiasis0?
a fever;
b diarrhea;
c spastic pains in hypogastria;
d repeated vomiting;
e all mentioned.
296. The universal medication for treatment
of patients with the intestinal amebiasis,
balantidiasis and lambliasis is:
a Metronidazole (Trichopol);
b Yatren (Chiniofon);
c Delagil (Chloroquine);
d Erythromycin;
e Norfloxacine.
297. What materials should be taken in case
of suspicion of lambliasis?
a contents of duodenum;
b feces with pathological admixtures;
c feces without pathological
admixtures;
d blood;
e vomiting masses..
299. What form of G. lamblia can infect
humans?:
a vegetative form;
b cysts;
c all mentioned;
300. Affection of what part of gastrointestinal tract is typical for lambliasis:
a stomach;
b small intestine;
c proximal part of large intestine;
d distal parts of large intestine.
e *
duodenum and small
intestine.

32

DISEASES WITH TRANSMISSIVE


MECHANISM OF TRANSMITTION

e all mentioned.
308. What is the drug for specific therapy of
tick-borne encephalitis?
a Desoxiribonuclease;
b Remantadine;
c Specific horse serum;
d *Donor immunoglobulin;
e Ribavirin.
309. Where should be treated patients with
meningeal forms of tick-borne encephalitis?
a in out-patient departments;
b in day time hospital;
c in infectious hospital;
d on the center of consultation and
diagnostic;
e at home.
310. Where should be treated patients with
local forms of tick-borne encephalitis?
a in out-patient departments;
b in day time hospital;
c *in infectious hospital;
d in the center of consultation and
diagnostic;
e at home.
311. For what of the following forms of tickborne encephalitis specific seroprophylactic
is used?
a *unvaccinated persons at first 3 days
after bite of tick;
b vaccinated persons at first 3 days
after bite of tick;
c all people in region;
d persons who have history of tickborne encephalitis;
e persons, who visited endemic zone.
312. How long is supervision for persons,
who had attacked by ticks?
a 10 days;
b *14 days;
c 21 days;
d 28 days;
e 30 days.
313.What is the main method of tick-born
encephalitis specific diagnostic?
a bacteriological;
b virusologycal;
c *serological;
d allergologycal;
e epidemiological
314.Which one of the following group of
persons is need in vaccination for
prevention of tick-borne encephalitis?
a patients with any form of tick-born
encephalitis up to7-th day of disease;
b after tick extraction;
c persons, who works in infectious
hospitals and consulting-rooms of
infection diseases in medical center;

ARBOVIRAL ENCEPHALITS
301. What is the genus of Russian springsummer encephalitis ?
a Hantavirus;
b Togavirus;
c Bunjavirus;
d *
Flavivirus
e .Bunyavirus.
302. Which two symptoms are not typical for
Russian spring-summer encephalitis?
a meningitis;
b encephalitis;
c hepatosplenomegaly;
d polyradiculoneuritis;
e *polyuria;
f *primary affect.
303. Which two symptoms are not typical for
Japanese encephaliti?:
a meningoencephalitis;
b early brain edema;
c polyradiculoneuritis;
d *early lungs edema;
e *residual dementia.
304. Which drug can not be used for the
Japanese encephalitis treatment:
a Albumin;
b Heparin;
c *Pentamin;
d Lasics;
e Reopolyglucin.
305. What changes in spinal fluid are typical
for fever form of tick-borne encephalitis?
a high neutrophilic pleocytosis;
b *high lymphocytic pleocytosis;
c low neutrophilic pleocytosis;
d low lymphocytic pleocytosis;
e changes are not present;
306. Which cells from following are most
typically affected in case of poliomyelitic
form of tick-borne encephalitis?
a *cells of anterior horns in cervical
and thoracal parts of spinal cord;
b meninges of brain and spinal cord;
c radices of peripheral nerves;
d nuclei of cranial nerves;
e all mentioned.
307. For what of following forms of tickborne encephalitis development of epileptic
attacks is typical?
a meningeal;
b *meningoencephalitic;
c poliomyelitic;
d feverish;

33

*persons, who are going to work in


natural areas of trick-born
encephalitis;
e all mentioned.
315. Which two methods of laboratory
investigation are not used for diagnostics of
arboviral infection?
a viral isolation;
b detection of specific antibodies;
c detection of antigen with IFT;
d *skin allergic tests;
e *detection of level of kreatinine
kinaze in serum.
316. What is the reservoir of infection for
Russian spring-summer encephalitis?
a human;
b mammals and birds;
c ticks;
d *
ticks, birds and mammals.
e mosquitoes.
317. What is the main way of transmission
for Japanese encephalitis?
a contact with animals;
b contact with infected human;
c alimentary (unboiled milk);
d through the bites of ticks;
e *through the bites of mosquitoes.
318. What are the two main ways of
transmission for Russian spring-summer
encephalitis?:
a contact with affected human;
b *alimentary (unboiled milk);
c *through the bites of ticks;
d through the bites of mosquitoes;
e all mentioned.
319. What are onset symptoms of arboviral
encephalitis (3):
a *headache;
b *vomiting;
c diarrhea;
d *muscular pains;
e cough.
320. What are the diagnostics methods of
the arboviral infections:
a detection of antigen with direct
immunofluorescent test;
b detection of antibodies in titer 1:200
with complement fixation test (CFT)
or hemagglutination inhibition test
(HAIT) in single serum sample;
c *four-fold rising of antibodies titer
with HAIT or CFT in serum of acute
and convalescent phase;
d isolation of the virus from blood or
spinal fluid;
e *
all mentioned.

LIME-BORRELIOSIS
321. What clinical sign is very important for
diagnostics of Lime-borreliosis?
a *presence of skin circular erythema;
b history of ticks bite;
c increase of body temperature and
presence of toxic symptoms;
d presence of serous meningitis;
e all mentioned.
322. What is the mechanism of affection of
joints in patients with Lime-borreliosis?
a bacteremia;
b *immunoallergy;
c toxemia;
d septicopiemia;
e productive inflammation.
323. What are the characteristics of
erythema typical for Lime-borreliosis?
a acute painful with fluctuation in the
center and unclear borders;
b moderately painful with clear borders
and elevations above the level of
healthy skin;
c *moderately painful with pale
cyanotic center and bright festooned
borders;
d scratched, painless of red-violet
color, with clear borders from
surrounded skin
e all mentioned.
324. Which one of the following drugs is
used for treatment of Lime-borreliosis?
a *Tetracycline;
b Gentamycin;
c Erythromycin;
d Lincomycin;
e Biseptol;
325. How long must be supervision for Limeborreliosis convalescents?
a 3 month;
b 6 month;
c 1 year;
d *2 years;
e life-long.
326. What type of cardiac disturbances is
most typical for Lime borreliosis?
a *myocarditis;
b endocarditis;
c infarct of myocardium;
d pericarditis;
e all mentioned.
327. What is the cause of relapses of Lime
borreliosis?
a septicemia;
b *intracellular persistence of
pathogen;
c reinfection;
d superinfection;

34

e immunosupression.
328. What is the cause of chronic Lime
borreliosis course?
a immunosupression;
b *intracellular persistence of
pathogen;
c reinfection;
d superinfection;
e all mentioned.
329. Which one of the following symptoms is
typical for Limes disease?
a generalized lymphadenopathy;
b glomerulonephritis;
c enterocolitis;
d *polyradiculoneuritis;
e otitis.
330. What are the most informative
laboratory methods for diagnostic of Lime
disease? (2)
a culturing of blood and spinal
fluid;
b detection of specific IgG
antibodies with ELISA;
c *
detection of specific IgM
antibodies with ELISA;
d isolation of pathogen from
joint fluid;
e *
detection of pathogen
DNA with PCR.

35

LOUSE-BORN TYPHUS

day of normal body temperature;


from the end of the incubation period
and up to standing out of rash;
338. What type of fever is typical for climax
period of louse-born typhus?
a *constant;
b intermittent;
c wave-like;
d remittent;
e recurrent.
339. What is the basic method for specific
laboratory diagnostic of louse-born typhus?
a isolation of rickettsia from blood;
b *detection of specific antibodies in
serum;
c skin-allergic test;
d isolation of rickettsia from feces;
e isolation of rickettsia from
cerebrospinal fluid.
340. What is the drug of choose for
treatment of louse-born typhus and Brills
disease?
a Penicillin;
b Streptomycin;
c Furazolidone;
d *Tetracycline;
e Delagil.
341. What cardiovascular disturbances are
typical for climax period of louse-born
typhus and Brills disease?
a relative bradycardia and moderate
hypotension;
b *tachycardia and hypotension;
c bradycardia and hypertension;
d tachycardia and hypertension;
e all mentioned.
342. What peculiarities of blood cell count
are typical for climax period of louse-borne
typhus?
a *Leukopenia, neutropenia,
eosinopenia, relative lymphocytosis;
b Normal count or insignificant
leukocytosis, tendency to
neutrophilia, drum -tick shift to the
left, monocytosis, presence of
plasmatic cells (Turk cells);
c Leukopenia, aneosinophylia,
lymphocytosis, thrombocytopenia,
ESR elevation;
d Leukocytosis, neutropenia,
lymphomonocytosis, atypical
mononuclears.
c

330. How long is maximal many days are


maximal incubation period of louse-born
typhus?
a 5 days;
b *14 days;
c 20 days;
d 25 days;
e 30 days
331. What complication is typical for climax
period of louse-born typhus?
a pneumonia;
b thromboembolism;
c *infectious toxic shock;
d peritonitis;
e all mentioned.
332. What complication is typical for
convalescent period of louse-born typhus?
a *thromboembolism;
b meningoencephalitis;
c psychosis;
d pneumonia;
e dementia.
333. What type of encephalitis is typical for
louse-born typhus?
a diffuse purulent;
b diffuse serous;
c local purulent;
d *local serous;
e all mentioned.
334. What type of exanthema is typical for
louse-born typhus?
a *roseolar and petechial;
b macular and papular
c roseolar and papular;
d vesicular and papular;
e erythematous and papular.
335. What is the main factor of louse-born
typhus pathogenesis?
a rickettsiemia;
b *toxemia;
c autoallergy;
d immunosuppression;
e necrosis.
336. What is the immunity acquired after
louse-born typhus?
a passive natural;
b passive artificial;
c *active natural;
d active artificial;
e all mentioned.
337.Tthe patient with louse-born typhus
may serve as source of infection in one of
following cases:
a *from last 1-2 days of incubation
period and within all period of fever;
b from a beginning of clinical
manifestation and up to the 21-st

36

37

MALARIA

d 12 h;
e 2 h.
350. Which one of the following types of
malaria may develop malarial coma?
a tertian;
b quartian;
c *tropical;
d ovale;
e all mentioned.
351. What is the main pathogenic
mechanism of jaundice in patients with
malaria?
a break of capture and conjugation of
bilirubin in hepatocytes;
b development of cholangitis;
c autoimmune erythrocytes hemolysis;
d *
superfluous hemolysis of
erythrocytes as a result of
schizogony;
e all mentioned.
352. What one of the following
complications is typical for tropical malaria?
a acute renal insufficiency;
b pneumonia;
c pulmonary edema;
d *cerebral coma;
e all mentioned.
353. What type of fever is typical for
malaria?
a remittent;
b constant;
c *intermittent;
d wave-like;
e hectic.
354. Which one of the following mechanisms
is the main cause of hemoglobinuric fever in
patients with malaria?
a hemolysis of erythrocytes as a result
of schizogony;
b hemolysis of erythrocytes as a result
of decrease of their osmotic
resistance;
c autoimmune mechanisms of
hemolysis of erythrocytes;
d *hypersplenism;
e all mentioned.
355. What is the main method of laboratory
diagnostic of malaria?
a detection of specific antibodies;
b *parasitoscopy of thin blood smears;
c cultivation of blood on selective
media with subsequent pathogen
identification;
d identification of pathogen with PCR;
e parasitoscopy of thin blood film and
thick blood drop.
356. What is the level of parasitemia in case
of malaria, complicated with hemoglobinuric

343. Which one of the following agents is


causative agent of malaria tertian?
a Pl. ovale;
b Pl. malariae;
c Pl. falciparum;
d *Pl. vivax;
e all mentioned.
344. Which one of the following agents is a
causative agent of tropical malaria?
a Pl. ovale;
b *Pl. falciparum;
c Pl. malariae;
d Pl. vivax;
e L. donavani.
345. What is the basic mechanism of
anemia in patients with malaria?
a spleen hyperplasia and
hypersplenism;
b *hemolysis of erythrocytes as a result
of erythrocytic schizogony;
c autoimmune hemolysis of
erythrocytes;
d low osmotic resistance of
erythrocytes with subsequent
hemolysis;
e inherent deficit of glucose-6phosphate dehydrogenase in
erythrocytes
346. Which one of the following forms of
plasmodia is the cause of early malaria
relapses?
a sporozoites;
b gametocytes;
c exoerythrocytic schizontes;
d *erythrocytic schizontes;
e all mentioned.
347. Which one of the following forms of
plasmodia is the cause of late malaria
relapses in patients with malaria ovale?
a sporozoites;
b gametocytes;
c *exoerythrocytic schizontes;
d erythrocytic schizontes;
e all mentioned
348. Which one of the following types of
malaria may develop remote relapses?
a Malaria vivax;
b Malaria ovale;
c Malaria falciparum;
d * Malaria malaria;
e all mentioned.
349. How long is interval between
paroxysms in patients with tertian malaria?
a 24 h;
b *48 h;
c 72 h;

38

fever?
a *absent;
b insignificant;
c moderate;
d significant;
357. What one of the following drugs is the
drug of chooses for therapy of malaria?
a Laevomycetin (Chloramphenicole);
b Penicillin;
c *Chloroquine (Delagil);
d Metronidazole (Trichopol);
e all mentioned.
358. Which one of the following drugs is the
drug of chooses for the treatment of severe
tropical malaria?
a Amodiaqine;
b Chloroquine (Delagil);
c Primaquine;
d *Quinine sulfate;
e all mentioned.
359. What one of the following drugs may
be used for prevention of malaria remote
relapses?
a Chloroquine (Delagil);
b *Primaquine;
c Quinine sulfate;
d Amodiaquine;
e all mentioned.
360. What are basic clinical symptoms of
climax period of malaria vivax?
a Constant fever, chills, significant
sweating, lymphoadenopathy,
muscular pain, enlargement of
spleen and liver;
b Constant type of fever, weakness,
adynamia, pale skin, poor roseolar
rush on lateral surfaces of trunk,
enlargement of spleen and lever;
c Fever of remitting type, chills,
significant weakness, petechial
enanthema on conjunctiva,
sometimes enlargement of liver;
d *Fever of intermitting type, on climax
of fever feeling of heat with
subsequent plentiful sweating,
enlargement of spleen and liver.
361. Which one of the following drags is
effective for therapy of paroxysms of
malaria tropica?
a *
Fansidar;
b Remantadine;
c Chloroquine (Delagil);
d Metronidazol (Trichopol);
e Ciprofloxacin.
362. Which one from the following drugs is
the drugmay be used for suppressive
prophylaxis of malaria in chloroquineresistant falciparum areas?
a Chloroquine;

b
c
d
e

39

Primaqine;
Clindamycin;
*
Mefloquine
Bigumal

HELMINTHES
362. What is the route of transmission of
Ascariasis?
a using of soil-contaminated food;
b anal-oral self-contamination;
c through skin;
d using of uncooked fish;
e using of undercooked beef.
363. What is the route of transmission of
Teniasis?
a using of soil-contaminated food;
b anal-oral self-contamination;
c using of undercooked meat;
d using of uncooked fish;
e through skin and by endogenous
autoreinfection.
364. What is the route of transmission of
Hookworms?
a using of soil-contaminated
food;
b anal-oral self-contamination;
c using of undercooked meat;
d using of uncooked fish;
e through skin and by
endogenous autoreinfection..
365. Pyrantel Pamoate is effective for
treatment of:
a Hookworms;
b Enterobiasis;
c Tapeworms;
d Opisthorchiasis;
e Stronglyloidiasis.
366. Which drug from mentioned may be
effective for treatment of cystecircosis?
a Pyrantel pamoate;
b Mebendasole;
c Thiabendasole;
d Albendasole;
e Ivermectin.
367. What drug is effective for treatment of
Tapeworms?
a Pyrantel pamoate;
b Mebendasole;
c Praziquantel;
d Thiabendasole;
e Albendasole.
368. For treatment of Hookworms the most
effective drug is:
a Mebendasole;
b Albendasole;
c Pyrantel pamoate;
d Ivermectin;
e Praziquantel.
369. What is the cause of anemia associated

Vitamin B12 deficiency caused by


competitive absorption by the
parasite;
c Malabsorption of folic acid as a
result of chronic diarrhea;
d Massive hemoptysis during the
initial phases of parasitic
maturation;
Persistent iron loss.

with ancylostomiasis?
a

Bone marrow depression caused


by parasitic invasion;

40

41

INFECTIOUS DISEASES WITH


INOCULATION AND PARENTERAL
MECHANISM OF TRANSMISSION

d herpes labialis;
e intestinal amebiasis;
376. Which one of the following clinical
syndromes is characteristic for HIV-infection
on stage of persistent generalized
lymphoadenopathy?
a enlargement of any 3 groups of
lymph nodes, painful on palpation,
tendency to suppuration;
b enlargement of inguinal lymph
nodes, soldered and painful on
palpation;
c *enlargement of any 3 groups of
lymph nodes, freely movable and
painless on palpation;
d enlargement of inguinal and axillary
lymph nodes (more than 3 sm.),
soldered and painful on palpation;
e enlargement of mesenteric lymph
nodes.
377. What is the basic medication for the
specific treatment of HIV-infection?
a Remantadine;
b Pentamidine;
c Interferon;
d *Azidothymidine;
e Acyclovlr.
378. When the diagnosis of AIDS can be
proved clinicaly, disite of negative results of
antibodies to HIV testing?
a diarrheas of isospore etiology;
b Kaposi`s sarcoma in persons senior
than 60 years;
c *pneumocystic pneumonia;
d histoplasmosis of lungs;
e all mentioned.

HIV INFECTION AND AIDS


369. Which envelop glycoproteins submit
antigenic structure of the HIV 1 type virus?
a gp41, gp20, gp160;
b *gp160, gp120, gp41;
c gp41, gp110, gp120;
d gp120, gp180, gp41;
e all mentioned.
370. Where does replication of the HIV
occur?
a plasmocytes and macrophages;
b *macrophages and T-helpers;
c T-helpers and -lymphocytes;
d -lymphocytes and plasmocytes;
e No one from mentioned.
371. What opportunistic infections and
invasions are typical for AIDS?
a generalized candidiasis and
pulmonary coccididomycosis;
b leishmaniasis and toxoplasmosis;
c *pneumocytosis and disseminated
histoplasmosis;
d extrapulmonary tuberculosis and
leishmaniasis;
e shigellosis and amebiasis.
372. What is the screening - method of
diagnostics of HIV-infection?
a *immune-enzyme analysis (IE);
b radioimmune analysis(RIA);
c immunoblot analysis;
d molecular hybridization (DNAPROBES);
e polymerase chain reaction.
373. What clinical syndrome is characteristic
for dementia in patients with HIV-infection?
a syndrome of liquor hypertension;
b multifocal leukoencephalopathy;
c *discirculatory infringements of a
cerebral circulation;
d edema of the brain;
e spongiform encephalopathy.
374. What is the coefficient 4/8 (the ratio
of subpopulations of -lymphocytes) typical
for AIDS?
a *0,5;
b 1,5;
c 2,0;
d 3,0;
e 4,0.
375. What is the AIDS-indicatory disease?
a pulmonary tuberculosis;
b *pneumocystic pneumonia;
c candidiasis of a mouth;

42

RABIES

b asphyxia;
c edema of the brain;
d dehydration;
e edema of the lung.
388. What clinical sign is characteristic for
early period of rabies?
a) rigidity of the muscles;
b) difficulty of swallowing of water
(hydrophobia );
c) opisthotonos;
d) paralysis of cranial nerves;
e) disorders of consciousness.
389. What clinical manifestation is most
typical for rabies?
a) myocarditis;
b) encephalitis;
c) laryngitis with croup;
d) cardiovascular autonomic disorders;
e) dehydration.
390. For diagnostics of rabies all mentioned
tests are used except of:
a) isolation of virus from spinal fluid;
b) isolation of virus from brain;
c) detection of virus RNA in PCR;
d) visualization of typical cellular changes
in Ammon horn (Babesh-Negry bodies);
e) isolation of virus from the blood;
391. For treatment of rabies all drugs are
used except of:
a) Interferon;
b) Ribavirin;
c) Arduan;
d) Reopolyglucine;
e) Seduxen.
392. Prophylaxis of rabies includes all
measures except of:
a) preexposure vaccination with rabies
vaccine of persons with high risk of
infection;
b) postexposure local wound therapy with
soap and water;
c) passive immunization with rabies either
equine or human immune globulin;
d) passive immunization with tetanus
toxoid;
e) postexpposure active immunization with
rabies vaccine.

379. The causative agent of rabies belong


to:
a Myxoviruses;
b Rhabdoviruses;
c Retroviruses;
d Hepadnaviruses;
e Arenaviruses.
380. Where is rabies mostly spread?
a in countries of tropic belt;
b in England;
c in Australia;
d in Antarctica;
e in Ukraine.
381. In which regions rabies is not
registered?
a in England;
b in Australia;
c in USA;
d in Antarctica.
382. What is the main source of rabies
infection ?
a carnivorous;
b herbivorous;
c fish;
d birds;
e insects.
383. What is the main route of transmission
of rabies?
a parenteral;
b through the bites of animals;
c through the wounds, contaminated with soil;
d sexual;
e through the bites of insects;
384. The mean incubation period of rabies
is:
a 1-4 days;
b 7 days;
c 14 days;
d 1-2 month;
e 1-2 years.
385. What is the most common form of
rabies?
a meningeal;
b furious form;
c asymptomatic form;
d medullar form;
e carriage.
386. What is the main cause of death in the
stage of excitement of rabies?
a rupture of spine;
b asphyxia;
c edema of the brain;
d dehydration;
e edema of the lung.
387. What is the main cause of death in
paralytic stage of rabies?
a rupture of spine;

43

TETANUS

c muscles of abdomen;
d muscles of chest;
e muscles of low extremities
401. Average incubation period of tetanus
is:
a 3-5 days;
b 7 days;
c 14 days;
d 20 days;
e 1 year.
402. What 2 symptoms may indicate severe
course of tetanus?
a trismus;
b rigidity of the neck;
c dysphagia;
d rigidity of abdomen muscles;
e spontaneous prolonged spasms.
403. All of following symptoms may be
indicators of severe tetanus beside of:
a moderate trismus;
b generalized cramps;
c tachypnoea in excess of 40/min;
d apnoic spells;
e hypertension.
404. The main role in treatment of tetanus
belongs to:
a antibiotics;
b human tetanus immune globulin or equine
tetanus antitoxin;
c discharging of toxin with infusion therapy;
d glucocorticoids;
e anticoagulants.
405. The main two methods of prevention
of tetanus are:
a*
active immunization with tetanus
toxoid;
b active immunization with humane tetanus
immune globulin;
c*
active immunization with equine
tetanus antitoxin;
d active immunization with attenuated
vaccine;
e active immunization with recombinant
vaccine.

393. What pathogen from mentioned is the


cause of tetanus?
a Clostridia;
b Corynebacteria;
c Listeria;
d Neisseria;
e Bordetella;
394. What is the main route of transmission
of tetanus?
a through the bites of insects;
b trough the wounds;
c parenteral;
d sexual;
e alimentary.
395. Where morbidity from tetanus is
higher?
a in Europe;
b in South-East Asia;
c in Arctic;
d in USA;
e in Europe.
396. In which cases risk of development of
tetanus is higher?
a cut wounds;
b wounds with active infection;
a stings of insects;
b bites of dogs;
c pulmonary abscesses.
397. Mark the main pathogenic factor of
tetanus, which has importance in clinical
nanifestation?
tetanolysin;
tetanospasinin;
lipopolysaccaride;
hemolytic membrane protein;
capsule.
398. In case of tetanus all parts of nervous
system may he involved beside of:
a spinal cord;
b brainstem;
c sympathetic chain;
d parasympathetic centers;
e cortex.
399. What is the main pathogenic
mechanism of tetanus?
a releasing of inhibitory neurotransmitters
with inhibitory neurons;
b blockage of producing acetylcholine in
peripheral synapses;
c releasing of excitatory neurotransmitters
with excitatory neurons;
d demyelination of neurons axons;
e necrotic changes in neurons.
400. Which group of muscles are involved in
manifestation of tetanus firs?
a muscles of face;
b muscles of hands;

44

ERYSIPELAS
406. Show the causative agent of erysipelas.
a) Corinebacteria;
b) Streptococci of group A;
c) Frysipelothrix;
d) Staphylococci;
e) Neisseria.
407. Where does erysipelas localize?
a) in skin;
b) in skin and mucous membranes;
c) in brain;
d) in sexual organs;
e) in joints and muscles.
408. When ought you to suggest erysipelas
as repeated form?
a) when it repeats half year after primary
disease;
b) when it repeats 1 year after primary
disease;
c) when it repeats 1,5-2 years after
primary disease;
d) when it repeats 2 weeks after primary
disease;
e) when it repeats 2-3 month after primary
disease.
409. What clinical form is not typical for
erysipelas?
a) erythematous;
b) erythematous-hemorrhagic;
c) skin-bubonic;
d) necrotic;
e) bullous.
410. The most frequent localization of
erysipelas is:
a) low extremities;
b) upper extremities;
c) face;
d) chest;
e) sexual organs.
411. What is the base for diagnostics of
erysipelas?
a) clinical manifestation;
b) isolation of causative agent from blood;
c) isolation of causative agent from skin
and mucous membranes;
d) serologic tests;
e) skin allergic test.
412. What is the drug of choice for
treatment of erysipelas?:
a) Penicillin G;
b) Nitrofurans;
c) Sulfonamides;
d) Metronidazole;
e) Albendazole.

45

LEPTOSPIROSIS

e nervous and digestive systems.


420. What type of exanthema is most
typical for leptospirosis?
a vesicular;
b macular;
c petechial;
d erythematous;
e papular.
422. Choose from the following pathogens
two causative agents of leptospirosis.
a S. enteritidis;
b L. Pomona;
c L. pneumophilia;
d L. icterohaemorrhagica;
e Ch. Psittaci.
424. Leptospirae can for a long time survive
in:
a food;
b milk;
c water;
d soil;
e air.
425. Choose two groups of risk for
leptospirosis.
a medical workers;
b veterinary workers;
c forest-guards;
d drivers;
e fishermen.
426. What is the possibility of human-tohuman transmission of leptospirosis?
a high;
b impossible;
c rare.
427. Which two from following ways of
transmission
are
not
typical
for
leptospirosis?
a alimentary;
b *
air-droplet;
c *
through the bites
of insects;
d through the contact of mucous
membranes and breaks in the skin
with water of rivers or soil.
e through the contact with animals.
428. What pathogenic mechanism is not
typical for of leptospirosis?
a development of systemic infectious
process;
b development only local infectious
process;
c necrotic changes in liver;
d affection of kidneys;
e penetration of the leptospirae into
CNS;
429. What pathogenic mechanism of
affection of kidneys is not typical for
leptospirosis?

413. What is the pathogen of leptospirosis?


a Spirochaeta;
b Listeria;
c Corynebacteria;
d Rickettsia;
e Virus.
414. What type of toxins toxin of Leptospira
belongs to?
a exotoxin;
b endotoxin;
c exo-and endotoxins;
415. What is the type immunity acquired
after leptospirosis?
a antitoxic unsteady;
b antitoxic steady;
c antibacterial unsteady;
d antibacterial steady;
e all mentioned.
416. Which two from following routes of
transmission are typical for leptospirosis?
a through the bites of insects;
b *
through the contaminated
food;
c air-droplet;
d *
through the contact of mucous
membranes and breaks in the skin
with water of rivers or soil;
e using of unboiled water from waterpipe.
417. What is the reservoir of leptospirosis?
a humans;
b ticks;
c rodents;
d birds;
e mosquitoes.
418. Which two from following factors are
typical for pathogenesis of leptospirosis?
a *
accomplishing of
circulating immune complexes in
tissues;
b suppression of immune system;
c *
circulating of leptospira in
bloodstream and establishing them
in parenchymatous organs;
d tropism of leptospira to lymphatic
system;
e proliferative processes in organs with
formation of granulomas.
419. Which human systems are most likely
injured in leptospirosis?
a urogenital and cardiovascular
systems;
b cardiovascular and respiratory
systems;
c respiratory and digestive systems;
d digestive and urogenital systems;

46

a
b
c

affection of glomeruli;
affection of tubular system;
disturbances of microcirculation in a
renal parenchyma;
d edema of intersticious space.
430. What is the cause of affection of lungs
in patients with leptospirosis?
a affinity of pathogen to the lung
tissue;
b interstitial edema;
c hemorrhagic changes in the lung;
d formation of apneumatosises;
e all mentioned.
431. Leptospirae can survive for a long time
in:
a lung tissue;
b tubular system of a kidneys;
c liver;
d CNS;
e eye..
432. What is the most typical incubation
period of leptospirosis?
a 6-12 h;
b 2-3 days;
c 1-2 weeks;
d 1 month.
e 3 month.
433. What is the most typical clinical form of
leptospirosis?
a asymptomatic;
b unicteric;
c icteric;
d severe forms with fatal outcome.
e gastrointestinal.
434. What symptom is not typical for clinical
manifestation
of
unicteric
form
of
leptospirosis?
a muscular pain;
b hyperemia of face and conjunctivas;
c tonsillitis;
d meningitis;
e hepatosplenomegaly.
435. What complication of leptospirosis is
nonspecific one?
a gastrointestinal bleeding;
b pneumonia;
c insufficiency of kidneys and liver;
d clouding of vitreous body;
e meningitis.
436.Relapses
after
first
attack
of
leptospirosis usually develop through the
period in:
a 1-2 days;
b week;
c month;
d 1 year;
e 3 years.
437. What changes in laboratory tests are
not typical for leptospirosis?

leukocytosis,
neutrophilia,
aneosinophylia, thrombocytopenia;
b leukocytosis with neutrophilic shift to
left, very high ESR;
c leukocytosis,
eosinophylia,
thrombocytosis;
d increased level of serum creatinine
and urea;
e increased level of serum indirect
bilirubin
438.
What
are
the
specimens
for
bacteriological diagnostics of leptospirosis?
(2)
a blood;
b vomiting masses;
c sputum;
d feces;
e urine;
439. What is the material for early
bacteriological diagnostics of leptospirosis
(first 5 days)?
a blood;
b vomiting masses;
c sputum;
d feces;
e urine;
440. What are the materials for diagnostics
of leptospirosis after 7-10 th days of the
disease (2):
a blood;
b vomiting masses;
c sputum;
d feces;
e urine sediment.
441. What are the two most important
methods of serological diagnostics of
leptospirosis?
a agglutination test;
b indirect hemagglutination test;
c reaction of microagglutination and
lysis of leptospira;
d immunoferment assay;
e compliment fixation test.
442. What are the medicines for treatment
of leptospirosis? (2):
a Penicillin;
b Gentamycin;
c Biseptol;
d Ampicillin;
e Remantadine.
443. What are the medicines for treatment
of the acute renal failure in patients with
leptospirosis? (2)
a Vicasole;
b 7,5% solution of potassium chloride;
c Lazix;
d Dexazone;
e Mechanical respiration.

47

444. Choose symptoms, typical for icteric


form of leptospirosis.
a Fever, lymphoadenopathy, oliguria,
moderate serum ALAT elevation,
hepatosplenomegaly;
b Lymphoadenopathy and fever are
absent, function of kidneys is normal,
hepatomegaly, significant serum
ALAT elevation;
c Fever, generalized
lymphoadenopathy, function of
kidneys is normal,
hepatosplenomegaly, moderate
serum ALAT elevation;
d Generalized lymphoadenopathy,
fever is absent, the function of
kidneys is normal, significant serum
ALAT elevation.

48

DISEASES WITH MULTIPLE WAYS OF


TRANSMITTION

452.
What
are
the
most
typical
hematological changes in patients with
hemorrhagic fever?
a normal
erythrocytes
count,
leukocytosis with shift to left, high
ESR;
b erythrocytosis,
thrombocytosis,
leukocytosis with shift to left, high
ESR;
c normal
erythrocytes
count,
leukopenia, lymphocytosis, slightly
increased ESR:
d normal
erythrocytes
count,
thrombocytopenia,
leukopenia,
eosinopenia, shift to left of the
leucocytes formula, high ESR;
e normal
erythrocytes
count,
leucocytosis with shift to left, very
high ESR, thrombocytopoenia.
453. What is the main reservoir of Marburg
hemorrhagic fever?
a rats;
b monkeys;
c birds;
d ticks;
e human.
454. What is the main way of transmission
of the Marburg hemorrhagic fever?
a contact with monkeys;
b human-to-human contact;
c contact with rats;
d through the bites of insects;
e alimentary.
455. Which ways of transmission are not
typical for Lassa hemorrhagic fever?
a air-droplet;
b contact with rodents;
c through the bites of insects;
d alimentary;
e human-to-human contact
456. What symptom is not typical for Lassa
hemorrhagic fever?
a high fever;
b ulcers on the mouth mucous;
c muscular pains;
d jaundice;
e hemorrhagic rush;
f affection of kidneys.
457. What symptom is not typical for
Murburg & Ebola hemorrhagic fever?
a high fever;
b hemorrhagic rush;
c nasal, stomach and gut bleedings;
d affection of kidneys;
e sore throat.
458. Which factors are not typical for
pathogenesis of arboviral infections?

HAEMORRAGIC FIVERS
445. What is the genus of the CongoCrimean hemorrhagic fever pathogen?
a Arenaviruses:
b Hantaviruses;
c Togaviruses;
d Bunyaviruses;
e Flaviviruses.
446. What is the genus of the hemorrhagic
fever with renal syndrome?
a Arenaviruses:
b Hantaviruses;
c Togaviruses;
d Bunyaviruses;
e Flaviviruses.
447. What is the genus of the Ebola fever
pathogen?
a Arenaviruses;
b Hantaviruses;
c Filoviruses;
d Togaviruses;
e Bunyaviruses.
448. Which two symptoms are not typical for
Congo-Crimean hemorrhagic fever?
a stomach and gut bleeding;
b kidneys affection;
c hemorrhagic rush on a skin and
mycoses;
d acute beginning;
e severe syndrome of intoxication;
f affection of respiratory tract.
449. What drugs can be used for the
treatment of hemorrhagic fevers?
a Lamivudin;
b Ribavirin;
c Aciclovir;
d Streptomycin;
e Cephtriaxon.
450. What drug cannot be used for the
treatment of hemorrhagic fevers?
a Cycloferon;
b Lasix;
c Heparin;
d Glucocorticoids;
e Aminoglycosides.
451. Choose two target tissues for pathogen
of Congo-Crimean hemorrhagic fever.
a vessels;
b kidneys;
c liver;
d nervous system;
e lungs.

49

reproduction in reticulo-endothelial
system;
b virusemia;
c syndrome of intoxication;
d affection of kidneys cells.
e all mentioned.
459. What changes in spinal fluid are typical
for arboviral encephalitis?
a cytosis of 500*106/l, with prevalence
of lymphocytes;
b cytosis from 10 up to 100*10 6/l, with
prevalence of lymphocytes;
c cytosis up to 1000*106/l or more,
with prevalence of neutrophiles;
d cytosis lesser than 10*106/l, with
prevalence of lymphocytes.
e all mentioned.

50

HEMORRHAGIC FEVER WITH RENAL


SYNDROME

467. What is the main mechanism of


occurrence of rash in patients with HFRS?
a sedimentation of immune complexes
in skins capillaries;
b carrying of infectious pathogens in
skin lymphatic vessels and
development there of inflammatory
changes;
c toxico-allergic impairment of
endothelial cells;
d generalized destructive-proliferative
thrombovasculitis;
e all mentioned.
468. Which one of the following
combinations of human systems is the most
likely injured with HFRS pathogen?
a) lymphatic system and respiratory one;
b respiratory system and
cardiovascular one;
c cardiovascular system and kidney;
d kidney and lymphatic systems;
e nervous system and digestive one.
469. What type of affection of kidney is
typical for hemorrhagic fever with renal
syndrome?
a nephrosclerosis;
b pyelonephritis;
c glomerulonephritis;
d nephrosonephritis;
e all mentioned.
470. Which of the following combinations of
symptoms are characteristic for the climax
period of hemorrhagic fever with renal
syndrome?
a hepatomegaly and fever;
b oliguria and hemorrhagic rash;
c vomiting and cough with purulent
phlegm;
d anuria and jaundice;
e diarrhea and vomiting.
471. What changing in urine analysis is
typical for hemorrhagic fever with renal
syndrome?
a hematouria, proteinuria, hyaline and
epithelial casts;
b crystalluria;
c pyuria, hyaline casts;
d proteinuria, hematocyturia;
e pyuria.
472. What is the main specific laboratory
test for diagnostic of HFRS?
a skin-allergic test;
b detection of antibodies with indirect
immunofluorescent test;
c detection of virus in blood with
enzyme immunoassays;
d contamination of laboratory animals
(biological test);

460. What is the source of the hemorrhagic


fever with renal syndrome?
a human;
b birds;
c *rodents;
d insects;
e fish.
461. What is the main factor of
pathogenesis of the hemorrhagic fever with
renal syndrome?
a immunosuppression;
b *dehydration;
c toxicosis;
d allergic;
e All mentioned.
462. What type of exanthema is typical for
hemorrhagic fever with renal syndrome
(HFRS)?
a *petechial;
b roseolar;
c urtical;
d macular and papular;
e vesicular.
463. Which morphological changes are
typical for affection of kidney in patients
with HFRS? (2)
a *
focal hydronephrosis;
b glomerulonephritis;
c pyelonephritis;
d *
nephroso-nephritis;
e urolithiasis.
464. What changes in common urine test
are typical for HFRS?
a pyuria;
b albuminuria, hematuria;
c albuminuria, microhematuria, casts
(hyaline, granular, epithelial);
d pyuria, albuminuria, casts (hyaline,
granular);
e microhematuria,
pyuria,
casts
(hyaline).
465. What complications are the typical for
HFRS?
a polyneuritis;
b rupture of kidney capsule;
c renohepatic insufficiency;
d gastrointestinal bleeding;
e edema of the lung.
466. What complications are typical for
HFRS?
a acute renal insufficiency;
b polyneuritis;
c acute renohepatic insufficiency;
d acute adrenal failure;
e edema of the brain.

51

e common blood test


473. What is the most typical complication
for hemorrhagic fever with renal syndrome?
a edema of the brain;
b acute renal failure;
c meningoencephalitis;
d peritonitis;
e edema of lungs;
474. Which one of the following drugs is
used for treatment of HFRS in climax period?
a Panangin;
b Furazolidone;
c Prednisolone;
d Delagil;
e Tymogen.

52

HERPESVIRAL INFECTION

b *Acyclovire;
c Arbidole;
d Remantadine;
e *Gancyclovire.
483. What is the main course of clinical
activation of CMV-infection?
a vitamin insufficiency;
b overcooling;
c *immunodeficiency;
d physical exhaustion;
e treatment with antibiotics.
484. CMV-infection affects all below
mentioned cells except of:
a neutrophiles;
b monocytes;
c *enterocytes;
d endocrine glands cells;
e B-lymphocytes.
485. Clinical manifestation of CMVinfection includes all below mentioned
signs except of:
a retinitis;
b *thyroiditis;
c pyelitis;
d hepatitis;
e pneumonia.
486. Clinical manifestations of CMVinfection includes all below mentioned
signs except of:
a prolonged fever;
b *endocarditis;
c *enteritis;
d nephritis;
e meningitis.
487. What kind of antibody in high titers
shows the affection of newborn with
CMV-infection?
a IgA;
b IgG;
c *IgM;
d IgE;
e IgF.
488. Which two drugs are the most
effective for treatment of CMV-infection?
a Cricsivan;
b Foscarnet;
c Azidotimidine;
d *Ganciclovire;
e Zovirax.
489. Human herpes virus HV-6 can be
the agent of all mentioned diseases
except of:
a exanthema of newborns;
b chronic fatigue syndrome;
c *meningoencephalitis;
d B-cell lymphoma;
e all mentioned.

475. Which two pathogens are not


belong to group of Herpesviridae?
a *Hantavirus;
b HSV-1 and HSV-2;
c Cytomegalovirus;
d Bunjavirus;
e Epstein-Barr virus;
476.
Which
one
of
following
pathomorphological
peculiarities
is
typical for herpetic encephalitis?
a formation
of
intracellular
insertions (Negri bodies);
b *myelinoclasis of nerves;
c hemorrhagic vasculitis;
d detachable neurons affection;
e disturbance of the synthesis of
acethylcholine in synapses.
477. What way of transmittion is not
typical for HSV-infection?
a contact;
b sexual;
c *through the blood;
d transplacentar;
e perinatal.
478. What are the typical course and
prognosis of the herpetic encephalitis?
a mild;
b moderate;
c severe;
d favorable;
e *unfavorable;
479. What are the typical course and
prognosis of the herpetic meningitis?
a mild;
b moderate;
c severe;
d *favorable;
e unfavorable;
480. What is the main way of
transmission
of
Varicella
zosterinfection?
a through the blood;
b *air-droplet;
c alimentary;
d sexual;
e through the bites of ticks.
481. The Epstein-Barr virus has tendency
to affect:
a neurons;
b *B-lymphocytes;
c T-lymphocytes;
d skin and mucous membranes;
e hepatocytes.
482. What are the two main antiherpetic
drugs?
a Ribavirine;

53

490. Which two symptoms from the


mentioned below are not typical for
herpes zoster-infection?
a fever;
b *polyadenitis;
c acute pain syndrome;
d vesicular rush on the skin
streaming along dermatome;
e *herpetic tonsillitis.
491. What are the two indications for
prescription of Acyclovire in case of
herpetic infection?
a herpes of lips;
b chicken pox;
c *primary genital herpes;
d nasal herpes;
e herpetic encephalitis.
492. The Tsank cells, typical for herpes
simplex-infection are:
a gigantic multinuclear cells with
intranuclear insertion;
b *cells with dark intranuclear
insertions,
encircled
with
enlightenment zone (owls eye);
c cells
with
cytoplasmic
eosinophilic insertions;
d multinuclear
syncytium
with
necrosis of the nuclei;
e multinuclear
syncytium
with
bazophilic incertions.
493. Which method from mentioned is
most
acceptable
for
herpetic
encephalitis diagnostics?
a clinical
and
epidemiological
investigation;
b detection
of
specific
IgG
antibodies in blood;
c *detection of viral DNA in the
spinal fluid with PCR;
d computer tomography of the
brain;
e all mentioned.
494. A 56-year old female under

2). What is the main cause of the


disease?
a) vitamin insufficiency;
b) overcooling;
c) *immunodeficiency;
d) physical exhaustion;
e) treatment with antibiotics
495. A 5-year-old child suffers from
vesicular rush on the mucous of the
mouth lusting for 3 weeks. Such rush
was observed several times during lust
year. What type of drug is most likely to
alleviate this childs symptoms?
a Ribavirine;
b *Acyclovire;
c Azidothymidine;
d Lamivudine;
e Arbidole.
496. The mechanism of action of
Acyclovire used to treat HS-infection
involves;
a *blocking of the active site of the
cells
DNA-dependent
RNA
polymerases;
b incorporation of a nucleoside
analog into a virus DNA chain;
c induction of the synthesis and
release of type I interferons;
d inhibition of the releasing of viral
nucleic acid into the host cell;
e viral proteinase inhibition.
497. The Hants syndrome is manifested
with:
a *combination of facial nerves
paralysis, pain and vesicular rush
in the ear area;
b combination of back passage and
waterworks disturbances with
pain and vesicular rush in the
anal and genital area;
c combination of acute pain in the
mandibular area with vesicular
rush on the mucous of the mouth.
498. Herpetic infection is prone to affect
all mentioned organs and tissues except
of:
a conjunctiva;
b liver;
c *suprarenal glands;
d bone marrow;
e bronchi.
499. Varicellaformic Kaposhis eczema
usually occurs:
a in HIV- infected persons;
b in aged persons;
c *in children ill with eczema;
d in young persons with allergic
diseases;

treatment for inoperable breast cancer


develops a fever and painful vesicular,
hemorrhagic rash on her right buttock.
1).

The

most

likely

diagnosis

is

superinfection with:
a

Kaposhis sarcoma;

CMV-infection;

erysipelas;

*herpes-zoster-infection;

meningococcal infection.

54

in all age groups of persons ill


with helminthiasis.
500. What drug is not used as etiological
treatment of herpetic infection?
a Zoviracs;
b Cycloferone;
c *Retabolile;
d Thebrofen ointment;
e all mentioned
501. A sexually active 22-year-old
college student presents to the local clinic
with a localized vesicular eruption on the
shaft of his penis. A scraping of the base of
one of the vesicles is positive for Tzank
cells. The patient mentions that he had a
similar eruption in the same area 2 month
earlier. The reappearance of this eruption
may be explained by:
a cell-mediated
immunity
(CMI)
deficiency in the patient;
b

prolonged

period

of

viremia

following the initial infection;


c

a second infection with a similar virus


with a different serotype;

failure of the patient to comply with


therapy

prescribed

at

the

initial

episode;
e

reactivation of a latent

infection.

55

PLAGUE. TULAREMIA

e axillaries.
509. Which one of the following groups of
lymph nodes is most likely affected in case
of pharyngeal form of tularemia?
a anterior cervical;
b inguinal;
c cubital;
d mesenteric;
e posterior cervical.
510. In which period of pneumonic form of
plague the patients are most dangerous for
other persons?
a incubation;
b initial;
c climax;
d convalescent;
e no one from mentioned.
511. What clinical form of plague is more
often registered on a beginning of epidemic?
a primary pneumonic;
b bubonic;
c primary septic;
d cutaneus..
512. What is the most often cause of death
in case of septic form of plague?
a acute renal insufficiency;
b acute respiratory failure;
c septic shock;
d edema of the brain;
e edema of the lungs.
513. What is the first step in the
management of patients with bubonic
plague?
a prescription of immune globulin;
b local bubo therapy;
c detoxication;
d antibiotic therapy;
e dehydration therapy.
514. What is the main method of plague
treatment?
a antibiotic therapy;
b serotherapy;
c detoxication therapy;
d dehydration therapy;
e rehydration therapy.
515. On examination of patient doctor has
suspected a plague. Which one of the
following facilities should be informed first?
a city center of state sanitary
epidemiological surveillance;
b regional center of state sanitary
epidemiological surveillance;
c head of current medical facility;
d police office;
e relatives of a patient.
516. What skin phenomena are typical for
skin form of plague?

501. What is the main way of transmission


of bubonic form of plague?
a through the insects bites;
b alimentary transmission;
c air-droplet transmission;
d transplacentar transmission (from
mother to fetus);
e through the contact with livestock;
502.What is the main way of transmission of
cutaneus form of plague?
a through the insects bites;
b water way of transmission;
c air-droplet transmission;
d through the contact with patients;
e through the contact with livestock.
503. How long is incubation period of
plague?
a 1-6 days;
b 3-7 days;
c 5-10 days;
d 7-14 days;
e 11-21 days;
504. How long is incubation period of
tularemia?
a 1-6 days;
b 3-7 days;
c 5-10 days;
d 7-14 days;
e 11-21 days;
505. What is the reservoir of plague?
a cattle;
b wild carnivores;
c rodents;
d poultry;
e ticks.
506. Which one of the following forms of
plague is localized one?
a secondary septic form;
b primary septic form;
c bubonic form;
d secondary pneumonic form;
e primary pneumonic form.
507. Which one of the following drugs may
be used for urgent prevention of plague?
a Erythromycin;
b Penicillin;
c Polymyxin;
d Streptomycin;
e Vancomicin.
508. Which one of the following groups of
lymph nodes is most likely affected in
bubonic plague?
a cervical;
b inguinal;
c cubital;
d mesenteric;

56

There is on the hand painless ulcer


23 cm with a blackened necrotic
eschar circled with elevated
hyperemic rim, where small satellite
vesicles, filled by serous-hemorrhagic
contents are seen. Expanding zone of
brawny edema, which jelly-like
fluctuates during shaking, surrounds
this affection..
b There is ulcer 11 cm with hard
yellowish bottom without purulent
content at the skin of leg. Ulcer is
circled by elevated intensively
hyperemic and sharply outlined rim
with cyanotic shade. Ulcer is sharply
painful.
c Moderate edema of skin. There is
crater-like ulcer under gray crust on
the highly infiltrated and hyperemic
base in the center of edema; pus is
excreted during pressing.
Lymphangitis is present. Touching of
ulcer is painful.
d Skin is markedly hyperemic, shining,
smoothed; a zone of affection has
precise festooned edges. On the
center of affection hyperemia is less
expressed, than on edges, where
bullas with serous content are seen.
517. For treatment of plague we have use all
antibiotics from mentioned below except of:
a Doxycycline;
b Chloramphenicole;
c Streptomycine;
d Cephasoline
e Gentamycine.
518.Wich two of the following methods of
laboratory investigation may be used for
express diagnostic of plague?
a culture of blood;
b *
direct immunofluorescent test;
c detection of specific IgM antibodies
with EIA (enzyme immunoassay);
d *
bacterioscopy with Grams
staining;
e detection of antibodies with indirect
hemagglutination test (IHAT).
519. Clinical manifestation of primary
pneumonic form of plague is characterized
with:
a an acute onset of disease, severe
signs of intoxication, sharp pain in
the chest from the 2-3 days of
disease, cough with abundant watery
bloodish sputum, dyspnea;
b an acute onset of disease, moderate
signs of intoxication, pain in the
chest from the 3-5 days of disease,
cough with scanty mucous rusty

57

sputum, dyspnea;
a gradual onset of disease, mild or
moderate signs of intoxication, cough
with scanty mucous bloodish sputum,
dyspnea;
an acute onset of disease, moderate
signs of intoxication, pain in the
chest from the 2-3 days of disease,
rare dry cough, dyspnea;
an acute onset of disease, expressed
signs of intoxication, mild pain in the
chest from the 4-6 days of disease,
cough with abundant purulent
sputum, dyspnea.

ANTHRAX

a) painful lymph nodes and rapid


purulence;
b) painful lymph nodes and absence of
purulence;
c) painless lymph nodes and absence
of purulence;
d) painless lymph nodes and rapid
prolipheration;
e) formation of periadenitis.
525. What is the most typical variant of
cutaneous form of anthrax?
a) erysipeloid-like;
b) edematous;
c) bullous;
d) carbunculous;
e) bubonic.
526. What is the basic treatment of
cutaneous anthrax?
a) bandage with Vishnevskys
ointment;
b) antibacterial therapy;
c) opening carbuncle and drainage;
d) surgical revision of carbuncule;
e) supportive therapy.
527. What is the drug of choose for
treatment of anthrax?
a Streptomycine;
b *Penicilline
c Tetracycline
d Laevomycetine;
e Erhythromycine.
528. For treatment of cutaneus anthrax we
may use all following means except of:
a Ceftriaxone;
b Penicillin;
c specific immunoglobulin;
d surgical removing of carbuncle;
e infusion of saline solutions.
528.What skin phenomena are typical for
cutaneous form of anthrax?
a There is on the hand painless ulcer
23 cm with a blackened necrotic
eschar circled with elevated
hyperemic rim, where small satellite
vesicles, filled by serous-hemorrhagic
contents are seen. Expanding zone of
brawny edema, which jelly-like
fluctuates during shaking, surrounds
this affection.
b There is ulcer 11 cm with hard
yellowish bottom at the skin of leg
without purulent content. Ulcer is
circled by elevated intensively
hyperemic and sharply outlined rim
with cyanotic shade. Ulcer is sharply
painful.
c Moderate edema of skin. There is
crater-like ulcer under gray crust on

517. What is the source of the infection with


anthrax?
f livestock;
g man;
h insects;
i river fish;
j birds.
518. What way of inoculation is most typical
for anthrax?
a) through respiratory tract;
b) through injured skin;
c) through gastrointestinal tract;
d) through conjunctiva;
e) all mentioned.
519. What kind of local inflammation is
typical for anthrax?
a) fibrinous;
b) productive;
c) serous-hemorrhagic;
d) purulent;
e) serous.
520. What is the content of anthrax
carbuncle cavity?
a) serous liquor;
b) pus;
c) blood;
d) all mentioned.
521. What is the main factor of anthrax
pathogenesis?
a autoallergy;
b immunosupression;
c toxemia;
d bacteremia;
e all mentioned;
522. What one of the following
manifestations is characteristic for anthrax
carbuncle?
a) presence of carbuncle, local edema
of tissue and acute pain;
b) presence of carbuncle, spread
edema of tissue, palpation is painless;
c) local hyperemia and edema of the
skin, fluctuation in the center and
acute pain;
d) spread hyperemia and edema of the
skin, palpation is painful.
523. What is the main method of anthrax
diagnostics?
a) bacteriological;
b) biological;
c) serological;
d) identification of toxin;
e) skin allergic test.
524. Which one of the following
manifestations is typical for anthrax regional
lymphadenitis?

58

the highly infiltrated and hyperemic


base in the center of edema; pus is
excreted during pressing.
Lymphangitis is present. Touching of
ulcer is painful.
Skin is markedly hyperemic, shining,
smoothed; a zone of affection has
precise festooned edges. On the
center of affection hyperemia is less
expressed, than on edges, where
bullas with serous content are seen

59

BRUCELLOSIS

b humans;
c birds;
d rodents;
e cattle.
535. Which food products do usually serve
as a source of brucellosis?
a *milk, cheese;
b salted fish;
c tinned meat;
d vegetables and fruit;
e eggs.
536. What are the basic clinical symptoms
of climax period of acute brucellosis?
a protracted remitting fever, chills,
significant sweating,
polylymphoadenitis, muscular pain,
enlargement of spleen and liver;
b constant type of fever, weakness,
adynamia, pale skin, poor roseolar
rush on the skin of abdomen,
enlargement of spleen and liver;
c fever of remitting type, chills,
significant weakness, petechial
enanthema on conjunctiva,
sometimes enlargement of liver;
d fever of intermitting type, arising of
fever is associated with paroxysm of
chills fallowed with feeling of heat
and subsequent plentiful sweating;
enlargement of spleen and liver;
e fever of hectic type, chills and
profuse sweating, pustular rash,
enlargement of spleen and liver.
537. The group of risk for brucellosis
includes:
a meat handlers;
b veterinarians;
c milkmaids;
d cattle-breeders;
e *all mentioned.
538. The drugs of choice for treatment of
brucellosis are the two from mentioned
below:
a Penicillin;
b Erythromycin;
c *Tetracycline;
d *Streptomycin;
e Ceftriaxone.
539. Clinical manifestations of chronic
brucellosis includes all mentioned below
signs except of:
a arthritis;
b endometritis;
c meningitis;
d orchitis;
e *eczematous dermatitis.
540. Clinical variants of chronic brucellosis
includes:

527. What kind of Brucellae is most


pathogenic for human?
a Br. abortus;
b Br. suis;
c *Br. melitensis;
d Br. canis;
e Br. neotomae;
528. What factors can influence on duration
of incubation period of brucellosis?
a pathogen activeness;
b quantity of infecting dose;
c way of transmission;
d reinfection;
e *all mentioned.
529. Which one of the following tests
reflects sensitivity of tissue to specific
brucellar antigen?
a) Coombss test;
b) Wrights test;
c) Heddlsons test.
d) *Burnets test.
e) All mentioned.
530. What is migration of brucellae?
a transfer of brucellae from one organ
to another of main host;
b transfer of brucellae from one animal
to another of the same species;
c *transfer of brucellae from main
animal host to another of different
species;
d all mentioned.
531. Portal of entry for brucellae into human
organism is (2):
a *dermal abrasions;
b *gastrointestinal tract;
c mucous membranes of respiratory
tract;
d urogenital tract
e all mentioned.
532. What laboratory test does allow to
diagnose an acute stage of brucellosis?.
a Burnets test;
b culture of bone marrow aspirate;
c detection of specific IgM antibodies
with EIA (enzyme immunoassay);
d detection of antibodies with indirect
hemagglutination test (IHAT) in titer
1:80;
e *all mentioned;
533. Does a cross-immunity after infection
with different types of brucellae possible?
a *yes;
b no.
534. What is the main reservoir of
brucellosis?
a *goats, sheep;

60

a
b
c
d
e

locomotor form;
urogenital form;
nervous form;
visceral form;
*all mentioned.

61

a
b
c
d
e

Q-FEVER

545. The main role in pathogenesis of Qfever belongs to:


a *
damaging of
reticuloendothelial tissue, ability of
causative agent to multiply in
macrophages;
b development of specific allergic
reactions;
c systemic vasculitis;
d tropism of causative agent to tissue
of liver and kidney;
e formation of necrotic foci in different
organs.
546. X-ray signs of Q-fever-pneumonia are
characterized with formation of:
a *
patchy interstitial
infiltrates;
b pleuritisl
c lobar consolidation of lung;
d abscesses;
e two-sided lobar pneumonia and
pleuritisl.
547. Clinical classification of Q-fever
includes all mentioned forms beside of:
a bronchopneumonia;
b typhoid-like form;
c meningoencephalitis;
d hepatitis;
e *
nephrosonephritis
548. Q-fever is geographically distributed in
a South America;
b Europe;
c Asia and Africa;
d North America;
e *
Worldwide.

537. What is the drug of choice for


treatment of Q-fever?
a) Penicillin;
b) *Tetracycline;
c) Furazolidone;
d) Streptomycin;
e) Chloroquine.
538. Which cardiovascular disturbances are
typical for climax period of Q-fever?
a *tachycardia and hypotension;
b relative bradycardia and moderate
hypotension;
c bradycardia and hypertension;
d tachycardia and hypertension;
e all mentioned.
539. How long is incubation period of Qfever?
a up to 5 days;
b *up to14 days;
c up to25 days;
d up to 45 days;
e up to 50days.
540. What is the basic method of laboratory
diagnostic of Q-fever?
culturing of blood;
*detection of specific antibodies in sera;
skin-allergic test;
culturing of feces;
culturing of cerebrospinal fluid.
541. Which one of the following systems is
typically affected with Q-fever?
a nervous system;
b cardiovascular system;
c *respiratory system;
d alimentary system;
e all mentioned.
542. Causative agent of Q-fever is:
a Leptospira icterohemorrhagia;
b Rikettsia prowazechii;
c *Coxiella burnetii;
d Rikettsia rikettsii;
e Chlamidia psittaci.
543. What is the reservoir of Q-fever?
a *
cattle;
b rodents;
c ticks;
d humans
e birds.
544. Most human infections with Q-fever are
acquired by (2):
a *
inhalation;
b bytes of insects;
c *
ingestion of contaminated
food;
d direct contact with ill patients;
e direct contact with ill animals.

TOXOPLASMOSIS
542. What is the source of infection with
toxoplasmosis?
a cats;
b birds;
c pigs;
d rabbits
e *
all mentioned.
543. Who are the definitive hosts of
toxoplasmosis?
a *
cats;
b birds;
c cattle;
d rabbits;
e all mentioned.
544.Who are the paratenic hosts of
toxoplasmosis?
a humans;
b rodents;
c pigs;
d birds;

62

e *
all mentioned.
545. The two main ways of transmission of
toxoplasmosis are those:
a air-droplet;
b *
alimentary;
c through the bites of insects;
d *
transplacental;
e through the skin/
546. The group of risk for toxoplasmosis
includes (2):
a aged persons;
b *
patients with HIVinfection;
c smocked persons;
d *
persons receiving
glucocorticoids;
e children under 1 year.
547. What is pseudocyst?
a It is oocyst, invaded epithelial cells of
the cats intestine;
b It is the group of female and male
gametes in the gut epithelial cells;
c *
It is the group of
infected macrophages containing
numerous bradyzoites, surrounded
by a thick wall;
d It is macrophages with intracellular
tachyzoites;
e All mentioned.
548. What form of T. gondii does usually
exist in infected organism of human in latent
form of toxoplasmosis?
a oocysts;
b gametes;
c tachyzoites;
d *bradyzoits;
e sporozoites.
549. What is the drug of choice for
treatment of toxoplasmosis?
a) Chloridin (Pyrimethamin);
b) Penicillin;
c Chloroquine (Delagil);
d Metronidazole (Trichopol);
e Laevomycetin.
550. Which combination of drugs is more
effective for treatment of toxoplasmosis?
a Fansidar and Metronidazole;
b Pyrimethamin and
Tetracycline;
c Fansidar and Spiromycin;
d Bisepthole and Clyndamycin;
e Chloroquine and tetracycline.
551. Toxoplasmosis usually causes in
neonates such (2) forms of disease as:
a osteomyelitis;
b *
meningoencephalitis;
c thyroiditis;
d glomerulonephritis;
e *
chorioretimitis.

552. Which tests from the mentioned below


may be used for diagnostics of
toxoplasmosis?
a culturing of the blood;
b identifying of IgM and IgG antibodies
with indirect fluorescent assay (IFA);
c microscopic examination of biopsy
material Giemsas stained;
d PCR;
e *
all mentioned.
553. Prevention of toxoplasmosis includes
two effective measures:
a *
avoiding of close contact
with cats;
b boiling of water;
c *
deep freezing of meat;
d deratization;
e killing of cats insects.

63

SEPSIS.

b complex antibiotic treatment is not


effective;
c glucocorticoids are not effective;
d immune therapy is not effective.
e *infusion therapy and glucocorticoids are
not effective
558. Most mediators of inflammation act as:
a activators of immune system;
b vasodilators;
c suppressors of immune system;
d factors increasing permeability of
microvessels;
e *vasodilators and factors increasing
permeability of microvessels.
559. Which one from following
immunocorrectors is most effective for
treatment of sepsis?
a Timalin;
b Interferon;
c Ronkoleikin;
d Inductors of endogenous interferon
(Amixin);
e *T-activin.
560. Intravenous drug-users more frequently
develop sepsis caused by:
a E. coli;
b Str. pneumonie;
c * S. aureus;
d S. enteritidisi;
e Ps. aeruginosa.

550. Choose possible causative agent of


sepsis from following:
a Sh. sonnei;
b *Str. pneumonie;
c Fr. tularence;
d R. prowazekii;
e M.pneumonie.
551. What is the main route of transmission
of sepsis?
a air-droplet;
b alimentary;
c transplacentar;
d contact;
e *activation of endogenous infection.
552. Is recovering from sepsis possible
without proper treatment?
a Yes, due to formation of stable immunity;
b Yes, unfavorable outcome possible only
among children and old persons;
c *No;
d Yes, unfavorable outcome possible only in
HIV infected persons.
553. Clinical sings of sepsis includes all
mentioned below except of:
a petechial rash;
b jaundice;
c high long temperature;
d *frequent liquid stool with admixture of
mucus and blood;
e hepatosplenomegaly.
554. What method of laboratory
investigation may be used as express one
for diagnosing of sepsis?
a *bacterioscopy of thick drop of Gram stained
blood;
b culture of blood;
c serological methods;
d immunofluorescent test;
e skin allergic test.
555. In case of staphylococcal sepsis the
most effective antibiotic is:
a Penicillin G in high dose;
b Cephalosporins of III generation;
c *Vancomycin;
d Macrolids of III generation;
e Tetracycline.
556. The main complications of sepsis are
all from mentioned below except of:
a ARDS;
b septic shock;
c acute renal failure;
d edema of the brain;
e *dehydrative shock.
557. Refractory septic shock it is
condition when:
a infusion therapy is not effective;

good example is the best sermon.

64

TESTS AND CLINICAL CASES FOR 6TH


YEAR STUDENTS

A 39-years-old patient was admitted to the


hospital on the 12th day of the disease.
Disease started with severe chill followed by
arising of temperature up to 39,5C, feeling
of heat, headache and vomit. Trough the 6
hours temperature critically decreased up to
36,5C with massive sweating. Trough the
one day all symptoms repeated and further
such attacks repeated with the same
periodicity. On examination: herpes of lips,
enlargement of spleen and liver are
presented, sclera are slightly icteric. The
cardiac sounds are weak. Common blood
test
presents
moderate
anemia,
leucopoenia,
lymphomonocytosis,
and
elevated ESR (21 mm /h).
a What is the diagnosis?
b Make a differential diagnostic.
c What are the possible urgent states
during treatment?
d Give the plan of laboratory
investigation.
e Prescribe a treatment.
CASE 4
The 22-year-old student developed a
disease
with
acute
onset,
febrile
temperature, frequent liquid stool with
admixture of slime and blood, spastic pain in
law parts of abdomen. Week ago he has
come back from village where the similar
cases of disease had been registered. It is
known that he has eaten unwashed apples
there. On admission to the clinic on the 4-th
day of disease a common state of patient
was moderate. Temperature was 37,5C. The
abdomen was moderately inflated, painful
during palpation of thick sigmoid bowel.
Frequency of stool numbered 10 times per
day; it was poor, liquid, with slime and
blood.
a Formulate the clinical diagnosis.
b Make a differential diagnostic.
c Make a plan of laboratory
investigation.
d What are possible complications and
urgent states during the current
disease?

CLINICAL CASES
CASE 1
The patient of 44-years-old has admitted to
the hospital on the 10-th day of disease with
complaints on common weakness, loss of
appetite, jaundice, darkening of urine. The
disease began gradually with fatigue,
nausea. On the 4th day he has noted
darkening of urine and the feeling of gravity
in the right upper part of abdomen. Current
state he had connected with exhaustion. On
the 8th day of disease the jaundice has
appeared and feces has lightened. On
admitting: common state is moderate,
temperature is 36,8C. Moderate jaundice of
sclera and skin was observed. Pulse rate
was 60 bites/min, BP 115/70 mm Hg.
Tongue was coated with white fur. Liver was
enlarged on 2 cm, its edge was acute and
surface was smooth. Palpation of liver was
slightly painful.
a What is the diagnosis?
b Make a differential diagnostics.
c Give a plan of laboratory
examinations and treatment.
d What outcomes of disease are
possible?
CASE 2
Patient was admitted to the hospital on the
3-d day of disease after the high
temperature (up to 39,0C), which was
accompanied with chills, frontal headache
and pain in muscle and joints within all this
period. He complained also of difficulties of
nasal breathing, serous secretion from the
nose, irritating nonproductive cough and
burning substernal discomfort. On
examination temperature was 38,2C,
injection of vessels of a sclera and eyelids
conjunctiva and hyperemia of soft palate
were observed. Isolated dry ronchi were
listened on auscultation over the lung.
a What is most likely diagnosis?
b Make a differential diagnostic.
c Make the plan of laboratory
examination.
d What urgent states are characteristic
for a current disease?
e What medical tactics are preferable?

CASE 5
A50-year-old
watchman
of
a
storehouse situated near a forest, developed
disease with acute onset, high temperature
(up to 39C) chills, headache, arthralgias,
followed by back pain from the 2-th day of
disease and by nasal bleeding from the 3-rd
day. He was brought to the emergency room
on the 5-th day of disease in severe state.
On examination delirium, tremor of limbs,
hyperemia of the face and neck and

CASE 3

65

abundant hemorrhagic rush on skin of a


trunk and extremities were noted. Diuresis
numbered 150 ml.
a Make your preliminary diagnosis.
b Make a differential diagnostic.
c Make
a
plan
of
laboratory
investigation.
d What urgent states did develop in
this case?
e What is your medical tactic?

respiratory failure. The disease began


gradually from sore throat, temperature of
37,8-38C
and
weakness.
Then
nonproductive cough, hoarseness of a voice
developed followed with aphonia, difficult
and noisy labored respiration. On admission
the patients state is severe. Cyanosis,
stridoric breathing with participation of
auxiliary muscles sweating, paroxysmal
tachycardia were noted. Respiratory rate
was 52/min. Cardiac sounds were dummy. In
oropharyngeal region cyanosis, edema and
hyperemia of a soft palate and tonsils were
registered; tonsils were covered with firm
grey membranes. During laryngoscopy
membranes of grey color were detected on
a mucous of larynx and then extracted.
a What is most likely diagnosis?
b Make diagnostics and treatment plan.
c What complication did develop in this
case?

CASE 6
The 25-years-old farmer presents on the 2th day of disease high temperature, chills,
headache, back pain and pain in calf
muscles. On examination his state was
moderate; hyperemia of face and sclera,
slight jaundice, enlargement of liver were
registered. Lien was palpated at an edge of
a rib arc. Tongue was dry, coated with brown
fur. The palpation of calf muscles was
painful. Urine was dark and feces were of
usual color. Diuresis numbered 600 ml.
a Make your preliminary diagnosis.
b Make a differential diagnostic.
c Make
a
plan
of
laboratory
investigation.
d What urgent states did develop in
this case?
e What is your medical tactic?

CASE 9
A 36-year old track driver was brought to
emergency room on the 4th day of disease in
a severe condition with complaints on
headache, sleeplessness and impossibility to
swallow liquids (liquid food, water and even
saliva). It was found out that at last month
on hunting he has caught the fox, brought it
to home, but fox has scratched him and
then escaped. On examination: there were
no bites, but multiple tracks of scratches on
the hands. The persons face was
hyperemic, with injection of scleras vessels;
exophthalmia, hypersalivation and an
abundant sweating were noted. Movements
with the lower extremities were restricted.
Periodic painful convulsive spasms of throat
and larynx muscles were registered,
accompanied with stenotic breathing and
expression of horror. The bright light, noise
of pouring water, movement of air or
attempt of swallowing of saliva provoked
such convulsive attacks. Pulse rate-160
beats/min. BP 140/100 mm Hg.
a Make your diagnosis. What is the
phase of the disease?
b Make a differential diagnostic.
c Make the plan of laboratory
investigation.
d What is your medical tactics?

CASE 7
The inhabitant of the Tyumen region
(Russia, West Siberia) used to eat a row fish
and a home made salted pike caviar within
last months noted common weakness,
dizziness, burning of tongue, pain in
muscles, loss of weight and discharging with
a feces particles of helminthes. The skin of
patient is pale; shin and feet are edematous.
The cardiac sounds are weak, systolic
murmur on the apex of heart is listened. BP
is 100/70 mm Hg. The liver is enlarged on 2
sm. Tongue is brightly red, with cracks.
CBT: erythrocytes 1,3 1012/l, hemoglobin
50 g/l, CI 1,2; in sample
megalocytes, poikilocytes, erythrocytes with
Jolis corpuscles and Kebots rings are
present.
a What is most likely diagnosis?
b Make diagnostics and treatment plan.
c What complication did develop in this
case?

CASE 10
A 11-year-old pupil of school was
admitted to the hospital on the 4th day of
disease with complaints on lack of motions
in the lower extremities and sharp pain in

CASE 8
A 45-year-old train conductor was
brought to emergency department on the
7th day of disease with the signs of acute

66

their muscles, high temperature, which


began to reduce. With drop of temperature
the pain in the lower extremities, loins, arms
and neck has aggravated. Within several
hours after admission the flaccid paralyses
have appeared in the muscles of legs, arms
with a dominance of a proximal affection of
extremities (femur, shoulders). The function
of sphincters is reduced. The hypotension of
muscles, absence of reflexes is followed with
coldening and cyanosis of legs. The
sensitivity is saved.
a Make your diagnosis accounting
phase of disease.
b Make the differential diagnosis.
c Make the plan of laboratory
diagnostics.
d What complications are possible?
e Your therapeutic tactics?

during
swallowing,
high
temperature,
sweating. The disease began with malaise,
subfebrile temperature, and sore throat.
Frequent attacks of tonsillitis in past history
were noted. On examination: temperature
37,7C. Skin is pale without a rush. Mucous
of throat is moderately hyperemic. The
tonsils are enlarged in size, edematous,
without purulent membranes. Axillary,
submandibular, neck posterior lymph nodes
are enlarged in size (0,8 1 cm), slightly
painful, mobile. Cardiac sounds are clear,
rhythmic and pulse rate is 94 beats/min. On
auscultation over the lung vesicular sound is
listened. Tongue is wet, coated with white
fur. Abdomen is soft, painless. The liver
enlarged on 1 cm.
a What is preliminary diagnosis?
b Make differential diagnostic.
c Make the plan of laboratory
investigation.
d What complications are possible?
e What is your medical tactics?

CASE 11
A
48-year-old
wool-sorter
was
admitted to clinic on the 2d day of disease
in severe condition with complaints on
dyspnea, expectoration of hemorrhagic
sputum (about 1 l/day), and pain in the
chest, connected with breathing. The
disease began suddenly with chill, weakness
and cough, at first dry and then with
expectoration of a great amount of serous
and serous-hemorrhagic sputum. It is known
that during last time he accepted and
unpacked bales with a wool delivered from
other countries. Among the employers cases
of
influenza
were
registered.
On
examination: general condition of patient
was severe, consciousness was saved, but
sometimes patient was inadequate. Skin
was cyanotic and wet. A mucous of
nasopharyngeal
region
was
slightly
hyperemic. Tongue was dry, coated with
dirty fur. Respiratory rate accounted 40/min.
On
percussion
over
the
lungs
in
interscapular
space
shortening
of
a
pulmonary sound was registered, on
auscultation a lot of dry and varied wet rales
were listened. Pulse rate was 120 beats/min,
blood pressure 80/40 mm Hg. Spleen was
palpated at edge of a rib arc.
a
What is preliminary diagnosis?
b
Make a differential diagnostic.
c
Make the plan of laboratory
investigation.
d
What complications are typical for
the disease?
CASE 12
A 18-year-old patient presents on the
8th day of the disease complaints on
common fatigue, headache, pain in throat

CASE 13
A 30 year-old nurse of a summer children
camp was admitted to hospital on the 3d
day of the disease with the complaints on
pain in throat, high temperature and
frequent liquid stool. Nine teenagers from
summer camp with similar symptoms were
hospitalized
in
department.
Bright
hyperemia of mucous of throat and soft
palate, palatine arcs, tonsils and back wall
of a pharynx were detected. There are a
group of shallow vesicles with transparent
contents, enclosed by a red crown on a
mucous of soft palate, which in 2-3 days
burst with formation of erosion. The sore
throat was strengthened during swallowing.
On the 7th day the fever has decreased,
epithelizationof erosions was observed.
General condition was improved, stool
normalized.
a What is most likely diagnosis?
b Make the differential diagnosis.
c Make the plan of laboratory
diagnostics.
d What is your medical tactic?
CASE 14
A 38-year-old female patient
developed disease with acute onset, arising
of temperature up to 38C, headache,
weakness and nausea. To the end of the 3d
day pain in left eye, swelling of eyelids with
purulent discharging from eye has appeared
followed with swelling and pain in the field
of left ear and neck.

67

It is known that last 2,5 weeks she worked


with thresher. On examination: skin of face
is hyperemic, dry. Pulse is rhythmic, 90
beats/min. Eyelids of the left eye are
edematous, in the angle of left eye white
pus is seen. The conjunctiva is sharply
edematous; there are plenty of small ulcers
with purulent content here. Asymmetry of
the face and neck at the expense of
enlarged subauricular and posterior neck
lymph nodes is seen. Lymph nodes are
painful, tough, with clear contours and
movable. Mucous of throat is moderately
hyperemic. Tonsils are not enlarged.. Tongue
is dry, coated with fur. Abdomen is soft,
painless during palpation. Liver and lien are
not palpated.
The CBT reveals neutrophilic leucocytosis;
ESR is 25 mm/h.
a What is most likely diagnosis?
b What are the methods of laboratory
diagnostics and therapy?

One week ago he has wounded the right


foot, and received prophylactic treatment
with tetanus antitoxin.
On examination on the 7th day of the
disease: temperature is 38,2C. There is
macular and papular itching rash on the skin
of trunk, arms, abdomen and lower
extremities. Face and fingers of hands are
edematous and pale. Disseminated
lymphoadenopathy is found. Tachycardia
and hypotension are registered.
In CBT: leucopoenia with a relative
lymphocytosis and accelerated ESR.
a What is most likely diagnosis?
b Make
the
plan
of
differential
diagnostic.
c What is your therapeutic tactics?
A
positive

divorced
for

CASE 17
mother

HIV-1

of

single

infection

test

during

investigation of a febrile illness presents

CASE 15
A 44-years-old female patient was admitted
to the hospital at the end of May with the
complaints on a strong headache of
pulsating character, photophobia, general
muscle pains, chill, fever, sleeplessness,
lack of appetite. It was found out that 8 days
before she has come back from Western
Siberia, where she had lived in village near
wood during 10 days. She has neither
contact with ill people, animals, nor bites of
mosquitoes and ticks.
On examination on the 3d day of disease:
temperature was 39,0C, consciousness was
clear, pulse was rhythmic, 80 beats/min, the
cardiac sounds were muffled. Over the lung
vesicular breathing was listened. Abdomen
was soft and painless on palpation. Mucous
of the throat was hyperemic. A left-sided
neck lymphadenitis was present. In left
parietal part of the head a crust of brown
color on edematous base painful on
palpation was seen. Slight neck rigidity was
present. Reflexes of pupils on light were
week, convergence was broken, mild
asymmetry of the face was noted.
a What is most likely diagnosis?
b Make
the
plan
of
differential
diagnostic.
c What is your therapeutic tactics?

disseminated lymphoadenopathy. A second


enzyme immunoassay (EIA) is performed,
and the result is the same. The woman
denies intravenous drug use. She has dated
several men since her divorce and cannot
be positive about their sexual habits or use
of intravenous drugs.
a

What is most likely diagnosis

What is the appropriate next step


in the management of this patient?

Make

plan

of

differential

diagnostic.
CASE 18
A 32-year-old patient ill with severe acute
tonsillitis was brought to the hospital and
treatment with Penicillin and Sulfodoxine
was undertaken. Soon the patient felt a
numbness of tongue, lips, face; giddiness,
headache, nausea, shortness of breath has
appeared.
On examination: patient is excited, his face
is hyperemic, sweated; cyanosis and
macular inching rush are present. Pulse is
rapid and thread-like. BP 40/0 mm Hg.
Cardiac sounds are dummy, the 2nd tone on
a pulmonary arteria is accentuated. The
respiration is frequent and superficial. Over
the lungs box-like pulmonary sound, and

CASE 16
A 40-year-old metal-turner on admission to
the hospital presents complains of
headache, strong weakness, fever, pain in
joints, itching abundant rush on skin of body.

68

numerous dry whistling rales were found.


Involuntary urination and defecation were
noted.
a What is most likely diagnosis?
b Make a treatment plan?

First step treatment measures.

CASE 21
A 61-year-old pensioner within 5 days
presents complaints on headache, insomnia,
chill, fever 39.3C, and weakness. Treatment
with antipyretics was not effective. Today on
skin of trunk roseolar and petechial rush
come out. It is known that 35 years ago he
had been ill with louse-borne typhus and
malaria and now he suffers from diabetes.
On examination: patient is slightly excited,
his face is edematous and hyperemic, eyes
glitter. There is enanthema on soft palate.
Not numerous roseolar and petechial rush
predominantly on the lateral parts of trunk
and internal surfaces of upper extremities is
seen. Moderate dyspnea is present. On
auscultation over the lung rough breathing
is listened. Cardiac sounds are dull, systolic
murmur on top projection. BP is 100/60 mm
Hg. Tongue is coated with fur, trembling
during examination. Abdomen is soft,
painless, liver and spleen are enlarged on 2
cm.
a What is the diagnosis?
b Make differential diagnostic.
c What complications are possible?
d Make plan of laboratory diagnostics
and treatment.

CASE 19
A 37-year-old veterinarian within a month is
complaining of weakness, sweating, pain in
knees, foot joints and subfebrile
temperature. Repeatedly sought for medical
help, was treated with antipyretics but
without effect. In the farmer where he works
cases of abortions in sheep were registered
On examination: a common patients state
is satisfactory, temperature is 37,6 C. Skin
and visual mucous are of usual color. Large
joints are edematous and painful on
palpation and motion; there are dense
painful nodules in a hypodermic fat near
joints, tendons and muscles. The cardiac
sounds are rhythmic, dull. Pulse rate is
80 /min, BP 120/80 mm Hg.
Abdomen is soft and painless on palpation.
The edge of a liver is palpated on 1,5 cm
below of a rib arc. The lien is not enlarged.
Stools and urine passage are normal.
a Make your presumable diagnosis.
b Make a plan of differential diagnostic.
c Make a plan of laboratory diagnostic.
CASE 20
A 21-year-old schoolteacher developed
acute disease with fever up to 38,5C,
running
nose,
nonproductive
cough,
photophobia and lacrimation. On the 3 rd day
temperature increased to 39.5C, headache
and nausea intensified and patients
condition deteriorated; on the next day rush
stood out on the face, neck, upper part of
thorax and then spread on whole body.
On examination on the 5th day: temperature
is 39.5C, patient is languid, adynamic. Face
and eyelids are hyperemic and edematous.
Conjunctives are hyperemic, injection of
scleras vessels is observed. There is
abundant macular and papular rush
sometimes merged on the skin of body and
face. Throat and palatal mucous is
hyperemic; on soft palate enanthema is
seen. On chick mucous, opposite molars,
small spots of white fur are observed.
Disseminated lymphoadenopathy is found.
During auscultation rough breathing is
listened. Cardiac sounds are moderately
dull, pulse rate is 92/min..
a What is your diagnosis?
b Make a differential diagnosis
c What complications are possible?

CASE 22
A 35-year-old milkmaid within 2 weeks
presents remitting fever with every day
chills and then profuse sweating, headache,
weakness, and pain in joints. Paroxysms of
temperature appeared in the afternoon.
Treatment with penicillin and antipyretics
was not effective. Despite of high
temperature, common condition of the
patient remained rather satisfactory.
It is known that during last year she has
contact with ill animals during calving.
On examination: temperature is 38.5C. Skin
is slightly cyanotic, without rush. Cervical
and axillary lymphoadenopathy is present.
Over the lungs vesicular breathing is
listened. The cardiac sounds are muffled
and rhythmic. Pulse rate is 80 beats/min.
Tongue is wet and pure. Abdomen is soft
and painless; edge of a liver is palpated on
1m below of rib arc. The lien is not
palpated. Stools and waterworks is normal
a What is most likely diagnosis?
b Make a differential diagnostic.
c Make the plan of laboratory
diagnostic and treatment.

69

CASE 23
A 25-year-old hunter was brought to the
emergency department in a severe state on
the 3rd day of disease. Onset of the disease
was acute with chill, high temperature (up to
39.0-39.5C), headache, vomiting, pain in
muscles, and pain in right inguinal region
followed with hallucinations. Patient is
accurately vaccinated. According to his
work, he most part of time spent in a wood
and steppe.
On examination: temperature is 39C, pulse
is weak, 140 beats/min, BP 80/40 mm Hg.
Patients consciousness is confused. Skin of
face is hyperemic and cyanotic. Cardiac
sounds are dull, respiratory rate account
26/min. Over the lung vesicular respiration
is listened. Tongue is dry and coated with
white fur. The abdomen is soft, painless. The
right leg is half bent. In inguinal area the
formation with indistinct contours of 15 25
cm in size is visually defined. Skin above it is
of blue-crimson color. During palpation of
this conglomerate the patient reacts with a
groan, tries to repel a hand of the doctor.
a What is the most likely diagnosis?
b What urgent state was developed in
this case?
c Make your tactics of treatment.

a
b

What is the most likely diagnosis?


What is your therapy?

CASE 25
A 35-year-old businessman, which week ago
has come back from trip in Guinea, was
brought to the emergency department in
severe state. Within 7 days he complained
of fever, sharp headache, and chills.
Temperature was high (39,0-40,0C) and
only sometimes on mornings was normal.
Patient was taking Analgin but had no effect:
the headache accrued, sleepiness appeared,
and then exaltation, unconsciousness state,
and cramps developed. It was found out that
month ago he has been ill with malaria.
On examination: patients state is severe.
Unconsciousness, mild neck rigidity and
periodical tonic and clonic seizures are
present. Reflexes of pupils on light are week;
miosis, increasing of tendinal reflexes, and
Chain-Stokss breathing are observed. Skin
and visual mucous are subicteric. Pulse is
weak, 130 beats/min, BP 90/40 mm Hg.
On auscultation of lungs heterogeneous wet
crackles are listened. The lien is illegibly
palpated.
a What is the most likely diagnosis?
b What agent does cause this disease?
c What is the form of disease?
d What is your therapeutic tactics?

CASE 24
A 21-year-old patient within 7 days presents
complaints on chill, febrile temperature,
headache, common weakness, and sore
throat. Treatment with Tetracycline and
Aspirin was not effective.
On examination: Patient was sluggish. Skin
was pale, without rush. Mucous membrane
of a throat is hyperemic; tonsils are enlarged
and covered with membranes of greenyellow color. Axillary, anterior and posterior
neck lymph nodes were enlarged, slightly
painful during palpation. Pulse was
rhythmic, 108 beats/min. The cardiac
sounds are dull. Harsh breathing is listened
on auscultation of the lung. Tongue is dryish,
coated with white-grey fur. Abdomen is soft,
slightly painful in right part of epigastria on
palpation. Lower edge of liver is palpated on
2 cm below of a rib arc. Spleen is palpated
at a level of a rib arc.
The CBT shows: erythrocytes 4 1012/l,
leukocytes 10 109 /l, drum stick 8 %,
segmented neutrophiles 32%,
lymphocytes 51 %, monocytes 6 %,
plasmatic cells 3%. ESR 27 mm/h. Some
lymphocytes have wide and vacuolated
cytoplasm.

CASE 26
A 26-year-old patient within 3 days
developed weakness, temperature of 38C,
followed with difficulty of swallowing of a
saliva and water and hypersalivation.
It was found out, that 4 months ago during
hunting he has been bitten by the fox in the
right hand and has not been treated with
vaccine.
On examination: patients state is severe,
temperature is 38.8C, face is hyperemic
and sweaty. In a reception ward at the
attempt to swallow water attack of painful
spasms of pharyngeal and laryngeal
muscles developed, that was accompanied
by excitement and disorders of respiration.
With cry throws from itself a cup of water,
throws back head and trunk. The face is
distorted, cyanotic, expresses horror, the
pupils are dilated, view stuck in one point.
The attack lasted some seconds. Then the
attack has repeated after a loud sound and
switching of light. Lethal outcome had
occurred during the next fit.

70

a
b
c

What is most likely diagnosis?


What is the period of diseases?
What medical help was need in this
case?
CASE 27
35-year-old migrant

worker

temperature up to 37.8C and general


weakness, with marked worsening of the
general state to the evening: temperature
arose up to 39C, severe headache,
vomiting, absence of effect from analgetics,
disorders of conscious.
a What is the most likely diagnosis?
b What is the level of consciousness
disorder?

is

admitted to the hospital with a high fever


and malaise on the 8 day of disease. When
he started feeling weak he checked his

CLINICAL TASKS

temperature and found it to be slightly

1. Chose symptoms, typical for icteric form


of viral hepatitis B:
a Gradual onset from nausea, gradual
loss of appetite, increasing itching of
skin. OE: jaundice of skin, with
scratches traces. Skin turgor is
reduced. Pulse is 86 /min; sizes of
liver by Kurlov: 13128 cm, in gallbladder projection elastic painless
formation 33 cm is palpated. Spleen
is not enlarged.
b Acute onset from nausea, pain in a
right part of abdomen, increasing of
fever up to 38C. OE: moderate
jaundice; tachycardia. Sizes of liver
by Kurlov: 12127 cm., palpation in
gall-bladder projection is painful.
Spleen is not enlarged.
c Gradual onset from nausea, pain in
joints, itch of skin, and loss of
appetite. OE: jaundice of skin, its
turgor is normal. Pulse is 68 /min.
Sizes of liver by Kurlov: 13128 cm.,
edge of liver is sensitive on
palpation. Spleen is not enlarged.
d Acute onset with chill, fever up to
38,8C, nausea, dizziness, weakness.
OE: moderate jaundice, temperature
is 39C. Tachycardia up to 122 /min.
Sizes of liver by Kurlov: 11108 cm.,
its edge is painless. Spleen is
palpated on 3 cm. below left costal
arch. Anuria within 10 hours.
2. Chose symptoms, typical for cholelitiasis:
a Gradual onset from nausea, gradual
loss of appetite, increasing itching of
skin. OE: jaundice of skin, with
scratches traces. Skin turgor is
reduced. Pulse is 86 /min; sizes of
liver by Kurlov: 13128 cm, in gallbladder projection elastic painless
formation 33 cm is palpated. Spleen
is not enlarged
b Acute onset from nausea, pain in a
right part of abdomen, increasing of
fever up to 38C. OE: moderate
jaundice; tachycardia. Sizes of liver

elevated. Over the last 4 days he has felt


progressively worse and his temperature
has reached 39 C. The resident on call in the
emergency room observes a scanty roseolar
rash

on

patients

trunk.

The

patient

mentions that another worker in his group


has been sick wit similar symptoms.
a
b
c

What is most likely diagnosis?


Make a plan of differential diagnostic.
Make a plan of laboratory
investigation and treatment.

CASE 28
A 18-year-old male was admitted to the
hospital on the 3rd day of the disease. On
the 1st day his temperature was 37.8C, and
he felt irritation in the throat, sneezing and
mucous secretion from the nose. On the 2nd
day temperature arose up to 38.8C and
strong headache, increasing to night and
single vomiting appeared. Next morning
patient developed cramps and became
disoriented. On examination: patients
common state is grave; productive contact
with him is impossible. Meningeal symptoms
are expressed. During examination patient
pushes away doctors hand. He periodically
develops clonic and tonic cramps. His face is
edematous
and
hyperemic.
Moderate
hyperemia of posterior wall of a throat is
seen. Pulse rate is 120 beats/min, BP
140/90 mm Hg, respiratory rate 30/min.
a What is the most likely diagnosis?
b What is the level of consciousness
disorder?
c What complication has develop in
this case?
CASE 29
A 30-year-old male was admitted on the
2nd day of the disease at night after acute
beginning of disease with arise of

71

by Kurlov: 12127 cm., palpation in


gall-bladder projection is painful.
Spleen is not enlarged.
c Gradual onset from nausea, pain in
joints, itch of skin, and loss of
appetite. OE: jaundice of skin, its
turgor is normal. Pulse is 68 /min.
Sizes of liver by Kurlov: 13128 cm.,
edge of liver is sensitive on
palpation. Spleen is not enlarged.
e . Acute onset with chill, fever up to
38,8C, nausea, dizziness, weakness.
OE: moderate jaundice, temperature
is 39C. Tachycardia up to 122 /min.
Sizes of liver by Kurlov: 11108 cm.,
its edge is painless. Spleen is
palpated on 3 cm. below left costal
arch. Anuria within 10 hours.
3. Chose symptoms, typical for tumor of
head of pancreas:
a Gradual onset from nausea, gradual
loss of appetite, increasing itching of
skin. OE: jaundice of skin, with
scratches traces. Skin turgor is
reduced. Pulse is 86 /min; sizes of
liver by Kurlov: 13128 cm, in gallbladder projection elastic painless
formation 33 cm is palpated. Spleen
is not enlarged
b Acute onset from nausea, pain in a
right part of abdomen, increasing of
fever up to 38C. OE: moderate
jaundice; tachycardia. Sizes of liver
by Kurlov: 12127 cm., palpation in
gall-bladder projection is painful.
Spleen is not enlarged
c . Gradual onset from nausea, pain in
joints, itch of skin, and loss of
appetite. OE: jaundice of skin, its
turgor is normal. Pulse is 68 /min.
Sizes of liver by Kurlov: 13128 cm.,
edge of liver is sensitive on
palpation. Spleen is not enlarged
d Acute onset with chill, fever up to
38,8C, nausea, dizziness, weakness.
OE: moderate jaundice, temperature
is 39C. Tachycardia up to 122 /min.
Sizes of liver by Kurlov: 11108 cm.,
its edge is painless. Spleen is
palpated on 3 cm. below left costal
arch. Anuria within 10 hours..
4. What pathogenetic mechanisms are
typical for cholera? (2)
a)
Dehydration, hypovolemia,
arterial pressure decreasing, reduction of
renal filtration;
b)
Toxemia, disturbance of
peripheral microcirculation;

c)
Dehydration, inflammatory
changes in intestine, metabolic acidosis,
lack of potassium in blood;
d) Increasing of water and electrolytes
secretion as a result of activation of
adenilatcyclaze and accumulation of
cAMP.
5. What basic clinical symptoms are
characteristic for colitic form of shigellosis?
a)
Gradual onset, absence of
intoxication, moderate spastic pains in lower
part of abdomen, alternation of constipation
and diarrhea, sometimes mucus in feces;
b)
Acute onset with sharp pain in
abdomen, symptoms of intoxication at first
are absent; stool is bloody, frequently
without feces;
c)
Acute onset, moderate
intoxication, diffuse pain in abdomen,
vomiting, then liquid stool with admixture of
mucus (and sometimes of blood);
d)
Acute onset, moderate
intoxication, spastic pain in left hypogastric
region, scanty stool with mucus and blood.
6. What are the basic clinical symptoms of
cancer of rectum?
a)
Gradual onset, absence of
intoxication, moderate spastic pains in lower
part of abdomen, alternation of constipation
and diarrhea, sometimes mucus in feces;
b)
Acute onset with sharp pain in
abdomen, symptoms of intoxication at first
are absent; stool is bloody, frequently
without feces;
c)
Acute onset, moderate
intoxication, diffuse pain in abdomen,
vomiting, then liquid stool with admixture of
mucus (and sometimes of blood);
d)
Acute onset, moderate
intoxication, spastic pain in hypogastric
region, scanty stool with mucus and blood.
7. What are the basic clinical symptoms of
thrombosis of mesenteric vessels?
a)
Gradual onset, absence of
intoxication, moderate spastic pains in lower
part of abdomen, alternation of constipation
and diarrhea, sometimes mucus in feces;
b)
Acute onset with sharp pain in
abdomen, symptoms of intoxication at first
are absent; stool is bloody, frequently
without feces;
c)
Acute onset, moderate
intoxication, diffuse pain in abdomen,
vomiting, then liquid stool with admixture of
mucus (and sometimes of blood);
d)
Acute onset, moderate
intoxication, spastic pain in left hypogastric
region, scanty stool with mucus and blood.

72

8. What are the basic clinical symptoms of


climax period of malaria vivax?
a Remittent fever, chills, significant
sweating, lympadenopathy, muscular
pain, enlargement of spleen and
liver;
b Constant type of fever, weakness,
adynamia, pale skin, scanty roseolar
rush on lateral surfaces of trunk,
enlargement of spleen and liver;
c Fever of remitting type, chills,
significant weakness, petechial
enanthema on conjunctiva,
sometimes liver is enlarged;
d Fever of intermitting type, on climax
of fever feeling of heat with
subsequent plentiful sweating,
enlargement of spleen and liver..
9. What are basic clinical symptoms of
climax period of acute brucellosis?
a Remittent fever, chills, significant
sweating, lympadenopathy, muscular
pain, enlargement of spleen and
liver;
b Constant type of fever, weakness,
adynamia, pale skin, scanty roseolar
rush on lateral surfaces of trunk,
enlargement of spleen and liver;
c Fever of remitting type, chills,
significant weakness, petechial
enanthema on conjunctiva,
sometimes liver is enlarged;
d Fever of intermitting type, on climax
of fever feeling of heat with
subsequent plentiful sweating,
enlargement of spleen and liver..
10. What are basic clinical symptoms of
climax period of staphylococcal sepsis?
a Remittent fever, chills, significant
sweating, lympadenopathy, muscular
pain, enlargement of spleen and
liver;
b Constant type of fever, weakness,
adynamia, pale skin, scanty roseolar
rush on lateral surfaces of trunk,
enlargement of spleen and liver;
c Fever of remitting type, chills,
significant weakness, petechial
enanthema on conjunctiva,
sometimes liver is enlarged;
d Fever of intermitting type, on climax
of fever feeling of heat with
subsequent plentiful sweating,
enlargement of spleen and liver.
11. What symptoms are most typical for
staphylococcal food poisoning?
a)
Acute onset with chill,
vomiting, short-term fever, diffuse

abdominal pain, frequent liquid stool with


greenish stool up to 15-20 times per day;
b)
Acute onset with chill,
repeated vomiting, sharp pains in epigastria,
frequent development of collapse, liquid
stool up to 4-5 times per day, fast
convalescence;
c)
Acute onset with liquid watery
stool, then vomiting; normal body
temperature, absence of abdominal pain,
oliguria;
e)
Gradual onset with long high
fever, increasing weakness, pale skin, from
the 2-d week of disease liquid stool up to
2-4 times per day.
12. What symptoms are most typical for
gastrointestinal salmonellosis?
a)
Acute onset with chill,
vomiting, short-term fever, diffuse
abdominal pain, frequent liquid stool with
greenish stool up to 15-20 times per day;
b)
Acute onset with chill,
repeated vomiting, sharp pains in epigastria,
frequent development of collapse, liquid
stool up to 4-5 times per day, fast
convalescence;
c)
Acute onset with liquid watery
stool, then vomiting; normal body
temperature, absence of abdominal pain,
oliguria;
e)
Gradual onset with long high
fever, increasing weakness, pale skin, from
the 2-d week of disease liquid stool up to
2-4 times per day.
13. What symptoms are most typical for
moderate form of cholera?
a)
Acute onset with chill,
vomiting, short-term fever, diffuse
abdominal pain, frequent liquid stool with
greenish stool up to 15-20 times per day;
b)
Acute onset with chill,
repeated vomiting, sharp pains in epigastria,
frequent development of collapse, liquid
stool up to 4-5 times per day, fast
convalescence;
c)
Acute onset with liquid watery
stool, then vomiting; normal body
temperature, absence of abdominal pain,
oliguria;
e)
Gradual onset with long high
fever, increasing weakness, pale skin, from
the 2-d week of disease liquid stool up to
2-4 times per day.
14. What symptoms are typical for typhoid
fever?
a Acute onset with fast increasing of
temperature within 1-2 days up to
39-40C; persistent headache,
sleeplessness, euphoria. Face is

73

hyperemic, sclera are injected,


spleen is increased since the 3-4
days from onset of disease;
b Gradual onset with slow (within 4-5
days) increasing of body
temperature, general weakness,
sleeplessness, pale skin; tongue is
thickened, with teeth prints, relative
bradycardia, spleen is enlarged since
the 6-7 days from onset of disease;
c Acute onset; within several hours
body temperature reaches 38-39C,
is accompanied with chills, headache
in forehead and temple area, pain in
eyes, nasal obstruction, dry cough,
tracheal pain; spleen is not enlarged;
d)
Acute onset with chill, fast
increasing of temperature up to 39-40C;
weakness, sweating, myalgias, excitation,
headache, sometime dry cough, hyperemia
of face, injection of sclera vessels, retro
orbital pain, sometimes palatal enanthema,
enlargement of spleen is quite often. In WBC
leucopoenia, neutropoenia.
15. What symptoms are typical for initial
period of louse-born typhus?
a)
Acute onset with fast
increasing of temperature within 1-2 days
up to 39-40C; persistent headache,
sleeplessness, euphoria. Face is hyperemic,
vessels of sclera are injected, spleen is
enlarged since the 3-4 days from onset of
disease, after 4-5 days occurrence of
roseolar-petechial rash on trunk.
b)
Gradual onset with slow
(within 4-5 days) increasing of body
temperature, general weakness,
sleeplessness, pale skin; tongue is
thickened, with teeth prints, relative
bradycardia, spleen is enlarged since the 67 days from onset of disease;
c)
Acute onset; within several
hours body temperature reaches 38-39C, is
accompanied with chills, headache in
forehead and temple area, pain in eyes,
nasal obstruction, dry cough, tracheal pain;
spleen is not enlarged;
d)
Acute onset with chill, fast
increasing of temperature up to 39-40C;
weakness, sweating, myalgias, excitation,
headache, sometime dry cough, hyperemia
of face, injection of sclera vessels,
retroorbital pain, sometimes palatal
enanthema, enlargement of spleen is quite
often. In WBC leucopoenia, neutropoenia.
16. What symptoms are typical for initial
period of acute Q-fever;
a)
Acute onset with fast
increasing of temperature within 1-2 days

up to 39-40C; persistent headache,


sleeplessness, euphoria. Face is hyperemic,
vessels of sclera are injected, spleen is
increased since the 3-4 days from onset of
disease;
b)
Gradual onset with slow
(within 4-5 days) increasing of body
temperature, general weakness,
sleeplessness, pale skin; tongue is
thickened, with teeth prints, bradycardia,
spleen is enlarged since the 6-7 days from
onset of disease;
c)
Acute onset; within several
hours body temperature reaches 38-39C, is
accompanied with chills, headache in
forehead and temple area, pain in eyes,
nasal obstruction, dry cough, tracheal pain;
spleen is not enlarged;
d)
Acute onset with chill, fast
increasing of temperature up to 39-40C;
weakness, sweating, myalgias, excitation,
headache, sometime dry cough, hyperemia
of face, injection of sclera vessels, retro
orbital pain, sometimes palatal enanthema,
enlargement of spleen is quite often. In WBC
leucopoenia, neutropoenia..
17. Choose symptoms, typical for
diphtheria:
a)
Acute onset, general
headache, pain in throat, fever 38 -40C,
disseminated lymphoadenopathy, mucous
of throat is hyperemic; there are easy
removed white-yellow purulent membranes
in lacunas of tonsils; enlargement of spleen;
b)
Acute beginning, strong
headache, pain in throat on swallowing,
fever 39-40C, mucous of throat is
hyperemic; there are easy removed grayyellow purulent membranes in lacunas of
tonsils; jugular lymph nodes are increased
and sharply painful;
c)
Acute beginning, moderate
pain in a throat, fever up to 38C, mucous of
throat is hyperemic, there are white colored,
hardly removed membranes on tonsils;
slightly increased and moderately painful
jugular lymph nodes;
d)
Acute onset, fever up to 3840C, mucous of throat is moderately
hyperemic, there is crater-like ulcer with
purulent content on left tonsil; regional
lymph nodes are slightly enlarged.
18. Choose symptoms, typical for infectious
mononucleosis:
a)
Acute onset, general
headache, pain in throat, fever 38-40C,
disseminated lymphoadenopathy, mucous
of throat is hyperemic; there are easy

74

removed white-yellow purulent membranes


in lacunas of tonsils, that more than 7-14
days does not disappear; enlargement of
spleen;
b)
Acute beginning, strong
headache, pain in throat on swallowing,
fever 39-40C, mucous of throat is
hyperemic; there are easy removed grayyellow purulent membranes in lacunas of
tonsils; jugular lymph nodes are increased
and sharply painful;
c)
Acute beginning, moderate
pain in a throat, fever up to 38C, mucous of
throat is hyperemic, there are white colored,
hardly removed membranes on tonsils;
slightly increased and moderately painful
jugular lymph nodes;
d)
Acute onset, fever up to 38 40C, mucous of throat is moderately
hyperemic, there is crater-like ulcer with
purulent content on left tonsil; regional
lymph nodes are slightly enlarged.
19. Choose symptoms, typical for acute
tonsillitis:
a)
Acute onset, general
headache, pain in throat, fever 38 -40C,
disseminated lymphoadenopathy, mucous
of throat is hyperemic; there are easy
removed white-yellow purulent membranes
in lacunas of tonsils; enlargement of spleen;
b)
Acute beginning, strong
headache, pain in throat on swallowing,
fever 39-40C, mucous of throat is
hyperemic; there are easy removed grayyellow purulent membranes in lacunas of
tonsils; jugular lymph nodes are increased
and sharply painful;
c)
Acute beginning, moderate
pain in a throat, fever up to 38C, mucous of
throat is hyperemic, there are white colored,
hardly removed membranes on tonsils;
slightly increased and moderately painful
jugular lymph nodes;
d)
Acute onset, fever up to 38 40C, mucous of throat is moderately
hyperemic, there is crater-like ulcer with
purulent content on left tonsil; regional
lymph nodes are slightly enlarged.
20. What symptoms are typical for initial
period of typhoid fever?
a)
Acute beginning with
increasing of body temperature within 1-2
days up to 39-40C, persistent headache,
sleeplessness, anxiety, euphoria. Face is
hyperemic, vessels of sclera are injected,
spleen is enlarged since the 3-4 day of
disease;

b)
Gradual onset with slow
(within 4-5 days) increase of temperature,
general adynamia, sleeplessness, pale skin,
tongue is thickened, with prints of teeth,
relative bradycardia, spleen is enlarged
since the 6-7 day of disease;
c)
Acute beginning of disease,
within several hours temperature reaches
38-39C, accompanied with chill, headache
in forehead and temple area, pain in eyes,
stiffness of a nose, dry cough, pain on
trachea projection. Spleen is not enlarged.
d)
Acute beginning of disease
with chill, fast increase of temperature up to
39-40C, weakness, sweating, myalgias,
excitation, headache, dry cough, hyperemia
of face, injection of sclera, sometimes
palatal enanthema, relative bradycardia,
spleen is enlarged quite often.
21. What symptoms are typical for acute
period of louse-borne typhus?
a)
The body temperature up to
39-40C, persistent headache, prostration,
hallucinations, photophobia, serous
meningitis, anxiety, euphoria can be
present.. Face is hyperemic, vessels of
sclera are injected, and spleen is enlarged
since the 3-4 day of disease; rash cone out
on the upper part of trunk on the fifth day of
fever, involving whole body except of the
face, palms, and soles. Initially this rash is
macular, maculopapular, petechial.
b)
Gradual onset with slow
(within 4-5 days) increase of temperature,
general adynamia, pale skin, tongue is
thickened, with prints of teeth, relative
bradycardia, spleen is enlarged since the 67 day of disease;
c)
Acute beginning of disease,
within several hours temperature reaches
38-39C, accompanied with chill, headache
in forehead and temple area, pain in eyes,
stiffness of a nose, dry cough, pain on
trachea projection. Spleen is not enlarged.
d)
Acute beginning of disease
with chill, fast increase of temperature up to
39-40C, weakness, sweating, myalgias,
excitation, headache, dry cough, hyperemia
of face, injection of sclera, sometimes
palatal enanthema, relative bradycardia,
spleen is enlarged quite often.
22. What symptoms are typical for initial
period of influenza?
a)
Acute beginning with
increasing of body temperature within 1-2
days up to 39-40C, persistent headache,
sleeplessness, anxiety, euphoria. Face is
hyperemic, vessels of sclera are injected,
spleen is enlarged since 3-4 day of disease;

75

b)
Gradual onset with slow
(within 4-5 days) increase of temperature,
general adynamia, sleeplessness, pale skin,
tongue is thickened, with prints of teeth,
relative bradycardia, spleen is enlarged
since the 6-7 day of disease;
c)
Acute beginning of disease,
within several hours temperature reaches
38-39C, accompanied with chill, headache
in forehead and temple area, pain in eyes,
stiffness of a nose, dry cough, pain on
trachea projection. Spleen is not enlarged.
d)
Acute beginning of disease
with chill, fast increase of temperature up to
39-40C, weakness, sweating, myalgias,
excitation, headache, dry cough, hyperemia
of face, injection of scleras vessels,
sometimes palatal enanthema, relative
bradycardia, spleen is enlarged quite often.
23. What symptoms are typical for
gastroenterocolitic form of shigellosis:
a)
Acute onset, increase of
temperature up to 39C, repeated vomiting,
diffuse abdominal pain, frequent plentiful
watery stool without mucus and blood, spasm
of calves muscles, oliguria, hypotonia;
b)
Acute onset, increase of
temperature up to 38,5-39C, vomiting 3-4
times, spastic abdominal pain in left
hypogastria, plentiful liquid stool with
admixtures of mucus and blood
c)
Acute onset, normal body
temperature, vomiting, frequent plentiful
watery stool without admixtures, spasm of
calves muscles, expressed dryness of
mucous, decrease of skin elasticity, oliguria;
d)
Acute onset, increase of
temperature up to 39C, spastic pain in the
left half of abdomen, frequent liquid poor
stool with mucus and blood, tenesmus.
24. What symptoms are typical for colitic
form of shigellosis?
a)
Acute onset, increase of temperature
up to 39C, repeated vomiting, diffuse
abdominal pain, plentiful watery stool without
mucus and blood, spasm of calves muscles,
oliguria, hypotension;
b)
Acute onset, increase of temperature
up to 38,5-39C, vomiting 3-4 times, spastic
abdominal pain in left hypogastria, frequent
plentiful liquid stool with admixtures of mucus
and blood;
c)
Acute onset, normal body
temperature, vomiting, frequent plentiful
watery stool without admixtures, spasm of
calves muscles, expressed dryness of
mucous, decrease of skin elasticity, oliguria;

d)
Acute onset, increase of temperature
up to 39C, spastic pain in the left half of
abdomen, frequent liquid poor stool with
mucus and blood, tenesmus.
25. What clinical signs are typical for
cholera?
a)
Acute onset, increase of temperature
up to 39C, repeated vomiting, diffuse
abdominal pain, plentiful watery stool without
mucus and blood, spasm of calves muscles,
oliguria, hypotension;
b)
Acute onset, increase of temperature
up to 38,5-39C, vomiting 3-4 times, spastic
abdominal pain in left hypogastria, frequent
plentiful liquid stool with admixtures of mucus
and blood;
c)
Acute onset, normal body
temperature, vomiting, frequent plentiful
watery stool without admixtures, spasm of
calves muscles, expressed dryness of
mucous, decrease of skin elasticity, oliguria;
d)
Acute onset, increase of temperature
up to 39C, spastic pain in the left half of
abdomen, frequent liquid poor stool with
mucus and blood, tenesmus.
26. What hemodynamic parameters are
typical for hemorrhagic fever with renal
syndrome?
a Tachycardia, hypotension, tendency to
collapse;
b Relative bradycardia, dycrotic pulse,
hypotension;
c Significant bradycardia, hypotension;
d Tachycardia up to 120-160 /min., cardiac
arrhythmia, hypotension.
27. What hemodynamic parameters are
typical for typhoid fever?
a Tachycardia, hypotension, tendency to
collapse;
b Relative bradycardia, dycrotic pulse,
hypotension;
c Significant bradycardia, hypotension;
d Tachycardia up to 120-160 /min., cardiac
arrhythmia, hypotension.
28. What hemodynamic parameters are
typical for louse-borne typhus?
a
Tachycardia, hypotension, tendency to
collapse;
b Relative bradycardia, dycrotic pulse,
hypotension;
c Significant bradycardia, hypotension;
d Tachycardia up to 120-160 /min.,
cardiac arrhythmia, hypotension.
29. What are the main symptoms of hepatic
coma?
a Patient is unconsciousness.
Temperature 39,8C. Skin is pale;
slight jaundice. Tachycardia up to
140 /min. Sizes of liver by Kurlov

76

12119 cm., spleen is palpated on 4


cm. below edge of costal arch;
b Patient is unconsciousness.
Temperature 37,2C. Bright jaundice,
single petechial rash. Tachycardia up to
98 /min. Sizes of liver by Kurlov 887
cm.. Spleen is not enlarged;
c Patient is unconsciousness.
Temperature 36,1C. Skin is pale, dry.
Tachycardia up to 110 /min. Sizes of
liver by Kurlov: 1098 cm. Spleen is
not enlarged;
d Patient is unconsciousness.
Temperature 38,5C. Skin is of lightpink color and plentiful hemorrhagic
rash is presented. Tachycardia up to
130 /min. Liver and spleen are not
enlarged.
30. What are the main symptoms of
hyperglycemic coma?
a
Patient is unconsciousness.
Temperature is 39,8C. Skin is pale;
slight jaundice. Tachycardia up to
148 /min. Sizes of liver by Kurlov
12119 cm., spleen is palpated on
4 cm. below edge of costal arch;
b)
Patient is unconsciousness.
Temperature is 37,2C. Bright jaundice, and
single petechial rash. Tachycardia up to
98 /min. Sizes of liver by Kurlov 887 cm..
Spleen is not enlarged;
c)
Patient is unconsciousness.
Temperature is 36,1C. Skin is pale, dry.
Tachycardia up to 110 /min. Sizes of liver by
Kurlov: 1098 cm. Spleen is not enlarged;
d)
Patient is unconsciousness.
Temperature is 38,5C. Skin is of light pink
colorand plentiful hemorrhagic rash is
present. Tachycardia up to 130 /min. Liver
and spleen are not enlarged.
31. What are the main symptoms of malarial
coma?
a)
Patient is unconsciousness.
Temperature is 39,8C. Skin is pale, slight
jaundice. Tachycardia up to 148 /min. Sizes
of liver by Kurlov 12119 cm., spleen is
palpated on 4 cm. below edge of costal
arch;
b)
Patient is unconsciousness.
Temperature is 37,2C. Bright jaundice and
single petechial rash. Tachycardia up to
98 /min. Sizes of liver by Kurlov 887 cm..
Spleen is not enlarged;
c)
Patient is unconsciousness.
Temperature is 36,1C. Skin is pale, dry.
Tachycardia up to 110 /min. Sizes of liver by
Kurlov: 1098 cm. Spleen is not enlarged;

d)
Patient is unconsciousness.
Temperature is 38,5C. Skin is of light pink
color and plentiful hemorrhagic rash is
presented. Tachycardia up to 130 /min. Liver
and spleen are not enlarged.
32. What are the typical symptoms of initial
period of pseudo-tuberculosis?
a Acute onset, in 2-6 hours body
temperature reaches 38-39C,
expressed headache in forehead and
eyes, sore throat, dry cough. Skin of
face is hyperemic, scleritis. Lymph
nodes and spleen are not enlarged.
Urine passage is normal;
b Acute onset with chill, in 12-24 hours
temperature reaches 40C, strong
headache, expressed muscular pain.
Skin of face and neck is hyperemic and
icteric; scleritis, petechial rash are
present, liver and spleen are enlarged.
Oliguria.
c Acute onset, on the 2-3 day of disease
temperature reaches 40C, headache,
loss of appetite, pain in a throat, liquid
stool 3-5 times per day. Skin of face,
neck, palms and feet are hyperemic,
pale nasal triangle. Liver and spleen
are enlarged. Urine passage is normal;
d Acute onset, on the 2-3 day of disease
body temperature reaches 39,5-40C,
strong headache, anxiety, quite often
excitation, euphoria; hyperemia of
face, vessels of sclera are injected,
spleen is enlarged. Urine passage is
normal.
33. What are the typical symptoms of initial
period of influenza?
a
Acute onset, in 2-6 hours body
temperature reaches 38-39C,
expressed headache in forehead and
eyes, sore throat, dry cough. Skin of
face is hyperemic, scleritis. Lymph
nodes and spleen are not enlarged.
Urine passage is normal;
b
Acute onset with chill, in 12-24 hours
temperature reaches 40C, strong
headache, expressed muscular pain.
Skin of face and neck is hyperemic
and icteric; scleritis, petechial rash
are present, liver and spleen are
enlarged. Oliguria.
c
Acute onset, on the 2-3 day of
disease temperature reaches 40C,
headache, loss of appetite, pain in a
throat, liquid stool 3-5 times per day.
Skin of face, neck, palms and feet are
hyperemic, pale nasal triangle. Liver

77

and spleen are enlarged. Urine


passage is normal;
d
Acute onset, on the 2-3 day of
disease body temperature reaches
39,5-40C, strong headache, anxiety,
quite often excitation, euphoria;
hyperemia of face, vessels of sclera
are injected, spleen is enlarged.
Urine passage is normal.
34. What are the typical symptoms of initial
period of leptospirosis?
a
Acute onset, in 2-6 hours body
temperature reaches 38-39C,
expressed headache in forehead
and eyes, sore throat, dry cough.
Skin is hyperemic, scleritis. Lymph
nodes and spleen are not enlarged.
Urine passage is normal;
b
Acute onset with chill, in 12-24
hours temperature reaches 40C,
strong headache, expressed
muscular pain (predominate in
calves muscles). Skin of face and
neck is hyperemic and icteric;
scleritis, petechial rash are present,
liver and spleen are enlarged.
Oliguria.
c
Acute onset, on the 2-3 day of
disease temperature reaches 40C,
headache, loss of appetite, pain in
a throat, liquid stool 3-5 times per
day. Skin of face, neck, palms and
feet are hyperemic, pale nasal
triangle. Liver and spleen are
enlarged. Urine passage is normal;
d
Acute onset, on the 2-3 day of
disease body temperature reaches
39,5-40C, strong headache,
anxiety, quite often excitation,
euphoria; hyperemia of face,
vessels of sclera are injected,
spleen is enlarged. Urine passage is
normal.
35. What symptoms are typical for
meningococcal nasopharyngitis?
a Granularity and hyperemia of a back wall of
throat;
b Hemorrhagic rash;
c Stiffness of neck muscles;
d Plentiful purulent nasal excretion.
36. What symptoms are typical for
meningococcal meningitis?
a Granularity and hyperemia of a back wall of
throat;
b Hemorrhagic rash;
c Stiffness of neck muscles;
d Plentiful purulent nasal excretion.
37.What symptoms are typical for
meningococcemia?

a)
Granularity and hyperemia of a back
wall of throat;
b)
Hemorrhagic rash;
c)
Stiffness of neck muscles;
d)
Plentiful purulent nasal excretion.
38. What symptoms of CNS affection are
typical for louse-borne typhus?
a)
Hyperesthesia, nausea and vomiting,
loss of consciousness, headache, positive
Kernings symptom, stiffness of neck muscles;
b)
Euphoria, excitation, anxiety,
acoustical and visual hallucinations, persistent
headache, symptoms of cranial nerves
affection;
c)
Adynamia, dormancy, dull headache;
d)
Dizziness, noise in ears, adynamia,
headache in forehead and temple areas.
39. What symptoms of CNS affection are
typical for typhoid fever?
a)
Hyperesthesia, nausea and vomiting,
loss of consciousness, headache, positive
Kernigs symptom, stiffness of neck muscles;
b)
Euphoria, excitation, anxiety,
acoustical and visual hallucinations, persistent
headache, symptoms of cranial nerves
affection;
c)
Adynamia, dormancy, dull headache;
d)
Dizziness, noise in ears, adynamia,
headache in forehead and temple areas.
40. What symptoms of CNS affection are
typical for purulent meningitis
a Hyperesthesia, nausea and
vomiting, quite often loss of
consciousness, headache,
positive Kernigs symptom,
stiffness of neck muscles;
b Euphoria, excitation, anxiety,
acoustical and visual
hallucinations, persistent
headache, symptoms of cranial
nerves affection;
c Adynamia, dormancy, dull
headache;
d Dizziness, noise in ears,
adynamia, headache in
forehead and temple areas.
41. What type of rash and its location are
typical for pseudo-tuberculosis?
a)
Roseolar and papular rash arises on
the 2-5 day of disease on top of trunk, then
spreads on face, extremities and around
joints;
b)
Plentiful, roseolar and petechial rash
arises on the 4-5 day of disease, mainly on
thorax, extremities, usually is absent on face,
palms and feet;
c)
Little-pointed erythematic rash arises
on the 2-6 day of disease on intact skin, is
located mainly on the lateral surfaces of a

78

trunk, axillary areas, abdomen, and


extremities, is concentrated on palms and feet
as "gloves" and "socks"; face is usually clear;
d)
Spotty-papular rash arises on the 3-4
day of disease on the face, then on trunk,
forearms thighs, and on shins, leaves
pigmentation and peeling after recovery;
42. What type of rush and its location are
typical for louse-borne typhus?
a Roseolar and papular rash arises on
the 2-5 day of disease on top of
trunk, then spreads on face,
extremities and around joints;
c Plentiful, roseolar and petechial rash
arises on the 4-5 day of disease,
mainly on thorax, extremities, usually
is absent on face, palms and feet;
d Little-pointed erythematic rash arises
on the 2-6 day of disease on intact
skin, is located mainly on the lateral
surfaces of a trunk, axillary areas,
abdomen, and extremities, is
concentrated on palms and feet as
"gloves" and "socks"; face is usually
clear;
e Spotty-papular rash arises on the 3-4
day of disease on the face, then on
trunk, forearms and thighs, then on
shins, leaves pigmentation and
peeling after recovery.
43. What type of rush and its location are
typical for tick-borne North Asian
rickettsiasis?
a)
Roseolar and papular rash
arises on the 2-5 day of disease on top of
trunk, then spreads on face, extremities and
around joints;
b)
Plentiful, roseolar and
petechial rash arises on the 4-5 day of
disease, mainly on thorax, extremities, usually
is absent on face, palms and feet;
c)
Little-pointed erythematic
rash arises on the 2-6 day of disease on intact
skin, is located mainly on the lateral surfaces
of a trunk, axillary areas, abdomen, and
extremities, is concentrated on palms and feet
as "gloves" and "socks"; face is usually clear;
d)
Spotty-papular rash arises on
the 3-4 day of disease on the face, then on
trunk, forearms thighs, and on shins, leaves
pigmentation and peeling after recovery;
44. What mechanism of rash formation is
typical for meningococcemia?
a)
Damage of capillarys
endothelium by microorganisms and toxins,
circulating in blood;
b)
Sedimentation of immune
complexes in capillaries of skin;

c)
Drift of pathogen in lymph
vessels of skin with subsequent development
of productive - inflammatory changes;
d)
Generalized destructive and
prolypherative vasculitis;
45. What mechanism of rash formation is
typical for typhoid fever?
a Damage of capillarys
endothelium by
microorganisms and toxins,
circulating in blood;
b Sedimentation of immune
complexes in capillaries of
skin;
c Generalized destructive and
prolypherative vasculitis;
d Drift of pathogen in lymph
vessels of skin with
subsequent development of
productive - inflammatory
changes;
46. What mechanism of rash formation is
typical for louse-borne typhus?
a)
Damage of capillarys
endothelium by microorganisms and toxins,
circulating in blood;
b)
Sedimentation of immune
complexes in capillaries of skin;
c)
Drift of pathogen in lymph
vessels of skin with subsequent development
of productive - inflammatory changes;
d)
Generalized destructive and
prolypherative vasculitis;
47. What abdominal symptoms are typical
for rotaviral infection?
a)
Spleen and liver are enlarged
since the 3-4 day of disease; tendency to
constipation, paradoxical ishuria, oliguria;
b)
Liver and spleen are enlarged
since the 6-7 day of disease, stool is liquid
3-4 times per day, meteorism;
c)
Liver and spleen are not
enlarged, abdomen is soft, slightly painful
around navel, stool is liquid from 2 up to 10
times per day;
d)
Liver and spleen are enlarged
since the 3-4 day of disease, sensitive on
palpation, meteorism, mesenteric lymph
nodes are palpated, stool 1-2 times a day.
48. What abdominal symptoms are typical
for louse-borne typhus?
a)
Spleen and liver are enlarged
since the 3-4 day of disease, soft, tendency
to constipation, paradoxical ishuria, oliguria;
b)
Liver and spleen are enlarged
since the 6-7 day, stool liquid 3-4 times per
day, meteorism;
c)
Liver and spleen are not
enlarged, abdomen is soft, slightly painful

79

around navel, stool is liquid from 2 up to 10


times per day;
d)
Liver and spleen are enlarged
since the 3-4 day of disease, sensitive on
palpation, meteorism, mesenteric lymph
nodes are palpated, stool 1-2 times a day.
49.What abdominal symptoms are typical
for typhoid fever?
a)
Spleen and liver are enlarged
since the 3-4 day of disease, soft;; tendency
to constipation, paradoxical ishuria, oliguria;
b)
Liver and spleen are enlarged
since the 6-7 day, tool is liquid 3-4 times per
day, meteorism;
c)
Liver and spleen are not
enlarged, abdomen is soft, slightly painful
around navel, stool is liquid from 2 up to 10
times per day;
d)
Liver and spleen are enlarged
since the 3-4 day of disease, sensitive on
palpation; meteorism, mesenteric lymph
nodes are palpated, stool 1-2 times a day.
50. What symptoms are typical for initial
period of leptospirosis?
a)
Acute onset with increase of
body temperature up to 38C, loss of appetite,
nausea, weakness and general unwell,
sometimes dry cough. Since the 2-4 day of
disease liver is enlarged, temperature of body
normalizes;
b)
Acute onset with increase of
body temperature up to 39,5C, muscular and
joint pains; a headache in forehead area, pain
in eyes, dry cough, tracheal pain. Liver is not
enlarged.
c)
Acute onset with increase of
body temperature up to 39C, headache,
myalgias (mainly in calves muscles),
weakness, pain in eyes. Injection of sclera,
face is hyperemic, petechial rash in axillary
areas. The liver is enlarged, dark urine,
oliguria.
d)
Acute onset with increase of
body temperature up to 38C, weakness,
running nose, dry cough, pain in eyes.
Conjunctiva are hyperemic. Neck, axillary
lymph nodes and liver are enlarged. Urine is
normal.
51. What symptoms are typical for initial
period of viral hepatitis A?
a)
Acute onset with increase of
body temperature up to 38C, loss of appetite,
nausea, weakness and general unwell,
sometimes dry cough. Since the 2-4 day liver
is enlarged, temperature of body is
normalized;
b)
Acute onset with increase of
body temperature up to 39,5C, muscular and
joint pains; a headache in forehead area, pain

in eyes, dry cough, tracheal pain. Liver is not


enlarged.
c)
Acute onset with increase of
body temperature up to 39C, headache,
myalgias (mainly in calves muscles),
weakness, pain in eyes. Vessels of sclera are
injected, face is hyperemic, petechial rash in
axillary areas. The liver is enlarged, dark
urine, oliguria.
d)
Acute onset of disease with
increase of body temperature up to 38C,
weakness, running nose, dry cough and pain
in eyes. Conjunctiva is hyperemic. Neck,
axillary lymph nodes and liver are enlarged.
Urine is normal.
52. What symptoms are typical for initial
period of adenoviral infection?
a)
Acute onset with increase of
body temperature up to 38C, loss of appetite,
nausea, weakness and general unwell,
sometimes dry cough. Since the 2-4 day of
disease liver is enlarged, temperature of body
normalizes;
b)
Acute onset with increase of
body temperature up to 39,5C, muscular and
joint pains; a headache in forehead area, pain
in eyes, dry cough, tracheal pain. Liver is not
enlarged.
c)
Acute onset with increase of
body temperature up to 39C, headache,
myalgias (mainly in calves muscles),
weakness, pain in eyes. Injection of sclera,
face is hyperemic, petechial rash in axillary
areas. The liver is enlarged, dark urine,
oliguria.
d)
Acute onset of disease with
increase of body temperature up to 38C,
weakness, running nose, dry cough, and pain
in eyes. Conjunctiva is hyperemic. Neck,
axillary lymph nodes and liver are enlarged.
Urine is normal. Diarrhea can be present.
53. What drug may be used for specific
therapy of shigellosis?
a)
Erythromycin;
b)
Cyprofloxacin;
c)
Penicillin;
d)
Metronidazol (trychopol);
e)
Chloroquin.
54. What drug may be used for specific
therapy of intestinal amebiasis?
a)
Erythromycin;
b)
Ciprofloxacin;
c)
Penicillin;
d)
Metronidazole (Trichopol);
e)
Chloroquine.
55. What drug may be used for specific
therapy of malaria?
a)
Erythromycin;
b)
Ciprofloxacin;

80

c)
Penicillin;
d)
Metronidazole (Trichopol);
e)
Chloroquine.
56. What symptoms are typical for initial
period of pseudo-tuberculosis?
a)
Acute onset with fast increase
of temperature of a body up to 39-40, strong
headache, euphoria, sleeplessness; injection
of scleras vessels, insignificant hyperemia of
the soft palate and back wall of throat, lymph
nodes are not enlarged, spleen is enlarged
since the 3-4 day of disease;
b)
Acute onset with increase of
temperature up to 38,5-40C, edema and pain
in ankle and knee joints, pain in a throat,
increase of regional lymph nodes, bright
hyperemia of the soft palate with precise
border, white coated tongue. Spleen and liver
are enlarged.
c)
Gradual onset, from the 3d
day increase of temperature up to 38C,
running nose, pain in a throat. Tonsils are
enlarged, hyperemic, without fur. Neck,
axillary lymph nodes are enlarged;
conjunctivitis; joints are not changed;
sometimes spleen is enlarged; dry cough is
quite often;
d)
Acute onset, for several hours
temperature reaches 40C, headache in
frontal area, pain in eyes, tracheal pain is
present. Hyperemia of soft palate, lymph
nodes and spleen are not enlarged, joints are
normal.
57. What symptoms are typical for initial
period of louse-borne typhus?
a)
Acute onset with fast increase
of temperature of a body up to 39-40 ,
strong headache, euphoria, sleeplessness;
injection of scleras vessels, insignificant
hyperemia of the soft palate and back wall of
throat, lymph nodes are not enlarged, spleen
is enlarged since the 3-4 day of disease;
b)
Acute onset with increase of
temperature up to 38,5-40C, edema and pain
in ankle and knee joints, pain in a throat,
increase of regional lymph nodes, bright
hyperemia of the soft palate with precise
border, white coated tongue. Spleen and liver
are enlarged.
c)
Gradual onset, from the 3d
day increase of temperature up to 38C,
running nose, pain in a throat. Tonsils are
enlarged, hyperemic, without fur. Neck,
axillary lymph nodes are increased;
conjunctivitis; joints are not changed;
sometimes spleen is enlarged;; dry cough is
quite often;
d)
Acute onset, in several hours
temperature reaches 40C, headache in

frontal area, pain in eyes, tracheal pain.


Hyperemia of soft palate, lymph nodes and
spleen are not enlarged, joints are normal.
58. What symptoms are typical for initial
period of adenoviral infection?
a)
Acute onset with fast increase
of temperature of a body up to 39-40, strong
headache, euphoria, sleeplessness; injection
of scleras vessels, insignificant hyperemia of
the soft palate and back wall of throat; lymph
nodes are not enlarged; spleen is enlarged
since the 3-4 day of disease;
b Acute onset with increase of
temperature up to 38,5-40C, edema
and pain in ankle and knee joints,
pain in a throat, increase of regional
lymph nodes, bright hyperemia of
the soft palate with precise border,
white coated tongue. Spleen and
liver are enlarged.
c Gradual onset, from the 3d day
increase of temperature up to 38C,
running nose, pain in a throat. Tonsils
are enlarged, hyperemic, without fur.
Neck, axillary lymph nodes are
increased; conjunctivitis; joints are
not changed; sometimes spleen is
enlarged; dry cough is quite often;
d Acute onset, in several hours
temperature reaches 40C, headache
in frontal area, pain in eyes, tracheal
pain. Hyperemia of soft palate, lymph
nodes and spleen are not enlarged;
joints are normal.

SKIN CHANGES IN SOME INFECTIOUS


DISEASES
59. What skin phenomena are typical for
skin form of plague?
e There is on the hand painless ulcer
23 cm with a blackened necrotic
eschar circled with elevated
hyperemic rim, where small satellite
vesicles, filled by serous-hemorrhagic
contents are seen. Expanding zone of
brawny edema, which jelly-like
fluctuates during shaking, surrounds
this affection..
f There is ulcer 11 cm with hard
yellowish bottom at the skin of leg
without purulent content. Ulcer is
circled by elevated intensively
hyperemic and sharply outlined rim
with cyanotic shade. Ulcer is sharply
painful.

81

Moderate edema of skin. There is


crater-like ulcer under gray crust on
the highly infiltrated and hyperemic
base in the center of edema; pus is
excreted during pressing.
Lymphangitis is present. Touching of
ulcer is painful.
a Skin is markedly hyperemic, shining,
smoothed; a zone of affection has
precise festooned edges. On the
center of affection hyperemia is less
expressed, than on edges, where
bullas with serous content are seen..
60. What skin phenomena are typical for
staphylococcal carbuncle?
b There is painless ulcer 23 cm with a
blackened necrotic eschar circled
with elevated hyperemic rim, where
small satellite vesicles, filled by
serous-hemorrhagic contents are
seen. Expanding zone of brawny
edema, which jelly-like fluctuates
during shaking, surrounds this
affection..
c There is ulcer 11 cm with hard
yellowish bottom at the skin of leg
without purulent content. Ulcer is
circled by elevated intensively
hyperemic and sharply outlined rim
with cyanotic shade. Ulcer is sharply
painful
d Moderate edema of skin. There is
crater-like ulcer under gray crust on
the highly infiltrated and hyperemic
base in the center of edema; pus is
excreted during pressing.
Lymphangitis is present. Touching of
ulcer is painful.
e Skin is markedly hyperemic, shining,
smoothed; a zone of affection has
precise festooned edges. On the
center of affection hyperemia is less
expressed, than on edges, where
bullas with serous content are seen.
61. What skin phenomena are typical for
skin form of anthrax?
a There is painless ulcer 23 cm with a
blackened necrotic eschar circled
with elevated hyperemic rim, where
small satellite vesicles, filled by
serous-hemorrhagic contents are
seen. Expanding zone of brawny
edema, which jelly-like fluctuates
during shaking, surrounds this
affection.
b There is ulcer 11 cm with hard
yellowish bottom at the skin of leg
without purulent content. Ulcer is
circled by elevated intensively

hyperemic and sharply outlined rim


with cyanotic shade. Ulcer is sharply
painful
c Moderate edema of skin. There is
crater-like ulcer under gray crust on
the highly infiltrated and hyperemic
base in the center of edema; pus is
excreted during pressing.
Lymphangitis is present. Touching of
ulcer is painful.
f Skin is markedly hyperemic, shining,
smoothed; a zone of affection has
precise festooned edges. On the
center of affection hyperemia is less
expressed, than on edges, where
bullas with serous content are seen.
62. What character and localization of rush
are typical for pseudo-tuberculosis?
a Roseolar and petechial rash occurs
on the 4-5 days of disease on lateral
surfaces of trunk, internal surfaces of
shoulders and forearms (intact face,
palms, plants);
b Rash is plentiful, roseolar and
papular, sometimes erythematous
(face, neck); it arises on the 2-3 days
of disease, condenses on skin of
joints, palms, feet and resembles
there "gloves" and "stocks";
c Single elements of roseolar rash arise
on the 8-10 day of disease, are
located on forward surface of
abdomen and thorax, some elements
are roseolopapular.
63. What character and localization of rush
are typical for typhoid fever?
a) Roseolar and petechial rash occurs
on the 4-5 days of disease on lateral
surfaces of trunk, internal surfaces of
shoulders and forearms (intact face, palms,
plants);
b) Rash is plentiful, roseolar and
papular, sometimes erythematous (face,
neck); it arises on the 2-3 days of disease,
condenses on skin of joints, palms, feet and
resembles there "gloves" and "stocks";
c) Single elements of roseolar rash arise
on the 8-10 day of disease, are located on
forward surface of abdomen and thorax,
some elements are roseolopapular.
64. What character and localization of rush
are typical for louse borne-typhus?
a) Roseolar and petechial rash occurs
on the 4-5 days of disease on lateral
surfaces of trunk, internal surfaces of
shoulders and forearms (intact face, palms,
plants);
b) Rash is plentiful, roseolar and
papular, sometimes erythematous (face,

82

neck); it arises on the 2-3 days of disease,


condenses on skin of joints, palms, feet and
resembles there "gloves" and "stocks";
d Single elements of roseolar rash arise
on the 8-10 day of disease, are
located on forward surface of
abdomen and thorax, some elements
are roseolopapular.

70. What is physicians tactic in case of


negative result of IEA in patient with
suspicion on HIV-infection?
a To perform common blood test;
b To repeat IEA with test-system of other
firm;
c To exclude diagnosis of HIV-infection;
d To prescribe antiretroviral therapy;
e To repeat IEA assay through one ear.
71. Which methods from mentioned below
are informative for cytomegalovirus
infection?
a Blood thin smear and tick drop microscopy;
b Common blood test;
c Detection of antibodies IgM with
immunoenzyme assay (IEA);
d Detection of antibodies IgG with IEA;
e Detection of specific-changed cells in urine.
72. What laboratory methods should be
applied for proving of herpesviral etiology of
meningoencephalitis?
a)
Investigation of dynamic titer
of antibodies IgM and IgG;
b)
PCR in CSF smears;
c)
Common blood test;
d)
Immunoenzyme assay with
detection of antibodies IgM, IgG;
e)
All mentioned.
73 What laboratory method is more
informative for proving of chronic
brucellosis?
a Culture of blood;
b Bacterioscopy of blood
smears;
c Allergological test;
d Detection of antibodies with
indirect hemagglutination
test.
e Biological method.
74. What smears from mentioned below
may be used for bacteriological
investigation of patients with
pseudotuberculosis?
a)
Blood;
b)
Nasal secretion;
c)
Sputum;
d)
Urine;
e)
All mentioned.
75 What changing in CSF is most
characteristic for meningococcal meningitis?
a)
CSF pressure elevation;
b)
High neutrophilic pleocytosis;
c)
Proteincell dissociation;
d)
Normal glucose amount;
e)
Greenish color of CSF.
76. The main advantage of passive

LABORATORY METHODS OF
DIAGNOSTICS
65. What methods are used for diagnosing
of HIV/AIDS? (2)
a Blood thin smear and thick drop
microscopy;
b Common blood test;
c Immune enzyme assay;
d Immune blotting;
e Culturing on embryo eggs
66. What tests are used for cytomegalovirus
infection diagnostics?
a)
Compliment fixation test;
b)
Common blood test;
c)
Immune enzyme detection of
specific IgM and IgG in serum of blood;
d)
Cytological assay of urine
sediment;
e)
Immunological assay
(estimation of absolute and relative amount of
lymphocytes subpopulation, titer of IgG, IgM,
IgA antibodies in serum of blood).
67. What methods are used for detection of
replication activity of HIV?
a Indirect agglutination test;
b PCR;
c Complement fixation test;
d Immunofluorescent assay with specific
monoclonal antibodies;
e Western blotting.
68. What is the defect of immunoenzyme
assay ( ELYSA) in HIV/AIDS diagnostics?
a Low sensitivity;
b Low specificity;
c Expensive cost;
d High level of false positive results;
e High level of false-negative results.
69. How to treat positive result IEA in case
of absence of clinical manifestation of
HIV/AIDS?
a)
Patient has stable protective
immunity to HIV;
b)
Patient may be in latent
period, acute stage was asymptomatic or
wasnt diagnosed;
c)
It is incubation period;
d)
It is an acute asymptomatic
stage of infection.

immunization over active immunization is


that passive immunization:

83

Eliminates the risk of


hypersensitivity reactions;
b Is effective against multiple
organisms;
c Is more cost-effective as a public
health measure;
d Provides immediate protection;
e Magnifies the specific immune
response to the offending
organism.
77. Which one of the following methods is
not informative for diagnosing of chronic
toxoplasmiasis?
a)
X-ray examination of brain
and muscles;
b)
Parasitological examination of
feces;
c)
Parasitological examination of
blood;
d)
ECG;
e)
Examination of eye bottom.
78. Which one of the following markers
indicates acute viral hepatitis A?
a HBsAg in blood serum;
b Anti-HBcor IgG in blood serum;
c Anti-HAV IgG in blood serum;
d Anti-HAV IgM in blood serum;
e Anti-HCV IgM in blood serum.
79. Which one of the following methods is
more informative for diagnosing of food
poisonings?
a Bacteriological investigation of
vomiting masses;
b Bacteriological investigation of feces;
c Bacteriological investigation of blood;
d Detection of toxin;
e Serological investigation.
80. What changes in blood count are typical
for shigellosis?
a Leukopenia;
b Moderate neutrophilic leukocytosis;
c Relative lymphomonocytosis;
d Drum-sticks shift to left.
81. Which one of the following methods is
more informative for early specific
diagnosing of ARVI?
a Rhinocytoscopy;
b Culturing on egg embryos;
c Serological;
d Immunofluorescent assay;
e Clinical and epidemiological investigation.
82. Which one of the following methods is
more informative for diagnosing of latent
forms of toxoplasmosis?
a Compliment fixation test;
b X-ray examination of brain;
c Parasitoscopy of feces;
d Sabin-Feldman test;
e Direct immunofluorescent test.

83. Which one of the following markers is


more informative for diagnosing of acute
viral hepatitis B?
a Anti-HAV IgM;
b HBs Ag;
c Hbe Ag;
d Anti-HBe;
e Anti-HBc IgM.
84. What methods of specific laboratory
diagnostics of shigellosis is usually apply?
a Bacteriological investigation of faeces;
b Bacteriological investigation of urine;
c Indirect hemagglutination test (IHAT);
d Immunofluorescent test;
e Immunoenzyme assay.
85. Which one of the following methods is
more informative for diagnosing of cholera?
a)
Bacterioscopy of feces ;
b)
Culture of blood;
c)
Serological investigation;
d)
Culture of feces and vomit
masses;
e)
Skin allergic test.
86. Which one of the following methods is
more informative for diagnosing of
botulism?
a Identification of botulinum toxin with
inoculation of mice with patients blood;
b Culture of blood;
c Serological method;
d Culture of urine;
e Culture of suspicious food.
87. What serological tests are usually apply
for diagnostic of brucellosis?
a Complement fixation test;
b Right and Heddelson agglutination test;
c Indirect hemagglutination test;
d Immunofluorescent test;
e Latex agglutination test.
88. Which one of the following groups of
methods is more informative for diagnosing of
pseudotuberculosis?
a) Epidemiological;
74 Clinical;
75 Serological.;
e Bacteriological;
f All mentioned.
89. Bacteriological investigation of which
one of the following smears is more
effective for early diagnosing of typhoid and
paratyphoid fevers?
a Feces;
b Blood;
c CSF;
d Urine;
e All mentioned.
90. In what case result of immunoenzyme
assay for diagnostic of HIV-infection may be
false negative?

84

a
In patients with malaria;
b In patients, treated with glucocorticoids;
c In pregnant woman;
d In homosexual persons;
e In drug abused persons.
91. What peculiarities of blood cell count are
typical for climax period of typhoid fever?
a Leukopenia, neutropenia,
eosinopenia, relative
lymphomonocytosis;
b Normal count or insignificant
Leukocytosis, tendency to
neutrophilic increase, drumstick shift to the left,
monocytosis, plasmatic Turk
cells;
c Leukocytosis, eosinophylia,
lymphocytosis, ESR elevation;
d Leukocytosis, neutropenia,
lymphomonocytosis, atypical
mononuclears.
e Leukocytosis, neutrophilia,
drum-stick shift to the left,
high level of ESR
92. What changes in CBC are typical for
influenza?
a Leukocytosis, eosinophylia,
neutropenia, lymphocytosis,
monocytosis, expressed elevated
ESR;
b Leukopenia, neutropenia,
lymphocytosis, normal ESR;
c Leukocytosis, neutropenia,
monocytosis, lymphocytosis, normal
ESR .
d Leukocytosis, neutrophilia, drumstick shift to the left, moderately
elevated ESR.
e Leukopenia, neutropenia, drum-stick
shift to the left, eosinopenia, relative
lymphomonocytosis, moderately
elevated ESR
93. What changes in CBC are typical for
infectious mononucleosis?
a a) Leukocytosis, neutropenia,
lymphocytosis, monocytosis;
b b) Leukopenia, neutropenia,
lymphocytosis;
c c) Leukocytosis, neutropenia,
monocytopenia, lymphocytosis.
d d) Leukocytosis, neutrophilia, drumstick shift to the left.
94. What changes in CBC are typical for
acute tonsillitis?
a Leukocytosis, neutropenia,
lymphocytosis, monocytosis;
b Leukopenia, neutropenia,
lymphocytosis;

Leukocytosis, neutropenia,
monocytopenia, lymphocytosis.
d Leukocytosis, neutrophilia, drumstick shift to the left.
95. What methods of specific diagnostics are
informative for rotoviral gastroenteritis?
b Bacteriological investigation
of feces;
c Microscopy of feces;
d Latex agglutination test;
e Rectosigmoidoscopy;
f Revealing of viral antigen in
feces with IFA.
96. Which one of the following methods is
more informative for specific diagnosing of
gastrointestinal form of salmonellosis?
a)
Culture of feces;
b)
Bacterioscopy of feces;
c)
Culture of urine;
d)
Rectosigmoidoscopy;
e)
Coprological investigation of feces
97. Which one of the following methods is
more informative for specific diagnosing of
lambliasis?
a)
Culture of feces;
b)
Bacterioscopy of feces;
c)
Culture of urine;
d)
Rectosigmoidoscopy;
e)
Coprological investigation of feces
98. Which one of the following appearances
of CSF is more typical for meningococcal
meningitis?
a Muddy, of white-grayish color;
b Transparent, colorless
c Hemorrhagic;
d Muddy, of greenish color;
e Transparent with fibrinous membrane
99. Which one of the following appearances
of CSF is more typical for viral meningitis?
a Muddy, of white-grayish color;
b Transparent, colorless
c Hemorrhagic;
d Muddy, of greenish color;
e Transparent with fibrinous membrane
100. Which one of the following
appearances of CSF is more typical for
antrax meningitis?
f Muddy, of white-grayish color;
g Transparent, colorless
h Hemorrhagic;
i
Muddy, of greenish color;
j
Transparent with fibrinous membrane
101. What blood cells count is typical for
meningococcemia?
a Neutrophilic hyperleukocytosis with
left shift, high ESR;
b Moderate neutrophilic leukocytosis,
ESR is moderately increased;

85

a
b
c
d

Leukopenia, lymphomonocytosis,
sometimes ESR increase;
d Leukopenia, relative lymphocytosis,
increase of plasmatic cells number,
increase of ESR.
102. What blood cell count is typical for tickborne encephalitis?
Neutrophilic hyperleukocytosis with left shift,
high ESR;
Moderate neutrophilic leukocytosis, ESR is
moderately increased;
Leukopenia, lymphomonocytosis, sometimes
ESR increase;
Leukopenia, relative lymphocytosis, increase
of plasmatic cells number, increase of ESR.
103. Choose the methods of specific
diagnosing of louse-borne typhus (Brills
disease):
a CFT, IHAT with Rickettsia provazekii;
b CFT with Rickettsia burneti;
c CFT, IHAT with Rickettsia sybirica;
d IEA with Borrelia burgdorferi;
e CFT, IHAT with Rickettsia mooseri.
104. Choose the methods of specific
diagnosing of North-Asian rickettsiosis:
a CFT, IHAT with Rickettsia provazekii;
b CFT with Rickettsia burneti;
c CFT, IHAT with Rickettsia sybirica;
d IEA with Borrelia burgdorferi;
e) CFT, IHAT with Rickettsia mooseri
105. Choose the methods of specific
diagnosing of Q-fever:
a CFT, IHAT with Rickettsia provazekii;
b CFT with Rickettsia burneti;
c CFT, IHAT with Rickettsia sybirica;
d IEA with Borrelia burgdorferi;
e CFT, IHAT with Rickettsia mooseri
106. Which one of the following results of
CSF investigations is typical for serous
meningitis?
a)
Colorless, transparent, cytosis
0,003 109 /l., all cells are lymphocytes;
protein 0,33 /. Test for protein
sedimentation is negative. CSF pressure is
250 mm of water;
b)
Colorless, transparent, cytosis
0,2 109 /l, lymphocytes 80 %, protein
1,0 g/l. Test for protein sedimentation is
positive. CSF pressure is 300 mm of water;
c)
Colorless, transparent, cytosis
0,01 109 /l, lymphocytes 85 %, protein 0,3
g/l, sediment reaction negative, pressure is
300 mm of a water;
d)
Muddy, white-yellow color,
cytosis 15,0 109 /l, neutrophils 100 %,
protein 6,6 g/l, sediment reaction are
sharply positive, pressure of 350 mm of a
water.

107. Which one of the following results of


CSF investigations is typical for purulent
meningitis?
a)
Colorless, transparent, cytosis
0,003 109 /l., all cells are lymphocytes;
protein 0,33 /. Test for protein
sedimentation is negative. CSF pressure is
250 mm of water;
b)
Colorless, transparent, cytosis
0,2 109 /l, lymphocytes 80 %, protein
1,0 g/l. Test for protein sedimentation is
positive. CSF pressure is 300 mm of water;
c)
Colorless, transparent, cytosis
0,01 109 /l, lymphocytes 85 %, protein 0,3
g/l, sediment reaction negative, pressure is
300 mm of water;
d)
Muddy, white-yellow color,
cytosis 15,0 109 /l, neutrophils 100 %,
protein 6,6 g/l, sediment reaction are
sharply positive, pressure of 350 mm of
water.
108. Which one of the following results of
CSF investigations is typical for tuberculosis
meningitis?
a)
Colorless, transparent, cytosis
0,003 109 /l., all cells are lymphocytes;
protein 0,33 g/l. Test for protein
sedimentation is negative. CSF pressure is
250 mm of water;
b)
Colorless, transparent, cytosis
0,2 109 /l, lymphocytes 70 %, glucose is
significantly decrease, protein 1,0 g/l,
fibrinous membrane is formed.
c)
Colorless, transparent, cytosis
0,01 109 /l, lymphocytes 85 %, protein 0,3
g/l, sediment reaction negative, pressure is
300 mm of a water;
d)
Muddy, white-yellow color,
cytosis 15,0 109 /l, neutrophils 100 %,
protein 6,6 g/l, sediment reaction are
sharply positive, pressure of 350 mm of a
water;
109.Resistance of St.aureus to methicillin is
most often caused by:
a

Alteration of the major target for


the drug;
b Cell membrane impermeability;
c Decreased uptake of the
antibiotic;
d Inactivation of autolysins;
e Synthesis of a potent lactamaze.
110. The molecular basis for the effect of
cholera toxin on mucosal cells is (2):
a
b

86

Activation of adenylate cyclase;


Inactivation of G protein;

Increased activity of potassium


pumps;
d Increased generation of cyclic
adenosine monophosphate
(cAMP);
e Ribosylation of a guanosine
triphosphate (GTP)binding
protein.
111. What is the most common cause of

patients with chronic streptococcal tonsillitis


may be:
a
b
c

aseptic meningitis of viral etiology?


a
b
c
d
e
113. The

Enteroviruses;
Herpesviruses;
Arboviruses;
Retroviruses;
Orthomyxoviruses.
finding of large, multinucleated

Persistent release of streptococcal


antigens from silent infectious
foci;
The release of carditoxin;
The sharing of epitopes between
streptococcal and cardiac tissue
glycoproteins;
The tropism of group A
streptococci for the cardiac
valves.
Induction of a specific
autoimmune response to cardiac
tissue;

clumps of cells in the bronchial secretion of


patient

with

acute

bronchopneumonia

suggests that this infection is caused by:


a
b
c
d
e

Epstein-Barr virus;
Str.pneumonia;
Mycoplasta hominis;
Rhinovirus;
*
Respiratorysynciytial
virus.
114. Which one of the following infection is

It is another pare of shoes.

URGENT STATES IN INFECTIOUS


PATHOLOGY

most likely to be classified as opportunistic?

118. For what kind of shock increasing of


relative density of blood plasma is typical?
a Infectious-toxic;
b Hypovolemic;
c Anaphylactic;
d Cardiogenic;
e Traumatic.
119. What way of drugs administration is
most effective in case of infectious-toxic
shock ?
a oral administration
b subcutaneously;
c intramuscularly;
d intravenously;
e intraartreioly.
120. What one of the following solutions is
most effective for initial treatment of
infectious-toxic shock?
a) Trisol;
b) 4% natrium hydrocarbonate solution;
c) Polyglucine;
d) Innunoglobuline;
e) Regydrone.
121. Choose the clinical criteria of sopor (2):
a Contact with patient is absent;
b Contact with patient is possible only
on level of verbal-mimic reactions;
c Directed reactions on pain irritations
are saved;

115.

Brucellosis in the child of a


migrant worker;
b Escherichia coli bacteremia in a
leukemic patient receiving
chemotherapy;
c Group A streptococcal pharyngitis
in a school-aged child;
d Pulmonary anthrax in a wool
sorter;
e Pulmonary tuberculosis in a
physician.
Which
one
of
the
following

immunizations

should

be

administrated

immediately after birth?


a

Diphtheria-pertussis-tetanus

(DPT)

vaccine;
b

Haemophilus

influenza

type

vaccine;
c

Hepatitis B vaccine;

HIV-vaccine;

Oral polio vaccine

116.The most important pathogenic factor


leading to the development of carditis in a

87

Reactions of pupils on light are


absent;
e Patients answers on questions are
monosyllabic and not always
adequate.
122. Choose the most typical early clinical
signs of brain edema:
a Hemorrhagical rash on the mucous
membrane of soft palate;
b Cardiac arrhythmia;
c Psycho-motor excitation;
d Change of consciousness
e Tonic-and clonic cramps.
123. What level of systolic arterial pressure
can result in blocking of renal filtration?
a 105/70 mm Hg;
b 100/60 mm Hg;
c 70/50 mm Hg;
d 95/60 mm Hg;
e 90/50 mm Hg.
124. What is drug of choose for treatment of
anaphylactic shock?
a Rhiboxine;
b Infusion of 5% glucose solution;
c Adrenaline;
d Glucocorticoids;
e Cordiamine.
125. Choose the appropriate remedies for
treatment of hypovolemic shock:
a Ringer-Locks solution;
b Isotonic solution of sodium chloride;
c Trisol solution;
d 10% albumin solution;
e Rheopolyglucine.
126. Which clinical signs are typical for
pulmonary edema? (2)
a Crepitating rales on auscultation of
lungs.
b Severe dyspnea while the patient is
at rest in the bed;
c Diffuse rales of large calibers on
auscultation of the lung;
d Bradycardia
e Noise of pleura friction
127. Choose the early clinical signs of acute
liver failure (2).
a Disorders of color sensation;
b Leucopoenia in CBC;
c Progressive decreasing of liver size;
d Disorders of consciousness;
e Low level of prothrombine index.
128. Choose the drug for treatment of
comatose form of malaria caused with
resistant to chloroquine plasmodia.
a Primaquine;
b Ribavirin;
c Zydovudin;
d Quinidine gluconate;
e Metronidazole.

129. What correlation of colloid and saline


solutions is optimal for intensive treatment
of septic shock?
a 1:1;
b 1:2;
c 2:1;
d 1:3;
e 3:1.
130. What daily dosage of dextrans is safe
for treatment of infectious toxic shock?
a 3 liters;
b 500 ml;
c 10 ml;
d 800 ml;
e 2 liters.
131. Which complications are possible when
dextrans are overdosed? (3)
a Necrosis of large muscles;
b Decreasing of acuteness of hearing;
c Brain edema;
d Pulmonary edema;
e Necrosis of epithelium cells of renal
tubules.
132. What speed of saline solutions infusion
you must keep during the first stage of
treatment of patient with hypovolemic
shock?
a 1 l/h;
b 3 l/h;
c 100-120 ml/min;
d 60 drops/min;
e 50 ml/min.
133. What drugs should be given for
patients with septic shock, refractory to
volume replacing treatment and to
glucocorticoids?
a To continue infusion of high doses of
glucocorticoids (up to 1000-1500
mg);
b To begin intravenous infusion of
dofamine;
c Treatment with enterosorbents;
d To begin transfusion of blood;
e To use lymposorbtion or
plasmosorbtion.
134. Choose 2 of all laboratory signs typical
for acute respiratory failure:
a PaO2 <50 mm Hg;
b PaCO2 >50 mm Hg;
c PaO2 40 mm Hg;
d PaCO2 40 mm Hg;
e pH of the blood 7,38.
135. Choose clinical sign that typical for
croup of II stage.
a Whistle dry rales over the lungs;
b Inspiratory stridor;
c Comatose state;
d Discharging of plentiful foamy
sputum;

88

Diffuse moist rales of different


calibers on auscultation of lungs.
136. Indicate the most characteristic clinical
signs of acute broncoobstruction (2).
a Whistle dry rales over the lung;
b Dry barking cough;
c Tympanic sound on percussion of
lungs;
d Discharging of abundant foamy
sputum;
e Diffuse moist rales of different
calibers on auscultation of lungs.
137. Indicate the most characteristic clinical
signs of pulmonary edema (2):
a Whistle dry rails over a lungs;
b Severe pain in the chest on inhale;
c Comatose state;
d Discharging of plentiful foamy
sputum;
e Diffuse moist rales of different
calibers on auscultation of lungs.
138. Choose appropriate early clinical and
laboratory signs of acute respiratory failure
(2):
a Dyspnea;
b PaO2 40 mm Hg;
c Diffuse cyanosis;
d PaCO2 40 mm Hg
e The work of additional muscles on
breathing
139. Which clinical and laboratory signs are
typical for acute renal failure (ARF) (2)?
a Dryness of mucous membranes;
b Level of creatinine in the blood 0,20
mkmol/l;
c Level of urea in the blood 0,08
mkmol/l;
d High temperature;
e Pain in abdomen.
140. Oliguria it is a state when volume of
diuresis is:
a up to 1000 ml;
b up to 700 ml;
c up to 500 ml;
d up to 300 ml;
e up to 100 ml.
141. Which volume of diuresis we can
classify as anuria?
a 500 ml;
b 300 ml;
c 400 ml;
d 600 ml;
e
450 ml.
142. These are all clinical and laboratory
signs of acute renal failure beside of:
a vomiting;
b diarrhea;
c dryness of mucous membranes;
d urea of the blood 0,08 mmol/l;

creatinine of the blood 0,27


mmol/l.;
143. Which infectious diseases can result in
acute renal failure (3)?
a Leptospirosis;
b Brucellosis;
c Haemorrhagical fevers;
d Salmonellosis;
e Cholera.
144.Indications for hemodialysis in patients
with acute renal failure are all following
beside of (2):
a Potassium of plasma 5,7 mmol/l;
b Urea of plasma 0.24 mkmol/l;
c Creatinine of plasma 0,5 mkmol/l;
d Creatinine of plasma 0,3 mkmol/l;
e Longevity of anuria stage more than
10 days.
145.Which drugs are effective for treatment
of acute respiratory failure (2)?
a Infusion of 10% solution of glucose;
b Infusion of 5% solution of glucose;
c Glucocorticoids;
d Prozerinum;
e Artificial pulmonary ventilation.
146. Which drugs can suppress formation of
foam in patients with pulmonary edema (2)?
a Inhaling of 78% ethanol alcohol;
b Pentaminum i/m;
c Inhaling of oxygen mixed with air;
d Inhaling of ether;
e Inhaling of antiphomsilanum.
147. What clinical signs are typical for
diphtheric croup:
a Sudden beginning with fast
development of laryngeal stridor and
wavy course;
b Sudden development of laryngeal
stridor during eating or game,
absence of fever and catharal
symptoms;
c Gradual development of laryngeal
stridor, frequently normal body
temperature, multiple of dry rales;
d Fever, hoarse voice, consequent
development of barking cough,
dysphonia, strydor, inspiratory
dyspnea, cyanosis of lips,
tachycardia.
148. What clinical signs are typical for
aspiration of foreign body?
a Sudden beginning with fast
development of laryngeal stridor and
wavy course;
b Sudden development of laryngeal
stridor during eating or game,
absence of fever and catharal
symptoms;

89

Gradual development of laryngeal


stridor, frequently normal body
temperature, multiple of dry rales;
d Fever, hoarse voice, consequent
development of barking cough,
dysphonia, stridor, inspiratory
dyspnea, cyanosis of lips,
tachycardia.
149. What clinical signs are typical for croup
as complication of influenza and ARVI:
a Sudden beginning with fast
development of laryngeal stridor and
wavy course;
b Sudden development of laryngeal
stridor during eating or game,
absence of fever and catharal
symptoms;
c Gradual development of laryngeal
stridor, frequently normal body
temperature, multiple of dry rales;
d Fever, hoarse voice, consequent
development of barking cough,
dysphonia, lengthened sonorous
breath, inspiratory dyspnea, cyanosis
of lips, tachycardia.
150. Chouse main pathogenic mechanism of
toxic shock:
a Dehydration;
b Primary decrease of cardiac input;
c Pathological increase of vascular
capacity;
d Severe deficiency of steroid
hormones;
151. Chouse main pathogenic mechanism of
cardiogenic shock:
a Dehydration;
b Primary decrease of cardiac input;
c Pathological increase of vascular
capacity;
d Severe deficiency of steroid
hormones;
152. Chouse main pathogenic mechanism of
hypovolemic shock:
a) Dehydration;
b Primary decrease of cardiac input;
c Pathological increase of vascular
capacity;
d Severe deficiency of steroid
hormones;
153. Which of basic logical Kochs postulates
can proof, that a microorganism causes the
disease?
a) Microorganism must be present in
every case of disease, but absent in
healthy individuals;
b) Suspected microorganism must be
isolated from affected person and grown
in culture;

c) The same disease must result when


isolated microorganism is introduced into
healthy persons;
d) the same microorganism must be
isolated again from second affected.
e) All mentioned

90

Every why has a therefore.

1 B 8 E
7 C 0

ADDITION

1 A 8 B
8 D 1

ANSWERS FOR TESTS FOR 5th YEAR


STUDENS
NA NA N
n
n
s
s
.
.
1 A 6 B 1
4 D 2
7
2 C 6 A 1
5 C 2
8
3 A 6 A 1
6
2
9
4 C 6 B 1
7
3
0
5 E 6 B 1
8
3
1
6 B 6
1
9
3
2
7 C 7 C 1
0
3
3
8 B 7 A 1
1
3
4
9 C 7 D 1
2
3
5
1
7 A 1
0
3
3
6
1 B 7 D 1
1 D 4
3
7
1 E 7 C 1
2
5
3
8
1 B 7 B 1
3
6
3
9
1 E 7 A 1
4
7
4
0
1 B 7 B 1
5 E 8
4
1
1 C 7 A 1

A NA N
n
n
s
s
.
.
D 1 A 2
9
5
0
3
B 1 D 2
9
5
1
4
D 1 C 2
9
5
2
5
B 1 D 2
9
5
3
6
E 1 D 2
9
5
4
7
C 1 B 2
9
5
5
8
B 1 C 2
9
5
6
9
B 1 D 2
9
6
7
0
B 1 C 2
9
6
8
1
C 1 D 2
9
6
9
2
A 2 D 2
0
6
0
3
B 2 B 2
0
6
1
4
A 2 C 2
0
6
2
5
C 2 D 2
0
6
3
6
B 2 B 2
0
6
4
7
A 2 C 2

A NA N
n
n
s
s
.
.
A 3 E 3
1
7
6
9
C 3 E 3
1
8
7
0
B 3 B 3
1 C 8
8
1
D 3 A 3
1 B 8
9 D 2
B 3 C 3
2
8
0
3
B 3 A 3
2
8
1
4
C 3 B 3
2
8
2
5
A 3 C 3
2
8
3
6
C 3 A 3
2
8
4
7
B 3 D 3
2
8
5
8
C 3 A 3
2
8
6
9
C 3 B 3
2
9
7
0
A 3 B 3
2
9
8
1
A 3 D 3
2
9
9
2
A 3 B 3
3
9
0
3
B 3 C 3

A NA N
n
n
s
s
.
.
B 4 A 5
4 D 0
2
5
A 4 C 5
4 D 0
3
6
C 4 A 5
4
0
4
7
A 4 D 5
4
0
5
8
B 4 B 5
4
0
6
9
D 4 C 5
4
1
7
0
B 4 B 5
4 F 1
8
1
B 4 B 5
4
1
9
2
C 4 C 5
5
1
0
3
B 4 A 5
5 E 1
1
4
B 4 D 5
5
1
2
5
A 4 B 5
5
1
3
6
E 4 A 5
5
1
4
7
A 4 B 5
5
1
5
8
A 4 D 5
5
1
6
9
B 4 E 5

1 B 8 D
9
2

A
n
s
.
D

2 C 8 A
0
3
2 B 8 B
1
4 E

2 A 8 B
2
5

2 C 8 B
3
6

2 C 8
4
7

2 A 8
5
8

2 C 8 D
6
9

2 B 9 B
7
0

2 D 9 E
8
1

2 A 9 D
9
2

3
0

9 C
3

3 B 9 A
1
4
3 D 9 C
2
5

3 C 9 A
3
6

3 A 9 D
4
7

3 B 9 B
5
8 D

91

4
2
1
4
3
1
4
4
1
4
5
1
4
6
1
4
7
1
4
8
1
4
9
1
5
0
1
5
1
1
5
2
1
5
3
1
5
4
1
5
5
1
5
6
1
5
7
1
5
8
1
5
9
1
6
0
1
6
1

A
B
E
J
B
A
C
E
B
B
C
D
C
A
B
D
B
C
C
E
C

0
5
2
0
6
2
0
7
2
0
8
2
0
9
2
1
0
2
1
1
2
1
2
2
1
3
2
1
4
2
1
5
2
1
6
2
1
7
2
1
8
2
1
9
2
2
0
2
2
1
2
2
2
2
2
3
2
2
4

B
A
C
D
B
A
C
A
B
B
C
C
D
B
D
E
C
B
D

6
8
2
6
9
2
7
0
2
7
1
2
7
2
2
7
3
2
7
4
2
7
5
2
7
6
2
7
7
2
7
8
2
7
9
2
8
0
2
8
1
2
8
2
2
8
3
2
8
4
2
8
5
2
8
6
2
8
7

B
C
C
D
B
A
B
E
D
E
A
E
B
C
C
D
A
D
C

3
1
3
3
2
3
3
3
3
3
4
3
3
5
3
3
6
3
3
7
3
3
8
3
3
9
3
4
0
3
4
1
3
4
2
3
4
3
3
4
4
3
4
5
3
4
6
3
4
7
3
4
8
3
4
9
3
5
0

A
D
A
B
C
A
A
B
D
B
A
D
B
B
D
C
B
B
C

9
4
3
9
5
3
9
6
3
9
7
3
9
8
3
9
9
4
0
0
4
0
1
4
0
2
4
0
3
4
0
4
4
0
5
4
0
6
4
0
7
4
0
8
4
0
9
4
1
0
4
1
1
4
1
2
4
1
3

B
A
B
E
A
A
B
C
E
A
B
A
B
B
C
A
A
A
A
A

5
7
4
5
8
4
5
9
4
6
0
4
6
1
4
6
2
4
6
3
4
6
4
4
6
5
4
6
6
4
6
7
4
6
8
4
6
9
4
7
0
4
7
1
4
7
2
4
7
3
4
7
4
4
7
5
4
7
6

D
A
C
B
A
D
C
B
A
C
C
D
B
A
B
B
C
A
B

2
0
5
2
1
5
2
2
5
2
3
5
2
4
5
2
5
5
2
6
5
2
7
5
2
8
5
2
9
5
3
0
5
3
1
5
3
2
5
3
3
5
3
4
5
3
5
5
3
6
5
3
7
5
3
8
5
3
9

C
B
A
C
D
B
C
E
D
C
D
E
A
E
E

B
A
B

3 B 9 B 1
6
9 C 6
2
3 A 1 A 1
7
0 B 6
0
3
3 C 1 D 1
8
0
6
1
4
3 B 1 C 1
9
0
6
2
5
4 A 1 E 1
0
0
6
3
6
4 B 1 C 1
1
0
6
4
7
4 D 1 D 1
2
0
6
5
8
4 B 1 B 1
3
0 C 6
6
9
4 C 1 E 1
4
0
7
7
0
4 A 1 C 1
5
0 E 7
8
1
4 C 1 A 1
6
0 B 7
9
2
4 C 1 B 1
7
1 D 7
0
3
4 C 1 A 1
8
1 C 7
1
4
4 A 1 A 1
9
1 B 7
2
5
5 A 1 E 1
0
1
7
3
6
5 A 1 D 1
1
1 E 7
4
7
5 B 1 E 1
2
1
7
5
8
5 C 1 B 1
3
1
7
6
9
5 B 1 C 1
4
1
8
7
0
5 D 1 A 1
5
1
8

A 2
2
5
C 2
2
6
A 2
2
7
B 2
2
8
D 2
2
9
A 2
3
0
C 2
3
1
B 2
3
2
B 2
3
3
C 2
3
4
A 2
3
5
A 2
3
6
B 2
3
7
B 2
3
8
C 2
3
9
B 2
4
0
C 2
4
1
C 2
4
2
E 2
4
3
A 2
4

B 2
8
8
C 2
8
9
B 2
9
0
C 2
9
1
D 2
9
2
D 2
9
3
B 2
9
4
F 2
9
5
A 2
9
6
D 2
9
7
B 2
9
8
D 2
9
9
A 3
0
0
C 3
0
1
D 3
0
2
C 3
0
3
A 3
0
4
C 3
0
5
D 3
0
6
B 3
0

B 3
5
1
C 3
5
2
C 3
5
3
C 3
5
4
A 3
5
5
C 3
5
6
C 3
5
7
B 3
5
8
A 3
5
9
A 3
6
0
A 3
6
1
B 3
6
2
B 3
6
3
E 3
6
4
E 3
F 6
5
D 3
E 6
6
C 3
6
7
E 3
6
8
A 3
6
9
B 3
7

A 4
1
4
D 4
1
5
C 4
1
6
D 4
1
7
B 4
1
8
A 4
1
9
C 4
2
0
C 4
2
1
B 4
2
2
D 4
2
3
C 4
2
4
A 4
2
5
C 4
2
6
C 4
2
7
B 4
2
8
D 4
2
9
C 4
3
0
B 4
3
1
B 4
3
2
B 4
3

B 4
7
7
C 4
7
8
B 4
7
9
C 4
8
0
D 4
8
1
A 4
8
2
C 4
8
3
B 4
8
4
B 4
D 8
5
C 4
8
6
C 4
8
7
D 4
E 8
8
B 4
8
9
C 4
9
0
B 4
9
1
D 1
9
2
C 4
9
3
B 4
9
4
C 4
9
5
C 4
9

C 5
4
0
E 5
4
1
D 5
4
2
B 5
4
3
B 5
4
4
B 5
E 4
5
C 5
4
6
C 5
4
7
B 5
4
8
B 5
C 4
9
C 5
5
0
D 5
5
1
C 5
5
2
B 5
E 5
3
C 5
5
4
B 5
5
5
C 5
5
6
5
5
7
5
5
8
A 5
5

8
5 C 1
6
1
9
5 A 1
7
2
0
5 A 1
8
2
1
5 C 1
9
2
2
6 B 1
0
2
3
6 D 1
1
2
4
6 A 1
2
2
5
6 C 1
3
2
6

C
D
A
B
B
C
C
D
A
B
E
C
D
A
C
E
E
E
C

92

1
A 1
D 8
2
A 1
B 8
E 3
D 1
8
4
D 1
8
5
B 1
E 8
6
C 1
8
7
C 1
8
8
A 1
8
9

4
B 2
4
5
A 2
4
6
A 2
4
7
A 2
4
8
A 2
4
9
B 2
5
0
C 2
5
1
C 2
5
2

7
B 3
0
8
A 3
0
9
C 3
1
0
A 3
1
1
C 3
1
2
B 3
1
3
B 3
1
4
B 3
1
5

0
D 3
7
1
C 3
7
2
C 3
7
3
A 3
7
4
B 3
7
5
C 3
7
6
D 3
7
7
D 3
E 7
8

3
C 4
3
4
A 4
3
5
C 4
3
6
A 4
3
7
B 4
3
8
C 4
3
9
D 4
4
0
C 4
4
1

6
C 4
9
7
B 4
9
8
C 4
9
9
C 5
0
0
A 5
0
1
A 5
0
2
A 5
E 0
3
C 5
E 0
4

9
A
C
C
C
A
A
A
B

It is hors another color

acute tonsillitis, localized form of


diphtheria, acute HIV infection,
leucosis.
14 Diagnosis: enteroviral disease.
Differential
diagnosis:
rotaviral
infection,
intestinal
iersiniosis,
echerichiosis, salmonellosis.
15 Diagnosis: tularemia, eye bubonic
form.
Differential diagnosis: plague, sodocu,
felinosis, purulent lymphadenitis.
16 Diagnosis: arboviral tick- borne
encephalitis.
Differential diagnosis: polyomielitis,
rabies,
meningococcal
meningoencephalitis.
17 Diagnosis: serum disease.
Differential diagnosis: measles,
iersiniosis.
18 Diagnosis: tularemia, eye bubonic
form.
Differential diagnosis: plague, sodocu,
felinosis,
purulent
lymphadenitis,
infectious mononucleosis.
19 Diagnosis: anaphylactic shock
Differential diagnosis: toxic infectious
shock, acute poisoning.
20 Diagnosis: acute brucellosis.
Differential diagnosis: typhoid fever,
tuberculosis,
lymphogranulematosis,
reumatic fever, iersiniosis.
21 Diagnosis: measles.
Differential
diagnosis:
rubella,
enteroviral exantema, serum disease,
infectious mononucleosis.
22 Diagnosis: Brills disease (endemic
louse born typhus).
Differential diagnosis: influenza,
arboviral encephalitis, meningococcal
infection.
23 Diagnosis: brucellosis.
Differential diagnosis: tuberculosis,
sepsis,
lymphogranulematosis,
pseudotuberculosis,
reumatic
fever,
malaria.
24 Diagnosis: plague, bubonic form.
Differential diagnosis: tularemia,
sodocu, purulent lymphadenitis.
25 Diagnosis: infectious mononucleosis.
Differential diagnosis: cytomegaloviral
infection, felinosis, acute tonsillitis,
localized form of diphtheria, acute HIV
infection, leucosis, viral hepatitis A.
26 Diagnosis: tropical malaria
Differential diagnosis: sepsis, typhoid
fever,
influenza,
leptospirosis,
hemorrhagic fevers.
27 : rabies, hydrophobic form.

ANSWERS FOR CLINICAL CASES AND


TESTS FOR 6th YEAR STUDENS

Diagnosis: viral hepatitis


Differential diagnosis: hepatitis B, C,
D, leptospirosis.
2 Diagnisis: tropical malaria, malarial
coma.
Differential diagnosis: sepsis, typhoid
fever,
influenza,
leptospirosis,
hemorrhagic fevers.
3 Diagnosis: influenza.
Differential diagnosis: parainfluenza,
adenoviral
infection,
measles,
leptospirosis, paratyphoid A.
4 Diagnosis malaria tertiana.
Differential diagnosis: influenza,
recurrent typhus.
5 Diagnosis: Shigellosis.
Differential diagnosis: amebiazis,
salmonellosis,
echerichiosis,
intestinal iersiniosis.
6 Diagnosis: hemorrhagic fever with
renal syndrome.
Differential diagnosis: leptospirosis.
7 Diagnosis: Leptospirsis.
Differential diagnosis: viral hepatitis,
hemorrhagic
fever
with
renal
syndrome, pseudotuberculosis.
8 Diagnosis: opistorchosis.
Differential diagnosis: typhoid and
paratyphoid A, B; viral hepatitis C, B,
acute cholecistitis.
9 Diagnosis: diphtheria of larynx;
Differential diagnosis: parainfluenza.
10 Diagnosis:
rabies.
Hydrophobic
phase.
Differential
diagnosis:
tetanus,
arboviral encephalitis, atropine or
strychnine
poisoning,
delirium
tremens.
11 Diagnosis: polyomielitis.
Differential diagnosis: botulism,
arboviral
encephalitis,
lymphocytar
meningitis;
ECHO and Kocksaki diseases.
12 Diagnosis: plague, pulmonary form.
Differential diagnosis: pulmonary
form
of
antrax;
pneumococcal
pneumonia, influenza, complicated
with edema of lungs.
13 Diagnosis: Epstine Burr infectious
mononucleosis.
Differential
diagnosis:
cytomegaloviral infection, felinosis,

93

Differential diagnosis: tetanus, tick


born encephalitis, delirium tremens.

A man of words and not of deeds is


like a garden full of weeds.

ANSWERS FOR TESTS FOR 6th YEAR


STUDENS
NA NA N
n
n
s
s
.
.
1 1
3
7
3

A NA NA N
n
n
n
s
s
s
.
.
.
4
6
8
9
5
1

2 B 1
8

3
4

5
0

6
6

8
2

1
9

3
5

5
1

6
7

8
3

2
0

3
6

5
2

6
8

8
4

2
1

3
7

5
3

6
9

8
5

2
2

3
8

5
4

7
0

8
6

2
3

3
9

5
5

7
1

8
7

2
4

4
0

5
6

7
2

8
8

2
5

4
1

5
7

7
3

8
9

1
0

2
6

4
2

5
8

7
4

9
0

1
1

2
7

4
3

5
9

7
5

9
1

1
2

2
8

4
4

6
0

7
6

9
2

1
3

2
9

4
5

6
1

7
7

9
3

1
4

3
0

4
6

6
2

7
8

9
4

1
5

3
1

4
7

6
3

7
9

9
5

1
6

3
2

4
8

6
4

8
0

9
6

A N A N A N
n
n
n
s
s
s
.
.
.
9
1
1
7
1
2
3
9
9
1
1
8
1
3
4
0
9
1
1
9
1
3
5
1
1
1
1
0
1
3
0
6
2
1
1
1
0
1
3
1
7
3
1
1
1
0
1
3
2
8
4
1
1
1
0
1
3
3
9
5
1
1
1
0
2
3
4
0
6
1
1
1
0
2
3
5
1
7
1
1
1
0
2
3
6
2
8
1
1
1
0
2
3
7
3
9
1
1
1
0
2
4
8
4
0
1
1
1
0
2
4
9
5
1
1
1
1
1
2
4
0
6
2
1
1
1
1
2
4
1
7
3
1
1
1
1
2
4
2
8
4

f
g
h
i
j

A
n
s
.

inflammatory changes;
toxicoallergic inflammation of
microvessels ;
k generalized destructive-proliferative
thrombovasculitis;
l
all mentioned.
468. What of the following human systems
are most likely injured with HFRS pathogen?
a lymphatic and respiratory systems;
b respiratory and cardiovascular
systems;
c cardiovascular and urogenital
systems;
d urogenital and lymphatic systems;
e nervous and digestive system.
469. What type of affection of kidney is
typical for hemorrhagic fever with renal
syndrome?
f nephrosclerosis;
g pyelonephritis;
h glomerulonephritis;
i
nephrosonephritis;
j
all mentioned.
470. What are the symptoms of climax
period of the hemorrhagic fever with renal
syndrome?
f hepatomegaly and fever;
g oliguria and hemorrhagic rash;
h vomiting and cough with purulent
phlegm;
i
anuria and hepatomegaly;
j
diarrhea and vomiting.
471. What changes in common urine test is
typical for hemorrhagic fever with renal
syndrome?
f hematuria;
g crystaluria;
h leucocyturia;
i
oxalaturia;
j
piuria.
472. What is the main specific laboratory
method for diagnostic of HFRS?
f skin-allergic test;
g antibodies detection in blood by a
indirect immunofluorescence
method;
h pathogen detection in blood by a
method of the immunoenzym
analysis;
i
infection of laboratory animals
(biological test);
j
clinical analysis of blood.

94

473. What is the most typical complication


of hemorrhagic fever with renal syndrome?
f edema of the braine;
g acute renalfailure;
h meningoencephalitis;
i
peritonitis;
j
edema of the lungs;
474. Which one from the following drugs
may be recommended for treatment of
HFRS in climax period?
f Panangine;
g Furazolidone;
h Prednisolone;
i
Delagil;
j
Thymogen.

95

HERPESVIRUS INFECTION

h arbidol;
i
remantadin;
j
gancyclovir.
483. What is the main course of CMV
infection clinical manifestations:
f vitamin insufficiency;
g overcooling;
h immunodifficiency;
i
physical exhaustion;
j
decayed food using.
484. CMV infection affects all
mentioned below cells except:
f neutrophiles;
g monocytes;
h epiteliocytes of gastro intestinal
tract;
i
endocrine glands cells;
j
B lymphocytes.
485. Clinical forms of CMV includes all
manifestations except:
f retinitis;
g thireoiditis;
h uveltis;
i
hepatitis;
j
pneumonia.
486. Clinical forms of CMV includes all
manifestations except (2):
f prolonged fever;
g endocarditis;
h esophagitis;
i
nephritis;
j
meningitis.
487. What kind of antibody in high titers
shows the affection of newborn with CMV
infection:
f IgA;
g IgG;
h IgM;
i
IgS;
j
IgF.
488. What are the medication for CMV
infection:
f cricsivan;
g foscarnet;
h azidotimidin;
i
ganciclovir;
j
zovirax.
489. Human herpesvirus HHV 6 can be
the agent of all mentioned diseases
except:
f exanthema of newborns;
g chronic fatigue syndrome;
h meningoencephalitis;
i
B cell lymphoma;
j
All mentioned.
490. What two symptoms are not typical
for herpes zoster:
f fever;

475. Which pathogens are not belong to


herpesvirus group? (2)
f Hantavirus;
g HSV-1 and HSV-2;
h Cytomegalovirus;
i
*Bunyavirus;
j
Epstain-Barr virus;
476. What are the herpetic encephalitis
peculiarities:
f intracellular insertions formation
(Babech Negri corpuscles);
g demielinisation
and
fast
development;
h hemorrhagic vasculitis;
i
detachable neurons affection;
j
disturbance of the acethilcholine
synthesis in synapses.
477. What way of transmittion is not
typical for HSV infection:
f contact;
g sexual;
h through the blood;
i
transplacentar;
j
parinatal.
478. What are the typical course and
prognosis of the herpetic encephalitis:
f mild;
g moderate;
h severe;
i
favorable;
j
unfavorable;
479. What are the typical course and
prognosis of the herpetic meningitis:
f mild;
g moderate;
h severe;
i
favorable;
j
unfavorable;
480. What is the main way of
transmission of Varicella zoster infection:
f through blood;
g air droplet;
h alimentary;
i
sexual;
j
tick bites.
481. The Epstain Barr virus has
tendency to affect:
f neurocytes;
g B lymphocytes;
h T lymphocytes;
i
skin and mucous layers;
j
hepatocytes.
482. What are the two main antiherpetic
medications:
f ribavirin;
g acyclovir;

96

g
h
i

polyadenitis;
acute pain syndrome;
vesicular rush on the skin
streaming along dermatom;
j
herpetic tonsillitis.
491. What are two indications for
prescription of asiclovir in case of
herpetic infection:
f herpes labialis;
g varicella zoster;
h primary herpes genitals;
i
mumps;
j
encephalitis.
492. The Camp cells, typical for herpes
infection are:
f gigantic
multinuclear
with
intranuclear insertion;
g cells
with
dark
intranuclear
insertions,
encircled
with
enlightenment zone (owls eye);
h cells
with
cytoplasmic
eosinophilic insertions;
i
multinuclear
syncytium
with
necrosis of the nuclei;
j
polynuclear
syncytium
with
bazophil incertions.
493. What methods are most acceptable
for herpetic encephalitis diagnostics:
f clinical and epidemic;
g typical cells detection in blood
and urine samples;
h IgM antibodies detection;
i
Monoclonal Immunofluorescence
Reaction of brain cells;
j
All mentioned.
494. A 56 year old female during the
treatment of breast tumor noticed
painful
vesicles,
in
places
with
hemorrhagic content, on the lover part
of the chest in the 9 10 ribs area,
accompanied by fever 38C.
a What is the most likely diagnosis
and form of the disease?
b What is the main cause of the
disease?
495. A 5 year old child periodically
suffers from vesiclous rush on the
mucous of the mouth lusting for 3
weeks. Such rush was observed several
times during lust year.
a What is the most likely diagnosis?
b What medications should be
prescribed?
496. What is the mechanism of action of
aciclovir:
f blocks viral nucleinic acid release;
g viral revertase inhibitor;
h viral proteinase inhibitor;
i
cellular ribonuclease activator;

j
DNA synthesis inhibitor;
497. The Hant syndrome is:
d combination of facial nervus
paralysis, pain and vesicular rush
in the ear area;
e combination of back passage and
waterworks disturbances with
pain and vesicular rush in the
anal and genital area;
f combination of acute pain in the
mandibular area with vesicular
rush on the mucous of the mouth.
498. Herpetic infection is prone to affect
all mentioned organs and tissues except:
f eyes mycoses;
g liver;
h suprarenal glands;
i
bone marrow;
j
bronchial tubes.
499.
Varicelloform
eczema
Kapoci
occurs:
f accompanied by HIV infection;
g in aged persons;
h in a childhood together with
eczema;
i
in young parsons with allergoses;
j
in all age groups together with
helminthes affection.
500. What drugs can not be used for
herpetic infection treatment:
f zoviracs;
g biovir; cicloferon;
h retabolil;
i
thebrofen ointment;
j
all mentioned.
PLAGUE. TULAREMIA
501. What is the main way of transmission
of bubonic form of plague?
f insect transmission;
g alimentary transmission;
h aerial transmission;
i
horizontal transmission;
j
livestock transmission.
502.What is the main way of transmission of
cutaneus form of plague?
f insect transmission;
g water way transmission;
h aerial transmission;
i
horizontal transmission;
j
livestock transmission.
503. How many days are incubation period
of plague?
f 1-6;
g 3-7;
h 5-10;
i
7-14;
j
11-21;

97

504. How many days are incubation period


of tularemia?
f 1-6;
g 3-7;
h 5-10;
i
7-14;
j
11-21;
505. What of the following forms of plague is
externally disseminated?
f secondary septic;
g primary septic;
h bubonic;
i
secondary pneumonic;
j
primary pneumonic.
506. What of the following form of plague is
localized?
f secondary septic;
g primary septic;
h bubonic;
i
secondary pulmonic;
j
primary pneumonic.
507. What of the following drugs are used
for emergency prevention in case of contact
with a plague?
f erythromycin;
g penicillin;
h polymycxin;
i
streptomycin;
j
vancomicin.
508. What of the following group of lymph
nodes are most likely affected with bubonic
plague?
f cervical;
g inguinal;
h cubital;
i
mesenteric;
j
all mentioned.
509. What of the following group of lymph
nodes are most likely affected in case of
anginous form tularemia?
f cervical;
g inguinal;
h cubital;
i
mesenteric;
j
all mentioned.
510. In what period of pneumonic form of
plague the patients are most dangerous?
f incubation;
g initial;
h climax;
i
reconvalescention;
j
no one from mentioned.
511. What clinical form of plague more often
registered on a beginning of epidemic?
e primary pulmonic;
f bubonic;
g primary septic;
h cutaneus;
512. What is most often cause of death in

case of septic form of plague?


f acute renal inflammation;
g acute respiratory inflammation;
h inflectional toxic shock;
i
brain edema;
j
edema of lungs.
513. What is the first step of management
to patient with bubonic plague?
f prescription of immunoglobulin;
g local bubo therapy;
h desintoxication therapy;
i
antibiotic prescription;
j
dehydratation therapy.
514. What is the main method of plague
treatment?
f antibiotic therapy;
g serotherapy;
h desintoxication therapy;
i
dehydratation therapy;
j
rehydratation therapy.
515. On examination of patient doctor has
suspected a plague. What of the following
facilities should be informed first?
f city center of state sanitary
epidemiological surveillance;
g regional center of state sanitary
epidemiological surveillance;
h head of current medical facility;
i
police office;
j
relatives of a patient.
516. What skin phenomena are typical for
skin form of plague:
f Affected places of skin are sharply
outlined, jelly - like fluctuation of skin
during shaking. On center of edema
painless black ulcer 34 sm. with
yellow purulent edge. In outer zone of
hyperemia - vesicula, filled by dark red
contents.
g Skin is hyperemic with cyanotic shade,
edema is moderate. On centre of
edema black ulcer, 11 sm. without
purulent content. Edema and ulcer are
sharply painful.
h Moderate edema of skin, in centre of
edema zone is crater - like ulcer under
grey crusta, pus is excreted during
pressing. Highly expressed infiltration,
lymphadenit and lymphangit are
presented. Touching to edges of ulcer
is painful.
i Skin is markedly hyperemic, shining,
form is smoothed, a zone of affection
with precise festooned edges and a
platen on end. On center affection
hyperemia is less expressed, than on
edges.

98

99

ANTHRAX

prolypheration;
e) all mentioned.
525. What is the most typical variant of
cutaneous form of anthrax?
a) erysipeloid - like;
b) edematous;
c) bullosis;
d) carbunculosis;
e) bubonic.
526. What is the basic treatment method of
cutaneous anthrax?
a) bandage with ointment by
Vishnevsky;
b) antibacterial therapy;
c) opening carbuncle and drainage;
d) surgical revision of carbuncule;
e) supportive therapy.

517. What is the source of the anthrax


infection?
k livestock;
l man;
m insects;
n river fish;
o birds.
518. What inoculation way is most typical
for anthrax?
a) mucosa of respiratory tract;
b) injured skin;
c) mucosa of gastrointestinal tract;
d) conjunctiva;
e) all mentioned.
519. What kind of local inflammation is
typical for anthrax?
a) fibrinous;
b) productive;
c) serous - hemorrhagic;
d) suppurative;
e) granulematous.
520. What is contents of anthrax carbuncle
cavity?
a) serous liquor;
b) pus;
c) blood;
d) all mentioned.
521. What is the main factor of anthrax
pathogenesis?
a) autoallergy;
b) immunosuppression;
c) toxemia;
d) all mentioned;
522. What of the following are characterized
ofr anthrax carbuncle?
a) edema of tissue and acute pain;
b) edema and painless of tissue;
c) absence of tissues edema and
acute pain;
d) absence of tissues edema and pain.
523. What is the main method of anthrax
diagnostics?
a) bacteriological;
b) biological;
c) serological;
d) bacterioscopy;
e) allergiological;
524. What of the following is typical for
anthrax regional lymphadenitis?
a) painful lymph nodes and rapid
purulence;
b) painful lymph nodes and absence of
purulence;
c) painless lymph nodes and absence
of purulence;
d) painless lymph nodes and rapid

100

BRUCELLOSIS

j
All, except human.
535. What are the ways of brucellosis
transmission?
f Food borne;
g Air borne;
h Through damaged skin and mucosa;
i
Vertical;
j
All, except vertical.
536. What are basic clinical symptoms of
climax period of acute brucellosis:
f Constant remitting fever, chills,
significant sweating,
polylymphadenopathy, muscular
pain, spleen and lever enlargement;
g Constant type of fever, weakness,
adynamia, pale skin, poor roseolar
rush on lateral surfaces of trunk,
spleen and lever enlargement;
h Fever of remitting type, chills,
significant weakness, petechial
bleedings on conjunctiva, sometimes
liver enlargement;
i
Fever of intermitting type, on climax
of fever fit feeling of heat with
subsequent plentiful sweating,
spleen and lever enlargement.

527. What kind of brucellas are most


pathogenic for human?
f bovis;
g suis;
h melitensis;
i
ovis;
j
canis;
k neotomae;
528. What factors influence on duration of
incubation period of brucellosis?
f Pathogen activeness;
g Quantity of infecting dose;
h Way of transmission;
i
Reinfection;
j
All mentioned.
529. Which of the following is the reaction of
sensibility tissue with specific brucellar
antigen?
a) reaction of Coombs;
b) Wrights reaction;
c) Raection of Heddlson.
d) Burnets reaction.
e) All mentioned.
530. What is migration of brucells:
e transfer of brucells from one organ to
another of main host;
f transfer of brucells from one animal
to another of the same species;
g transfer of brucells from main animal
host to another of different species;
h all mentioned.
531. Inoculation gates of brucells into
human organism:
f damaged skin;
g mucous;
h gastrointestinal tract;
i
all mentioned.
532. What methods for diagnostics of
brucellosis:
f Bacterioscopy;
g Bacteriological method (cultivation);
h Serological method;
i
Allergic test;
j
All, except bacterioscopy;
533. Is there a cross immunity after
infection with different types of brucells
possible?
f yes;
g no.
534. What are the sources of brucellosis?
f Affected human;
g Main hosts of different types of
brucells;
h Another animais, affected with
migrated brucells;
i
Rodents;

101

a
b
c
d
e

Q-FEVER
537. What is the drug for specific therapy of
Q-fever?
a) penicillin;
b) tetracycline;
c) furazolidone;
d) streptomycin;
e) cloroquin.
538. What cardiovascular disturbances are
typical for climax period of Q-fever?
f tachycardia and hypotonia;
g relative bradicardia and moderate
hypotonia;
h bradicardia and hypertonia;
i
tachycardia and hypertonia;
j
all mentioned.
539. How many days are maximal duration
of Q -fever incubation period?
f 5;
g 14;
h 25;
i
45;
j
50
540. What is the basic method for specific
laboratory diagnostic of Q - fever?
bacteriological blood investigation;
serum blood investigation;
skin-allergic test;
bacteriological investigation of faeces;
bacteriological investigation of cerebrospinal
liquor.
541. What of the following system are
typically affected with Q-fever?
f nervous system;
g cardiovascular system;
h respiratory system;
i
alimentary system;
j
all mentioned.

102

TOXOPLASMOSIS
542. For what of parasitoses a cat is a main
source of invasion?
f Amebiasis;
g Balantidiasis;
h Lambliosis;
i
Toxoplasmosis;
j
Pneumocystosis.
543. For what form mentioned pathogens
transplacentar transmission is possible?
f Toxoplasma;
g Lamblia intestinalis;
h Balantidium;
i
Coccidium;
j
Negleria.
544. Where does the intestinal phase of
development of toxoplasm occure?
f in human;
g in pigs;
h in large horned cuttle;
i
in cats;
j
all mentioned.
545. What is the source of toxoplasmosis
infection?
a) human;
b) animals from feline family;
c) large horned cuttle;
d) pigs;
e) insects.
546. What does the life cycle of toxoplasma
development in the human organism
include?
a) sporogony;
b) endodiogeny (internal budding);
c) gametogony;
d) schizogony;
e) all mentioned.
547. What kind of the immunity from
toxoplasmosis?
a) stable, sterile;
b) unstable, sterile;
c) stable, unsterile;
d) unstable unsterile;
e) all mentioned.
548. In what form toxoplasms are kept in the
organism of human in latent toxoplasmosis?
a) oocysts;
b) pseudocysts;
c) tachyzoites (endozoites);
d) tissure cysts;
e) chizonts.
549. What is the drug for specific treatment
of toxoplasmosis?
a) chloridin (pyrimethamin);
b) penicillin;
c) chloroquine (delagil);
d) metronidazole (trichopol);
e) levomycetin.

SEPSIS.
550. Choose possible causative agent of
sepsis from following:
a Sh. Sonnei;
b Str. Pneumonie;
c Fr. Tolarence;
d R. Prowazekii:
e S. Enteritidis.
551. What is the main route of transmission
of sepsis?
f Air-droplet;
g Alimentary;
h Transplacentar;
i Contact;
j From supportive foci of organism.
552. Is recovering from sepsis possible
without proper treatment?
e Yes, due to formation of stable immunity;
f Yes, unfavorable outcome possible only
among children and old persons;
g No;
h Yes, unfavorable outcome possible only in
HIV infected persons.
553. Clinical sings of sepsis are all following
except:
f Petechial rash;
g Jaundice;
h High long temperature;
i Frequent liquid stool which contain mucus
and blood;
j Hepatosplenomegaly.
554. Express methods of sepsis diagnosing
is:
f Bacterioscopy of thick drop of blood;
g Blood culture;
h Serological methods;
i Immunofluorescent test;
j Allergic test.
555. In case staphylococcal sepsis the most
effective antimicrobial drug is:
f Penicillin G in high dose;
g Cephalosporins of III generation;
h Vancomycin;
i Macrolides of III generation;
j Tetracyclines.
556. The main complications of sepsis are
all except:
f ARDS;
g Septic shock;
h Acute renal failure;
i Edema of the brain;
j Degidrative shock.
557. Refractory septic shock it is condition
when:
f Infusion therapy is not effective;
g Complex antibiotic treatment is not
effective;

103

h Glucorticoides are not effective;


i Immune therapy is not effective.
j Infusion therapy and corticoids are not
effective
558. Most mediators of inflammation act as:
f Activators of immune system;
g Vasodilatators;
h Suppressors of immune system;
i Factors increasing permeability of
microvessels;
j Vasodilatators and increasing permeability
of microvessels.
559. Among the drugs which act as
immunocorrectors in case of sepsis is most
effective:
a Timalin;
b Interferon;
c Ronkoleikin;
d Inductors of endogenous interferon (amixin)
e T-activin.

CASE 2
The patient ., 35 years. Has admitted in
clinic in a severe state, unconsciousness,
periodically
clonic
cramps.
From
accompanied persons was found out, that
patient was on business trip in Guinea.
Come back one week ago, at once fell ill. All
days before had a temperature, kept on high
figures (39-40C) and only on mornings
sometimes normal. Patient was complaining
on sharp headache, chilling, feeling of fever.
Had taken Analginum, but the state was not
improved: the headache accrued, the
sleepiness has appeared, and then had lost
consciousness,
the
cramps
began,
excitement have appeared.
On examination: skin and visual mucous
slightly icteric, respiration of Chain Stocks,
tachycardia. Pulse - 130 /min., weak filling
and strain. BP - 90/40 mm hg.. Mild wet
variegated crepitating. The lien is palpated
indistinctly. Pupillary reactions are flaccid,
miosis, tendinal hyperreflexia. A mild rigidity
of a neck muscles.
f What is the diagnosis?
g What is the form of disease?
h What are urgent states of
current disease?
i
Make a diagnostics and
treatment plan.

TESTS AND CLINICAL CASES FOR 6TH


YEAR STUDENTS

CLINICAL CASES
CASE 1
The patient ., 44 years old, has admitted
for 10-th day of disease with complaints on
common weakness, irritability, loss of
appetite, jaundice, darkening of urine. The
disease began gradually with fatigue,
nausea. On 4th day has noted darkening of
urine, the feeling of weight in the right
appear part of abdomen has appeared.
Current
state
had
connected
with
exhaustion. On 8th day the jaundice has
appeared, the feces has brightened. Was
guided to hospital by general practitioner.
On examination: common state is moderate,
temperature 36,8C. Moderate jaundice of
mucous and skin was observed. Heart and
lungs without singularities. Pulse - 60 1/min..
BP - 115/70 mm hg.. Tongue is a little
edematous, dryish, coated by brownish fur.
Meteorism of I degree. Liver was enlarged
on + 2 sm., edge acute, surface was
smooth, painful during palpation.
a What is the diagnosis?
b Give a plan of laboratory
examinations and treatment.
c What outcomes of disease are
possible.
d Make a differential
diagnostics.

CASE 3
Patient fall ill acute with rising of
temperature up to 39,0C, shill, headache in
frontotemporal areas, filling of general
unwell, muscle and joint pains. On the
second day has joined block of nose
breathing, moderate excretions from a nose,
dry often cough, feel tickle in a throat and
pain in throat and behind a breast bone
during coughing. On examination by the
doctor on 3-th day of disease temperature
38,2 C, injection of vessels of a conjunctiva
and blepharons, scleras and soft palate
hyperemia was observed. On auscultation hard respiration, simple dry crepitation.
a
What is most likely diagnosis?
b
The plan of laboratory examination.
c
What urgent states are characteristic
for a current disease?
d
What medical tactics are preferable?
CASE 4
The patient 39 years old, admitted to the
clinic of infectious diseases on 12th day of
the disease after acute onset with startling
shill, which was replaced by feeling of fever,
temperature has raised up to 39,5C with

104

indications of intoxication for 6 hours and


critically decreased up to 36,5C with
massive sweating. The attacks are repeated
every 3-rd day. On examination: herpes
labialis, sub icterus of scleras. The cardiac
sounds are weak. The moderately expressed
hepatolienal syndom was observed. In the
clinical blood analysis - moderate anemia,
leukopenia, lymphomonositosis, ESR - 21
mm / hour.
e What is the diagnosis,
laboratory diagnostics plan.
f Prescribe a treatment.
g What are possible urgent
states during treatment?

Diagnostics. Possible urgent


states?
m Medical tactics.
CASE 7
The patient ., 25 years old, was admitted
in infectious hospital on 2-th day of disease
with the complaints on headache, back pain,
in calf muscles, shivering. His state was
moderate, scleras subicterical, mucous of
pharynx is hiperemic. Tongue dry, coated
with brown fur. The stomach is inflated. The
liver is 2 sm lower than edge of a rib arc. A
lien at edges of a rib arc. The palpation of
muscles, especially calf is painful, dark
urine, feces of usual color.
a Mention urgent states, which
appropriate for this pathology.
b Their diagnostics and
treatment.

CASE 5
The patient 22 year old, student,
admited in clinic on 4-th day of disease, with
the complaints on general weakeness,
spasmatic pain in abdomen, frequent liquid
stool with slime and blood. The disease
began from shill, headache, tenesms and a
liquid stool. Patient lives in isolated
apartment. All relatives are healthy. Week
before disease has come back from village,
where were cases of similar disease. A
common state is moderate. Temperature is
37,5C. A skin pale, tongue wet, coated with
white fur. The cardiac sounds are weakened.
The abdomen is moderately inflated, painful
during palpation of thick sigmoid intestine.
Stool up to 10 times per day, liquid, with
slime and blood.
h Formulate and prove the
clinical diagnosis.
i
Prescribe laboratory
diagnostics for
acknowledgement of the
diagnosis.
j
What are possible
complications and urgent
states during the current
disease?

CASE 8
The inhabitant of the Tyumen area
(Russia), frequently used a fresh fish, home
made solty pike caviar. Within last months
notes common weakness, dizziness, pain in
tongue, muscles, weight loss and excretion
with a feces particles of helminths. The scin
is pale, edemas on shin and loins. The
cardiac sounds are weakend, systolic
murmur on the apex. BP 100 / 70 mm. hg.
The liver enlarged on 2 sm. Tongue is bright
- red, with cracks.
The analysis of a blood: erythrocytes - 1,3 *
1012 1/, haemoglobin - 50 g/l, CI - 1,2, in
sample
megaloytes,
poikilocytes,
erythrocytes with corpuscles of Joli and rings
of Kebot.
a What is most likely diagnosis?
b Make diagnostics and
treatment plan.
c Complications. Their
diagnostics and treatment.
CASE 9
Female patient, 45 year old, works as
a train conductor. Was admitted on 10-th
day of disease with the diagnosis of
peritonsillar abscess which was by the
otolaryngologist, but suppuration was not
obtained. The disease began gradually from
sore throat, then the rasping tussis
accompanied, hoarse of a voise, further aphonia, noisy labored respiration has
joined. Was admitted to hospital in a severe
state. Cyanosis, dyspnea with participation
of an auxiliary muscles. Frequency of
respiration 52 /min., sweating, paroxysmal
tachycardia.
Cardiac
sounds
dummy.

CASE 6
The patient 50 year old, watchman of
a depot posed on edge of a forest. The
disease began from rising of temperature up
to 39C, shill, headache. From 2-th day of
disease - back pain, from 3-rd - nasal
bleeding. On 5-th day of disease, on date of
entering in clinic, state severe - delirium,
tremor of limbs, hyperemia of the face and
neck. An abundant hemorrhagic rush on skin
of a trunk and extremities. Oliguria was
observed.
k Make your preliminary
diagnosis, substantiation.

105

Periodic attacks of asphyxia, cyanosis,


edema and hyperemia of a soft palate,
tonsils; on tonsils - firm grey film. During
laryngoscopy fibrinous grey color film was
detected of a on a mucous of larynx. From
Throat and the larynx dense film were
extracted.
a Give your diagnosis, plan of
diagnostics and treatment.
b What
are
possible
complications,
their
diagnostics and treatment.

joined together with coldening and cianosis


of legs. The sensitivity is saved.
f Make your diagnosis accounting
phase of disease.
g The differential diagnosis?
h The plan of diagnostics?
i
What complications are possible?
j
Your therapeutic tactics?
CASE 12
The male patient ., 48 year old.
Works in a wool reception depot. Was
admitted to clinic on 2 - th day of the
disease in severe condition with complaints
on dyspnea, expectoration of serum hemorrhagic sputum (about 1 l/day), pain in
a thorax, connected with breathing. The
disease
began
suddenly
with
chill,
weakness, the dry cough, then with
expectoration of a great amount of serum
and serum - hemorrhagic sputum. It is
known from epidemic anamnesis, that
during his work in a depot, accepted and
unpacked bales with a wool delivered from
other countries. Among the employees in a
place cases of influenza took place. In the
past history had had only common cold.
On examination: general condition was
serious, consciousness saved, but
sometimes patient is inadequate. Skin
without eruption, wet. A mucous of
nasopharynx slightly hyperemic. Tongue was
dry, coated with dirty fur. Respiration
through a nose and mouth 40 / min. On
percussion above lungs, in interscapular
area shortening of a pulmonary sound, on
auscultation there are a lot of dry and varied
wet crepitation. Pulse - 120 /min.. Blood
pressure - 80/40 mm hg.. The lien at edge of
a rib arc.
e
What is preliminary diagnosis?
f
Tactics of the doctor in the anti
epidemic ration?
g
The plan of examination.
h
What complications are typical for
the disease?
i
Medical measures.

CASE 10
The male patient, 36 year old, track driver,
was admitted in a severe condition with
complaints on headache, sleeplessness,
impossibility of swallowing of liquids. From
the anamnesis was found out that the last
year on hunting has caught the fox, has
brought it to the home, but it has escaped.
There was no bites, but multiple scratches.
The persons face hiperemic, with ingection
of scleras, hipersalivation. Complaints on
irritability from light and driving of air.
Movement of the lower extremities labored;
an abundant sweating. Periodic convulsive
spastic of throat muscles and larynx, the
respiration labored. exophtalm. Pulse -160
1/min. BP elevated.
The noise of pouring water and movement
of air is causing convulsive attacks, between
them - the patient is adequate. Asks about
the help.
a Make your diagnosis.
b What phase of the disease?
c The plan of laboratory
diagnostics? Medical tactics?
d With what it is necessary to
differentiate?
e Outcomes of current disease?
CASE 11
The male patient G., pupil, is
admitted in clinic of infectious diseases on
3-th day of disease , with the complaints to
lack of motions in the lower extremities and
sharp pain in muscles, high temperature,
which began to reduce. However, with drop
of temperature the pain in the lower
extremities, loins, arms and neck has
aggravated. Patient in consciousness. Within
several hours for 3-th day of disease the
flabby paralyses have appeared in the
muscles of legs, arms with a dominance of a
proximal affection of extremities (femur,
shoulders). The function of sphincters is
reduced. The atony of muscles, areflexia has

CASE 13
The patient ., 18 year old, has
admitted in clinic on 8 - th day of the
disease with the complaints to common
fatigue, headache, pain in throat during
swallowing, high temperature, sweating. The
disease began with malaise, subfebrile
temperature, pain in a throat. Frequent
tonsillitis in past history. On examination:
temperature 37,7C. Paleness of scin,
without a rush. Mucous of throat moderately

106

hyperemic. The tonsils enlarged in sizes,


oedematous, without purulent corks, films.
Axillary, submandibular, ugular lymph nodes
were observed, size 0,8 1 sm., slightly
painful, mobile. Cardiac sounds clear,
rhythmic, pulse - 94 /min.. On auscultation
vesicular sound. Tongue wet, coated with
white fur. Abdomen soft, painless. The liver
enlarged on 1 sm. Physiological excretions
are normal.
f Substantiate the diagnosis, laboratory
and differential diagnostics.
g What complications are possible?
h Medical tactics?

Anamnesis epidemica: there was no contact


with infectious patients, with animal also.
Last 2,5 weeks work with threshingmashine. On examination: moderate
condition. Scin of face hyperemic, dry.
Temperature 38,3C, S- 90 ./min., rhythmic,
satisfactory qualities. Eyelids of the left eye
dropsical, in an angle of left eye - white pus.
The conjunctivas is sharply dropsical, plenty
of small ulcers with purulent content.
Asymmetry of the face and neck at the
expense of enlarged subauricular and neck
lymph nods, morbid, tough, with clear eges
and movable. Mucous of throat are
moderately hyperemic. Tonsils are not
enlarged. Heart - tones clear. Respiration
vesicular with a hard nuance. Tongue dryish,
coated with fur. A stomach soft, painless
during palpation. Liver and lien are not
palpated.
The clinical blood test: red cell count without singularities, white - neutrophil
leukocytosis, ESR 25 mm/hr..
c What is most likely diagnosis?
d Methods of laboratory diagnostics
and therapy?

CASE 14
Female patient D., 30 year old. nurse of a
children local clinic department, was
admitted to infectious diseases hospital on 3
- th day of the disease with the complaints
on pain in throat, high temperature and
frequent liquid stool. 9 patients of teenage
age from improving camp with similar
symptoms were hospitalized in department.
Bright hyperemia of mucous of throat and
soft palate, palatine arcs, tonsils and back
wall of a pharynx. On a mucous of soft
palate
group
shallow
vesicles
with
transparent contents, enclosed by a red
crown, which in 2-3 days bursted with
formation of erosion. The pain in throat were
strengthened during swallowing. In 7 days
the
fever
has
decreased,
mucous
epitelisation
was
observed.
General
condition was improved, stool normalized.
e What is most likely diagnosis?
f The differential diagnosis?
g Methods of laboratory diagnostics?
h What complications are possible?
i
Medical tactics.

CASE 16
Patient , 44 years old, was admitted to the
hospital at the end of May with the
complaints on a strong headache of
pulsating character, photophobia, general
muscle pains, chill, fever, sleeplessness,
lack of appetite. 3-rd day of the disease. The
disease began from a mild malaise,
headaches, rising of temperature up to
37,4C, vomiting, photophobia, doesnt take
any medicine.
Anamnesis epidemica: 8 days before has
come back from Western Siberia (Abakan
district), lived there in a wood during 10
days. There were no contacts with ill people,
animals, mosquitoes, ticks bites.
OE: condition is moderate. The contact is
entered
well,
consciousness
clear,
temperature 39,0C; pulse - 80 1/min.,
rhythmic, the cardiac sounds are muffled.
On lung examination vesicular breathing.
Tongue is dryish. Abdomen soft during
palpation, painless. A liver, lien are not
enlarged. Mucous of the throat hyperemic. A
left-sided neck lymphadenitis was observed.
In left parietal part of the head - a crust of
brown color, a band of a tumescence around
it, the palpation is painful.
Neurologic status: pupillary reactions on
light are sluggish, the convergence is
broken. Mild assymetria of the face, slight
rygudity of neck muscles.

CASE 15
Female patient G, 38 year old
admitted in hospital on 4 - th day of disease
with the complaints on rising of temperature
to 38,3C, headache, dizziness, nausea,
weakness, pain in the left eye, massive
suppuration from eye, tumescence and pain
in the field of left ear and neck.
Anamnesis of disease : Onset was acute:
temperature up to 38 C, headache,
weakness. On second day the dizziness has
joined, a nausea, and to the end of 3 day
has appeared pains in left eye, swelling of
eyelids, next day - purulent discharge. Then
patient noticed swelling and pain in the field
of left ear and neck. Was hospitalized in a
hospital.
Past history - without singularities.

107

a
b

Your presumable diagnosis? Its


substantiation?
Therapeutic tactics?

contoured, relative frame, mobile. Mucous of


throat is moderately hiperemic. Tonsils
without significance. Heart - tones rhythmic,
clear. Respiration: vesicular with a hard
nuance. Tongue is dryish, coated with white
fur. Abdomen soft, painless. A liver, lien are
not enlarged.
The analysis of a blood: red - without
singularities,
white
-neuthrophil
leukocytosis, elevated ESR.
a What is the diagnosis?
b Methods of laboratory diagnostics
and therapy?

CASE 17
Patient M, 40 years, metal-turner of a
factory, was admitted to the infectious
hospital 4.06.79. with complaints on
headache, strong weakness, temperature,
pain in joints, inching abundant rush on skin.
One week ago has received a wound in the
field of the right foot joint, with the
preventive purpose antitetanic serum was
injected.
On ehamination: 7th day of the disease :
temperature 38,2C. Fase is dropsical and
pale. The enlarged submandibular, cervical
and axillar lymph nodes. Fingers of hands
are dropsical. On a skin of trunk, arms,
abdomen and lower extremities the maculopapular pruritic rush. CVS: tachycardia,
hypotonia.
In a blood analysis: leukopenia with a
relative lymphocytosis and accelerated ESR.
a) Your diagnosis and tactics of treatment of
the patient?

CASE 19
The patient , 32 years, admitted to
hospital after acute onset of the disease
with pharyngalgias and painful swallowing,
temperature 39,4C, headache. Penicillinum
and sulfodimetoxin were prescribed. Soon
the patient the felt a numbness of tongue,
lips, face; giddiness, headache, nausea,
shortness has appeared.
On examination of the patient: the face is
hyperemic, sweated, acrocianosis macular
inching rush. Pulse often, thread-like. BP 40/0 mm hg. Cardiac sounds dummy. Accent
of 2th tone on a pulmonary arteria. The
respiration often and superficial, later noisy
also was listened apart. In lungs box-like
pulmonary sound, during auscultation - dry,
whistling crepitation. Patient excited, rushes
in bed. A consensual urination and act of
defecation.
c What is most likely diagnosis?
d Make a treatment plan?

CASE 18
Patient G, 38 years was admitted to the
hospital on 4th day from beginning of the
disease with complains on temperature ap
to 38,8C, headache, dizziness, nausea, pain
in the left eye, abundant suppuration from
eye, tumescence and pain in region of left
ear and neck.
Anamnesis of disease : Onset was acute,
with temperature up to 38C, headache,
weakness. On the second day the giddiness
has joined, a nausea, and to the end of 3th
day has appeared pain in eye, tumescence
of blepharons, on 4th day - a suppuration.
Then patient has noted swelling and pain in
the field of left ear, necks. Was admitted to
the hospital.
Anamnesis of life - without singularities.
Anamnesis epidemica: there were no
contacts with infectious patients, animals.
During last 2,5 week worked with threshing
machine.
OE: a common state is moderate. The skin
of the face is hyperemic, dry. Temperature
38,3C,
uls90
1/min.,
rhythmic,
satisfactory qualities. Eyelids of left eye
hydropic, in an angle of an eye - white pus.
The conjunctiva is sharply hyperemic, large
number
of
ulcers
with
suppuration.
Asymmetry of the fase and neck at the
expense of enlarged auricular and anteriocervical lymph nodes, painful, tight, good

CASE 20
Patient ., 37 years, has complaints on
weakness, sweating, pain in knees, foot
joints. Temperature 37,6C.
Anamnesis of disease: consider himself ill
about a months. During this period bothered
weakness, pain in large joints. Repeatedly
sought for a medical help, was treated, but
without effect.
Anamnesis of life: a pneumonia, ARVI.
Anamnesis epidemica: patient a veterinarian
by profession. Had contact with a ill animal sheeps during lambing.
Objective examination: temperature 37,6 C.
A common condition satisfactory. Skin and
visual mucous of usual colouring. Edema,
morbidity of large joints was found; in the
field of joints in a hypodermic fat, tendons,
muscles the dense painful nodules are
palpated.
Vesicular sound on auscultation. The cardiac
sounds rhythmic, dull. Pulse - 80 1/min.,

108

satisfactory qualities. BP - 120/80 mm. hg.


Stomach during palpation soft, painless. The
edge of a liver is palpated on 1,5 sm below
of a rib arc. The lien is not enlarged. Stools
and urine passage are normal.
d Make your presumable diagnosis?
e
Make a plan of diagnostics?
CASE 21
Patient K, 21 year old, school teacher was
admitted to hospital on 5th day of the
disease after acute beginning with fever up
to 38,5C, running nose, frequent dry cough,
fear of bright light, epiphora. After treatment
at home temperature decreased to 37.5C.
On
3rd
day
condition
deteriorated,
temperature increased to 39.5C, headache
and nausea intensified; the same day rush
raised on face, neck, upper part of thorax
and then spread on whole body.
On admission: temperature 39.5C,
patient is languid. Face and eyelids are
dropsical. Conjunctives are hyperemic,
injection of sclera is observed. On skin of
body and face lavish macular - papular
rush, sometimes merged. Throat and palatal
mucosa are hyperemic. On chick mucosa,
opposite molars, spots of tender white fur
observed. During auscultation rough
breathing was found. Cardiac sounds are
moderately dull, pulse is 92/min..
What is your diagnosis?
b Make a differential diagnosis.
c What complications are possible?
d First step treatment measures.

f
g

What complications are possible?


Make diagnostics and treatment plan.

CASE 23
Patient I. ., 35 years, was admitted in
hospital with complaints on sweating, joint
pain, weakness, headache, temperature
39,5C, chill.
Anamnesis of disease: fell ill about 2 weeks
ago. The disease began from a headache,
weakness, sweating, temperature 37,538,0C with chill. Each time chill and rising
of temperature was finished with a profuse
sweating. Paroxysms of temperature
appeared in the afternoon. On 5th day of
disease visited the doctor. Was treated with
Aspirin, Analgin, but without effect. After
second visit was hospitalized. Despite of
remitting temperature with peak in the
afternoon, common condition of the patient
remained rather satisfactory.
Anamnesis of life: without singularities.
Anamnesis epidemica: during one year
works on a farm as a milkmaid, had contact
with the ill animals during calving.
OE: common condition is satisfactory.
Temperature 38C. The dermal integuments
of the person are a little acyanotic, the
eruption misses. The lymph nodes - cervical,
axillary, magnitude about a string bean,
painless are palpated. On auscultation:
lungs without singularities. The cardiac
sounds rhythmic, are muffled. Pulse 80
/mines., satisfactory qualities. Tongue
wet, pure. The stomach at a palpation weak,
painless, is palpated edge of a liver of a
below rib arc on 1. The lien is not
palpated. A stools and waterworks is normal
d Your diagnosis?
e The plan of treatment?

CASE 22
Patient K., 61-year-old pensioner, was
admitted to hospital with complaints on
headache, insomnia, chill, fever 39.3C, and
weakness. Fell ill 5 days before, was treated
by general practitioner with antipyretics and
anti-inflammatory drugs. Today on skin of
trunk roseolar and petechial rush rose. From
past history was known that during II World
War in partisan detachmen had louse-borne
typhus (30 years ago).
OE: General state moderate. Face skin is
dropsical and hyperemic; excitement, glitter
of eyes, moderate cyanosis of lips, pinch
symptom is positive. On palatine mucosa
fine enanthema was observed. Moderate
dyspnoe. On lung auscultation vesicular
breathing. Cardiac sounds are dull, systolic
murmur on top projection. PB is 100/60 mm
hg. Tongue is coated with fur, trembling
during examination. Abdomen is soft,
painless, lever and spleen enlarged on 2 sm.
e What is the diagnosis?

CASE 24
The young hunter, 25 years, was taken to a
hospital in a severe state expressed with
temperature 39,0C, repeated vomiting,
sharp hyperemia of the face, acrocianosis,
confused consciousness. From words of
accompanied his wife was established, that
the patient fell ill 3 days before. Onset of the
disease was acute with chill, headache,
vomiting, pain in muscles, especially right
lower extremity. Temperature during a first
day was 39,0C. The hallucinations have
appeared. Was admitted in a hospital with a
passing auto. All surrounding people were
healthy. As a part of his work, majority of
time he spent on a nature - in a wood, in
steppe. Was accurately vaccinated.

109

Objective examination: pulse - 140 1/min.,


weak filling. BP - 80/40 mm. hg. Cardiac
sounds dull, RR - 26 in min.. Vesicular
respiration above a lungs. Tongue was
coated with fur. The abdomen soft, takes
part in an act of respiration, does not react
to palpation. The right leg was half bend. In
inguinal area the formation with indistinct
contours 15 25 sm. was visually defined.
Skin above it was blue - crimson color.
During palpation of this conglomerate the
patient reacts with a groan, tries to repel a
hand of the doctor.
d What is your diagnosis?
e Make your tactics of treatment?

f
g

What is your diagnosis?


What is your therapy?

CASE 26
Patient S, 35 years. Was admitted in clinic in
a severe condition, the unconsciousness,
periodically clonic cramps. In previous was
on business trip in Guinea. Come back to
Ukraine one week ago and fell ill at once.
All days before entering temperature kept
high (39,0-40,0C) and only in mornings
sometimes normal. Sharp headache, chill,
feeling of fever troubled. Patient was taking
Analgin, but without effect: the headache
accrued, sleepiness appeared, and then
consciousness was lost, cramps, exaltation
appeared.
Anamnesis epidemica. In Guinea one month
ago had a malaria, probably one month ago.
OE: skin and visual mucous slightly
subicterous, respiration of Shain - Stoks.
Tachycardia, pulse - 130 1/min., weak filling
and strain, BP - 90/40 mm. hg.. On
auscultation of lungs - variegrated wet
crepitations. The lien was palpated not
legiblly. Pupillary tests sluggish, miosis was
observed. Hyperreflexia of tendinal reflexes,
mild rigidity of a neck muscles.
a) What is your diagnosis?
b) What is the form of disease?
c) What is your therapeutic tactics?

CASE 25
Patient G. B., 21 years, was taken in hospital
on 7-th day of disease with complaints on
chill, high temperature, headache, common
weakness, sore throat. Onset of the disease
was acute from rising temperature up to
38C and pain in a throat.
At home was treated with Tetracyclin,
Aspirin, but the condition without
improvement. No experience of previous
diseases.
Anamnesis epidemica: quiet.
On examination - state was moderate.
Patient was sluggish. Skin was pale, without
rush. Mucous of a throat is hyperemic,
tonsils enlarged, crumbly with films of
green-yellow color. Submandibular and back
of neck lymph nodes were enlarged, size
about a string bean, slightly painful during
palpation. Pulse was 108-118 1/min.,
rhythmic. The cardiac sounds are dull,
tachycardia. In lungs harsh breathing on
auscultation. Tongue was dryish, coated with
white-grey fur. Abdomen was soft, a little
painful during palpation in right
hypochondrium. Liver at a level of a rib arc.
Stool, urination were normal.
The clinical analysis of blood: Eryth. 4
1012/l, Leuk. -10 109 l, dr/st.- 8 %,
cegments 52%, lymphs. - 31 %, monoc - 6
%, plasm. cells 2%. ESR -27 mm/h.
Hemogram in 4 days: Leukoc. - 109 l,
eosynoph. 0%, dr./st. - 8%, segment. 47%, lymphs. - 38%, monoc. - 9%, ESR - 32
mm/hr.. Lymphocytes with wide protoplasm.
Pain in throat decreased in 5 days after
admission have disappeared, temperature
normalized in 15 days. To the moment of
discharge 29th day of disease - the patient
complained on fatigability, giddiness. In a
haemogram - limphomonocitosis with wide
protoplasm lymphocytes.

CASE 27
Patient I. I., 26 years, was admitted in
hospital in a severe condition with
complaints on weakness, difficulty of
swallowing a saliva and water, anxiety,
temperature 38.8C.
Anamnesis: Fell ill about 3 days before,
when weakness, temperature 38C, then
difficulty of swallowing of a saliva, and then
water, hypersalivation has appeared. During
first 2 days not appealed to doctor. On 3-rd
day of disease the state has worsened
sharply: appearance of water, driving of air
have provoked spasts of pharynx, larynx.
Anamnesis of life: without singularities.
Anamnesis epidimica: During last 4 months
before admission in hospital, during hunting
was bitten by the fox for the right hand.
Didnt received any vaccination.
Objective examination: Condition of patient
was severe, temperature 39.8C, hyperemia
of a face. Skin was wet. In a reception ward
the attack was developed, that was
accompanied by motive exaltation, crick of

110

muscles of a pharynx and larynx, abundant


hypersalivation, disorder of respiration. With
cry throws from itself a cup of water, throws
back head and trunk. The face was
distorted, cyanotic, expresses horror, the
pupils dilated, view stuck in one point. The
attack last some seconds. Then the attack
has repeated as a result of a loud sound and
switched on light. Lethal outcome had
occurred during the fit.
a) What is your diagnosis?

111

CLINICAL TASKS

OE: moderate jaundice, temperature


is 39C. Tachicardia up to 122 /min.
Liver sizes by Kurlov: 11108 sm.,
edge painless. Spleen enlarged on 3
sm. below left costal arch. Anuria
about 10 hours.
3. Chose symptoms, typical for tumor of
head of pancreas:
a Gradual onset from nausea, gradual
loss of appetite, increasing itch of
skin. OE: jaundice of skin, with
scratches traces. Skin turgor is
reduced. Pulse is 86 /min.; liver sizes
by Kurlov: 13128 sm, in gall bladder projection elastic painless
formation 33 is palpated. Spleen is
not enlarged.
b Acute beginning from nausea, pain in
a right part of abdomen, fever up to
38C. OE: moderate jaundice;
tachicardia up to 104 /min. Liver
sizes by Kurlov: 12127 sm.,
palpation of gall bladder is painful.
Spleen is not enlarged.
c Beginning gradual from nausea, pain
in joints, itch of skin, and loss of
appetite. OE: skin is icteric, turgor is
normal. Pulse is 68 /min. Liver sizes
by Kurlov: 13128 sm., edge of lever
sensitive. Spleen is not enlarged.
d Acute onset with chill, fever up to
38,8C, Nausea, dizziness, weakness.
OE: moderate jaundice, temperature
is 39C. Tachicardia up to 122 /min.
Liver sizes by Kurlov: 11108 sm.,
edge painless. Spleen enlarged on 3
sm. below left costal arch. Anuria
about 10 hours.
4. What pathogenetic mechanisms of
diarrhea are typical for cholera:
a Dehydratation, hypovolemia, arterial
pressure decreasing, reduction of
renal filtration;
b Toxinemia, disturbance of peripheral
microcirculation;
c Dehydratation, metabolic acidosis,
hypokaliumemia;
d Increase of water and electrolytes
secretion as a result of activation of
adenilatcyclaza and accumulation of
cAMP.
5. What basic clinical symptoms of colitic
form of shigellosis:
a Gradual onset, absence of
intoxication, moderate spasmatic
pains in lower part of abdomen,
alternation of constipation and
diarrhea, sometimes slime in faeces;

1. Chose symptoms, typical for icteric form


of viral hepatitis B:
a Gradual onset from nausea, gradual
loss of appetite, increasing itch of
skin. OE: jaundice of skin, with
scratches traces. Skin turgor is
reduced. Pulse is 86 /min.; liver sizes
by Kurlov: 13128 sm, in gall bladder projection elastic painless
formation 33 is palpated. Spleen is
not enlarged.
b Acute beginning from nausea, pain in
a right part of abdomen, fever up to
38C. OE: moderate jaundice;
tachicardia up to 104 /min. Liver
sizes by Kurlov: 12127 sm.,
palpation of gall bladder is painful.
Spleen is not enlarged.
c Beginning gradual from nausea, pain
in joints, itch of skin, and loss of
appetite. OE: skin is icteric, turgor is
normal. Pulse is 68 /min.. Liver sizes
by Kurlov: 13128 sm., edge of lever
sensitive. Spleen is not enlarged.
d Acute onset with chill, fever up to
38,8C, Nausea, dizziness, weakness.
OE: moderate jaundice, temperature
is 39C. Tachicardia up to 122 /min.
Liver sizes by Kurlov: 11108 sm.,
edge painless. Spleen enlarged on 3
sm. below left costal arch. Anuria
about 10 hours.
2. Chose symptoms, typical for cholelitiasis:
d Gradual onset from nausea, gradual
loss of appetite, increasing itch of
skin. OE: jaundice of skin, with
scratches traces. Skin turgor is
reduced. Pulse is 86 /min.; liver sizes
by Kurlov: 13128 sm, in gall bladder projection elastic painless
formation 33 is palpated. Spleen is
not enlarged.
e Acute beginning from nausea, pain in
a right part of abdomen, fever up to
38C. OE: moderate jaundice;
tachicardia up to 104 /min. Liver
sizes by Kurlov: 12127 sm.,
palpation of gall bladder is painful.
Spleen is not enlarged.
f Beginning gradual from nausea, pain
in joints, itch of skin, and loss of
appetite. OE: skin is icteric, turgor is
normal. Pulse is 68 /min.. Liver sizes
by Kurlov: 13128 sm., edge of lever
sensitive. Spleen is not enlarged.
g Acute onset with chill, fever up to
38,8C, Nausea, dizziness, weakness.

112

Acute onset with sharp pain in


abdomen, symptoms of intoxication
on the beginning are absent; stool is
bloody, frequently without faeces;
c Acute onset, moderate intoxication,
diffuse pain in abdomen, vomiting
from onset, then liquid stool with
admixture of slime (and sometimes
of blood);
d Acute onset, moderate intoxication,
spasmatic pain in left hypogastria,
poor stool with slime and blood.
6. What basic clinical symptoms of cancer of
rectum:
a Gradual onset, absence of
intoxication, moderate spasmatic
pains in lower part of abdomen,
alternation of constipation and
diarrhea, sometimes slime in faeces;
b Acute onset with sharp pain in
abdomen, symptoms of intoxication
on the beginning are absent; stool is
bloody, frequently without faeces;
c Acute onset, moderate intoxication,
diffuse pain in abdomen, vomiting
from onset, then liquid stool with
admixture of slime (and sometimes
of blood);
d Acute onset, moderate intoxication,
spasmatic pain in left hypogastria,
poor stool with slime and blood.
7. What basic clinical symptoms of
thrombosis of mesenteric vessels:
a Gradual onset, absence of
intoxication, moderate spasmatic
pains in lower part of abdomen,
alternation of constipation and
diarrhea, sometimes slime in faeces;
b Acute onset with sharp pain in
abdomen, symptoms of intoxication
on the beginning are absent; stool is
bloody, frequently without faeces;
c Acute onset, moderate intoxication,
diffuse pain in abdomen, vomiting
from onset, then liquid stool with
admixture of slime (and sometimes
of blood);
d Acute onset, moderate intoxication,
spasmatic pain in left hypogastria,
poor stool with slime and blood.
8. What are basic clinical symptoms of
climax period of malaria vivax:
e Constant remitting fever, chills,
significant sweating,
polylymphadenopathy, muscular
pain, spleen and lever enlargement;
f Constant type of fever, weakness,
adynamia, pale skin, poor roseolar

rush on lateral surfaces of trunk,


spleen and lever enlargement;
g Fever of remitting type, chills,
significant weakness, petechial
bleedings on conjunctiva, sometimes
liver enlargement;
h Fever of intermitting type, on climax
of fever fit feeling of heat with
subsequent plentiful sweating,
spleen and lever enlargement.
9. What are basic clinical symptoms of
climax period of acute brucellosis:
a Constant remitting fever, chills,
significant sweating,
polylymphadenopathy, muscular
pain, spleen and lever enlargement;
b Constant type of fever, weakness,
adynamia, pale skin, poor roseolar
rush on lateral surfaces of trunk,
spleen and lever enlargement;
c Fever of remitting type, chills,
significant weakness, petechial
bleedings on conjunctiva, sometimes
liver enlargement;
d Fever of intermitting type, on climax
of fever fit feeling of heat with
subsequent plentiful sweating,
spleen and lever enlargement.
10. What are basic clinical symptoms of
climax period of staphylococcal sepsis:
a Constant remitting fever, chills,
significant sweating,
polylymphadenopathy, muscular
pain, spleen and lever enlargement;
b Constant type of fever, weakness,
adynamia, pale skin, poor roseolar
rush on lateral surfaces of trunk,
spleen and lever enlargement;
c Fever of remitting type, chills,
significant weakness, petechial
bleedings on conjunctiva, sometimes
liver enlargement;
d Fever of intermitting type, on climax
of fever fit feeling of heat with
subsequent plentiful sweating,
spleen and lever enlargement.
11. What symptoms are most typical for
staphylococcal food poisoning:
a Onset with chill, vomiting, short term increase of temperature; diffuse
abdominal pain, liquid stool with
greenish shade is often (15-20 of
time per day);
b Acute beginning with chill, repeated
vomiting, sharp pains in epigastria,
collapse is quite often, liquid stool 45 times per day, fast convalescence;
c Acute onset with liquid watery stools,
subsequent vomiting connection;

113

oliguria, normal body temperature,


absence of abdominal pain;
d Gradual onset with fever, increasing
weakness, pale skin, duration of
fever about 2 weeks, from the end of
first week - liquid stool 2-4 times per
day;
12. What symptoms are most typical for
gastrointestinal salmonellosis:
a Onset with chill, vomiting, short term increase of temperature; diffuse
abdominal pain, liquid stool with
greenish shade is often (15-20 of
time per day);
b Acute beginning with chill, repeated
vomiting, sharp pains in epigastria,
collapse is quite often, liquid stool 45 times per day, fast convalescence;
c Acute onset with liquid watery stools,
subsequent vomiting connection;
oliguria, normal body temperature,
absence of abdominal pain;
d Gradual onset with fever, increasing
weakness, pale skin, duration of
fever about 2 weeks, from the end of
first week - liquid stool 2-4 times per
day;
13. What symptoms are most typical for
moderate form of cholera:
a Onset with chill, vomiting, short term increase of temperature; diffuse
abdominal pain, liquid stool with
greenish shade is often (15-20 of
time per day);
b Acute beginning with chill, repeated
vomiting, sharp pains in epigastria,
collapse is quite often, liquid stool 45 times per day, fast convalescence;
c Acute onset with liquid watery stools,
subsequent vomiting connection;
oliguria, normal body temperature,
absence of abdominal pain;
d Gradual onset with fever, increasing
weakness, pale skin, duration of
fever about 2 weeks, from the end of
first week - liquid stool 2-4 times per
day;
14. What symptoms are typical for initial
period of typhoid fever;
d Acute onset with fast increase of
temperature within 1-2 days up to
39-40C; a persistent headache,
sleeplessness, euphoria. Face is
hyperemic, sclera are injected,
spleen is increased since 3-4 days
from onset;
e Gradual onset with slow (4-5 days)
increase of body temperature,
general weakness, slackness, pale

skin; tongue is thickened, with teeth


prints, bradicardia, spleen is enlarged
since 6-7 days from onset;
f Acute onset, within several hours
body temperature reaches 38-39C,
accompanied with chill, headache in
forehead and temple area, pain in
eyes, nasal obstruction, dry cough,
tracheal pain. Spleen is not enlarged;
g Acute onset with chill, fast increase
of temperature up to 39-40C;
weakness, sweating, muscular pain,
excitation, headache, dry cough,
hyperemia of face, injection of sclera,
sometimes palatal enantema,
relative bradicardia, spleen
enlargement is quite often.
15. What symptoms are typical for initial
period of louse - born typhus;
a Acute onset with fast increase of
temperature within 1-2 days up to
39-40C; a persistent headache,
sleeplessness, euphoria. Face is
hyperemic, sclera are injected,
spleen is increased since 3-4 days
from onset;
b Gradual onset with slow (4-5 days)
increase of body temperature,
general weakness, slackness, pale
skin; tongue is thickened, with teeth
prints, bradicardia, spleen is enlarged
since 6-7 days from onset;
c Acute onset, within several hours
body temperature reaches 38-39C,
accompanied with chill, headache in
forehead and temple area, pain in
eyes, nasal obstruction, dry cough,
tracheal pain. Spleen is not enlarged;
d Acute onset with chill, fast increase
of temperature up to 39-40C;
weakness, sweating, muscular pain,
excitation, headache, dry cough,
hyperemia of face, injection of sclera,
sometimes palatal enantema,
relative bradicardia, spleen
enlargement is quite often.
16. What symptoms are typical for initial
period of Q fever;
a Acute onset with fast increase of
temperature within 1-2 days up to
39-40C; a persistent headache,
sleeplessness, euphoria. Face is
hyperemic, sclera are injected,
spleen is increased since 3-4 days
from onset;
b Gradual onset with slow (4-5 days)
increase of body temperature,
general weakness, slackness, pale
skin; tongue is thickened, with teeth

114

prints, bradicardia, spleen is enlarged


since 6-7 days from onset;
c Acute onset, within several hours
body temperature reaches 38-39C,
accompanied with chill, headache in
forehead and temple area, pain in
eyes, nasal obstruction, dry cough,
tracheal pain. Spleen is not enlarged;
d Acute onset with chill, fast increase
of temperature up to 39-40C;
weakness, sweating, muscular pain,
excitation, headache, dry cough,
hyperemia of face, injection of sclera,
sometimes palatal enantema,
relative bradicardia, spleen
enlargement is quite often.
17. Choose symptoms, typical for
diphtheria:
a Acute onset, general headache, pain
in throat, fever 38 -40C,
lymphadenopathy, mucous of throat
is hyperemic, white - yellow purulent
removed films in lacunas of tonsils;
spleen enlargement;
b Acute beginning, strong headache,
pain in throat as swallowing, fever
39-40C, mucous of throat is
hyperemic, grey - yellow purulent
removed films in lacunas of tonsils,
jugular lymph nods are increased and
sharply painful;
c Acute beginning, moderate pain in a
throat, fever up to 38C, mucous of
throat is hyperemic, white colored,
hardly removed films, slightly
increased and moderately painful
jugularis lymph nodes;
d Acute onset, fever up to 38 - 40C,
mucous of throat is moderate
hyperemic, on left tonsil crater - like
ulcer with purulent content, lymph
nodes are not enlarged.
18. Choose symptoms, typical for infectious
mononucleosis:
a Acute onset, general headache, pain
in throat, fever 38 -40C,
lymphadenopathy, mucous of throat
is hyperemic, white - yellow purulent
removed films in lacunas of tonsils;
spleen enlargement;
b Acute beginning, strong headache,
pain in throat as swallowing, fever
39-40C, mucous of throat is
hyperemic, grey - yellow purulent
removed films in lacunas of tonsils,
jugular lymph nods are increased and
sharply painful;

Acute beginning, moderate pain in a


throat, fever up to 38C, mucous of
throat is hyperemic, white colored,
hardly removed films, slightly
increased and moderately painful
jugularis lymph nodes;
d Acute onset, fever up to 38 - 40C,
mucous of throat is moderate
hyperemic, on left tonsil crater - like
ulcer with purulent content, lymph
nodes are not enlarged.
19. Choose symptoms, typical for acute
tonsillitis:
a Acute onset, general headache, pain
in throat, fever 38 -40C,
lymphadenopathy, mucous of throat
is hyperemic, white - yellow purulent
removed films in lacunas of tonsils;
spleen enlargement;
b Acute beginning, strong headache,
pain in throat as swallowing, fever
39-40C, mucous of throat is
hyperemic, grey - yellow purulent
removed films in lacunas of tonsils,
jugular lymph nods are increased and
sharply painful;
c Acute beginning, moderate pain in a
throat, fever up to 38C, mucous of
throat is hyperemic, white colored,
hardly removed films, slightly
increased and moderately painful
jugularis lymph nodes;
d Acute onset, fever up to 38 - 40C,
mucous of throat is moderate
hyperemic, on left tonsil crater - like
ulcer with purulent content, lymph
nodes are not enlarged.
20. What symptoms are typical for initial
period of typhoid fever:
a
Acute beginning with fast increase of
body temperature within 1-2 days up to 3940C, persistent headache, sleeplessness,
anxiety, euphoria. Face is hyperemic, sclera
are injected, spleen is enlarged since 3-4
day of disease;
b
Gradual onset with slow (4-5 days)
increase of temperature, general adinamia,
slackness, pale skin, tongue is tickened, with
prints of teeth, relative bradicardia, spleen is
enlarged since 6-7 day of disease;
c
Acute beginning, within several hours
temperature reaches 38-39C, accompanied
with chill, headache in forehead and temple
area, pain in eyes, stiffness of a nose, dry
cough, morbidity on trachea projection.
Spleen is not enlarged.
d
Acute beginning with chill, fast
increase of temperature up to 39-40C,

115

weakness, sweating, muscular pain,


excitation, headache, dry cough, hyperemia
of face, injection of sclera, sometimes
palatal enantema, relative bradicardia,
spleen is enlarged quite often.
21. What symptoms are typical for initial
period of louse - borne typhus:
a Acute beginning with fast increase of body
temperature within 1-2 days up to 39-40C,
persistent headache, sleeplessness, anxiety,
euphoria. Face is hyperemic, sclera are
injected, spleen is enlarged since 3-4 day of
disease;
b Gradual onset with slow (4-5 days) increase
of temperature, general adinamia,
slackness, pale skin, tongue is tickened, with
prints of teeth, relative bradicardia, spleen is
enlarged since 6-7 day of disease;
c Acute beginning, within several hours
temperature reaches 38-39C, accompanied
with chill, headache in forehead and temple
area, pain in eyes, stiffness of a nose, dry
cough, morbidity on trachea projection.
Spleen is not enlarged.
d Acute beginning with chill, fast increase of
temperature up to 39-40C, weakness,
sweating, muscular pain, excitation,
headache, dry cough, hyperemia of face,
injection of sclera, sometimes palatal
enantema, relative bradicardia, spleen is
enlarged quite often.
22. What symptoms are typical for initial
period of Q fever:
a Acute beginning with fast increase of body
temperature within 1-2 days up to 39-40C,
persistent headache, sleeplessness, anxiety,
euphoria. Face is hyperemic, sclera are
injected, spleen is enlarged since 3-4 day of
disease;
b Gradual onset with slow (4-5 days) increase
of temperature, general adinamia,
slackness, pale skin, tongue is tickened, with
prints of teeth, relative bradicardia, spleen is
enlarged since 6-7 day of disease;
c Acute beginning, within several hours
temperature reaches 38-39C, accompanied
with chill, headache in forehead and temple
area, pain in eyes, stiffness of a nose, dry
cough, morbidity on trachea projection.
Spleen is not enlarged.
d Acute beginning with chill, fast increase of
temperature up to 39-40C, weakness,
sweating, muscular pain, excitation,
headache, dry cough, hyperemia of face,
injection of sclera, sometimes palatal
enantema, relative bradicardia, spleen is
enlarged quite often.
23. What symptoms are typical for
gastroenterocolitic form of shigellosis:

a Acute onset, increase of temperature up to


39C, repeated vomiting, diffuse abdominal
pain, plentiful watery stool without slime
and blood, spasm of led muscles, oliguria,
hypotonia;
b Acute onset, increase of temperature up to
38,5-39C, vomiting 3-4 times, spasmatic
abdominal pain in left hypogastrium, often
plentiful liquid stool with admixture of blood
and slime;
c Acute onset, normal body temperature,
vomiting, often plentiful watery stool
without slime and blood, spasm of leg
muscles, expressed dryness mucous,
decrease of skin elasticity, oliguria;
d Acute onset, increase of temperature up to
39C, spasmatic pain in the left half of
abdomen, often liquid poor stool with slime
and blood, tenesms.
24. What symptoms are typical for colitic
form of shigellosis:
a Acute onset, increase of temperature up to
39C, repeated vomiting, diffuse abdominal
pain, plentiful watery stool without slime
and blood, spasm of led muscles, oliguria,
hypotonia;
b Acute onset, increase of temperature up to
38,5-39C, vomiting 3-4 times, spasmatic
abdominal pain in left hypogastrium, often
plentiful liquid stool with admixture of blood
and slime;
c Acute onset, normal body temperature,
vomiting, often plentiful watery stool
without slime and blood, spasm of leg
muscles, expressed dryness mucous,
decrease of skin elasticity, oliguria;
d Acute onset, increase of temperature up to
39C, spasmatic pain in the left half of
abdomen, often liquid poor stool with slime
and blood, tenesms.
25. What symptoms are typical for
gastroenteritic form of salmonellosis:
a Acute onset, increase of temperature up to
39C, repeated vomiting, diffuse abdominal
pain, plentiful watery stool without slime
and blood, spasm of led muscles, oliguria,
hypotonia;
b Acute onset, increase of temperature up to
38,5-39C, vomiting 3-4 times, spasmatic
abdominal pain in left hypogastrium, often
plentiful liquid stool with admixture of blood
and slime;
c Acute onset, normal body temperature,
vomiting, often plentiful watery stool
without slime and blood, spasm of leg
muscles, expressed dryness mucous,
decrease of skin elasticity, oliguria;

116

d Acute onset, increase of temperature up to


39C, spasmatic pain in the left half of
abdomen, often liquid poor stool with slime
and blood, tenesms.
26. What hemodynamics parameters of
hemorrhagic fever with renal syndrome:
e Tachicardia, hypotonia, tendency to
collapse;
f Relative bradicardia, dycrotia of pulse,
hypotonia;
g Significant bradicardia, hypotonia;
h Tachicardia up to 120-160 /min., arrhythmia,
hypotonia.
27. What hemodynamics parameters of
typhoid fever:
e Tachicardia, hypotonia, tendency to
collapse;
f Relative bradicardia, dycrotia of pulse,
hypotonia;
g Significant bradicardia, hypotonia;
h Tachicardia up to 120-160 /min., arrhythmia,
hypotonia.
28. What hemodynamics parameters of
louse - borne typhus:
a Tachicardia, hypotonia, tendency to
collapse;
b Relative bradicardia, dycrotia of pulse,
hypotonia;
c Significant bradicardia, hypotonia;
d Tachicardia up to 120-160 /min., arrhythmia,
hypotonia.
29. What are main symptoms of hepatic
coma as complication of viral hepatitis B:
e Patient is unconsciousness. Temperature
39,8C. Skin pale, damp, jaundice.
Tachicardia up to 148 /min.. Liver sizes by
Kurlov: 12119 sm., spleen enlarged on 4
sm. below edge of costal arch;
f Patient is unconsciousness. Temperature
37,2C. Bright jaundice, individual petechia.
Tachicardia up to 98 /min.. Liver sizes by
Kurlov: 887 sm.. Spleen is not enlarged;
g Patient is unconsciousness. Temperature
36,1C. Skin pale, dry. Tachicardia up to
110 /min.. Liver sizes by Kurlov: 1098 sm.
Spleen is not enlarged;
h Patient is unconsciousness. Temperature
38,5C. Skin of light pink color, plentiful
hemorrhagic rash is presented. Tachicardia
up to 130 /min.. Liver and spleen are not
enlarged.
30. What are main symptoms of
hyperglycemic coma as complication of
sugar diabetes:
a Patient is unconsciousness. Temperature
39,8C. Skin pale, damp, jaundice.
Tachicardia up to 148 /min.. Liver sizes by
Kurlov: 12119 sm., spleen enlarged on 4
sm. below edge of costal arch;

b Patient is unconsciousness. Temperature


37,2C. Bright jaundice, individual petechia.
Tachicardia up to 98 /min.. Liver sizes by
Kurlov: 887 sm.. Spleen is not enlarged;
c Patient is unconsciousness. Temperature
36,1C. Skin pale, dry. Tachicardia up to
110 /min.. Liver sizes by Kurlov: 1098 sm.
Spleen is not enlarged;
d Patient is unconsciousness. Temperature
38,5C. Skin of light pink color, plentiful
hemorrhagic rash is presented. Tachicardia
up to 130 /min.. Liver and spleen are not
enlarged.
31. What are main symptoms of malarial
coma as complication of tropical malaria:
a Patient is unconsciousness. Temperature
39,8C. Skin pale, damp, jaundice.
Tachicardia up to 148 /min.. Liver sizes by
Kurlov: 12119 sm., spleen enlarged on 4
sm. below edge of costal arch;
b Patient is unconsciousness. Temperature
37,2C. Bright jaundice, individual petechia.
Tachicardia up to 98 /min.. Liver sizes by
Kurlov: 887 sm.. Spleen is not enlarged;
c Patient is unconsciousness. Temperature
36,1C. Skin pale, dry. Tachicardia up to
110 /min.. Liver sizes by Kurlov: 1098 sm.
Spleen is not enlarged;
d Patient is unconsciousness. Temperature
38,5C. Skin of light pink color, plentiful
hemorrhagic rash is presented. Tachicardia
up to 130 /min.. Liver and spleen are not
enlarged.
32. What are typical symptoms of initial
period of pseudo-tuberculosis:
e Acute onset, in 2-6 hours body temperature
reaches 38-39C, expressed headache in
forehead and eyes, sore throat, dry cough.
Lymph nodes and spleen are not enlarged.
Urine passage is normal;
f Acute onset with chill, in 12-24 hours
temperature reaches 40C, strong
headache, are expressed muscular pain.
Skin of face and neck hyperemic, scleritis,
petechial rash, liver and spleen enlarged.
Oliguria.
g Acute onset, on 2-3 day of disease
temperature reaches 40C, headache, loss
of appetite, pain in a throat, liquid stool 3-5
times per day. Skin of face, neck, palms and
feet are hyperemic, pale nasal triangle. liver
and spleen enlarged. Urine passage is
normal;
h Acute onset, on 2-3 day of disease body
temperature reaches 39,5-40C, strong
headache, anxiety, quite often excitation,
euphoria; hyperemia of face, sclera injected,

117

spleen enlargement. Urine passage is


normal.
33. What are typical symptoms of initial
period of influenza:
a Acute onset, in 2-6 hours body temperature
reaches 38-39C, expressed headache in
forehead and eyes, sore throat, dry cough.
Lymph nodes and spleen are not enlarged.
Urine passage is normal;
b Acute onset with chill, in 12-24 hours
temperature reaches 40C, strong
headache, are expressed muscular pain.
Skin of face and neck hyperemic, scleritis,
petechial rash, liver and spleen enlarged.
Oliguria.
c Acute onset, on 2-3 day of disease
temperature reaches 40C, headache, loss
of appetite, pain in a throat, liquid stool 3-5
times per day. Skin of face, neck, palms and
feet are hyperemic, pale nasal triangle. liver
and spleen enlarged. Urine passage is
normal;
d Acute onset, on 2-3 day of disease body
temperature reaches 39,5-40C, strong
headache, anxiety, quite often excitation,
euphoria; hyperemia of face, sclera injected,
spleen enlargement. Urine passage is
normal.
34. What are typical symptoms of initial
period of leptospyrosis:
a Acute onset, in 2-6 hours body temperature
reaches 38-39C, expressed headache in
forehead and eyes, sore throat, dry cough.
Lymph nodes and spleen are not enlarged.
Urine passage is normal;
b Acute onset with chill, in 12-24 hours
temperature reaches 40C, strong
headache, are expressed muscular pain.
Skin of face and neck hyperemic, scleritis,
petechial rash, liver and spleen enlarged.
Oliguria.
c Acute onset, on 2-3 day of disease
temperature reaches 40C, headache, loss
of appetite, pain in a throat, liquid stool 3-5
times per day. Skin of face, neck, palms and
feet are hyperemic, pale nasal triangle. liver
and spleen enlarged. Urine passage is
normal;
d Acute onset, on 2-3 day of disease body
temperature reaches 39,5-40C, strong
headache, anxiety, quite often excitation,
euphoria; hyperemia of face, sclera injected,
spleen enlargement. Urine passage is
normal.
35. What symptoms are typical for
meningococcal nasopharingitis:
e Granularity and hyperemia of a back wall of
throat;

f hemprrhagic rash;
g stiffness of neck muscles;
h Plentiful purulent nasal excretion.
36. What symptoms are typical for
meningococcal meningitis:
e Granularity and hyperemia of a back wall of
throat;
f hemprrhagic rash;
g stiffness of neck muscles;
h Plentiful purulent nasal excretion.
37.What symptoms are typical for
meningococcemia:
a Granularity and hyperemia of a back wall of
throat;
b hemprrhagic rash;
c stiffness of neck muscles;
d Plentiful purulent nasal excretion.
38. What symptoms of CNS affection are
typical for louse - borne typhus:
a hyperestesya, nausea and vomiting, quite
often loss of consciousness, headache,
positive symptom of Kernig, stiffness of neck
muscles;
b Euphoria, excitation, anxiety, acoustical and
visual hallucinations, persistent headache,
symptoms of cranial nerves affection;
c adinamia, dormancy, dull headache;
d dizziness, noise in ears, adinamia, headache
in forehead and temple areas.
39. What symptoms of CNS affection are
typical for typhoid fever:
a hyperestesya, nausea and vomiting, quite
often loss of consciousness, headache,
positive symptom of Kernig, stiffness of neck
muscles;
b Euphoria, excitation, anxiety, acoustical and
visual hallucinations, persistent headache,
symptoms of cranial nerves affection;
c adinamia, dormancy, dull headache;
d dizziness, noise in ears, adinamia, headache
in forehead and temple areas.
40. What symptoms of CNS affection are
typical for purulent meningitis:
a hyperestesya, nausea and vomiting, quite
often loss of consciousness, headache,
positive symptom of Kernig, stiffness of neck
muscles;
b Euphoria, excitation, anxiety, acoustical and
visual hallucinations, persistent headache,
symptoms of cranial nerves affection;
c adinamia, dormancy, dull headache;
d dizziness, noise in ears, adinamia, headache
in forehead and temple areas.
41. What rush type and localization is typical
for pseudo-tuberculosis:
a Arises on 2-5 day of disease on top of trunk,
then spreads on extremities and around
joints, to a face, feet, roseolar and papular;

118

b Plentiful, roseolar and petechial, arises on 45 day, mainly on thorax, extremities, not
typical on face, palms and feet;
c Arises on 2-6 day, little - pointed, on intact
skin, located mainly on lateral surfaces of a
trunk, axilar areas and abdomen,
extremities, is concentrated in skin plaites,
face is usually clear; frequently erythema of
palms and feet as "gloves" and "socks";
d Arises on 3-4 day on top of face, then on
trunk, forearms and thighs, then on shins,
spotty papulous, leaves pigmentation and
peeling after recovery;
42. What rush type and localization is typical
for louse - borne typhus:
a Arises on 2-5 day of disease on top of trunk,
then spreads on extremities and around
joints, to a face, feet, roseolar and papular;
b Plentiful, roseolar and petechial, arises on 45 day, mainly on thorax, extremities, not
typical on face, palms and feet;
c Arises on 2-6 day, little - pointed, on intact
skin, located mainly on lateral surfaces of a
trunk, axilar areas and abdomen,
extremities, is concentrated in skin plaites,
face is usually clear; frequently erythema of
palms and feet as "gloves" and "socks";
d Arises on 3-4 day on top of face, then on
trunk, forearms and thighs, then on shins,
spotty papules, leaves pigmentation and
peeling after recovery;
43. What rush type and localization is typical
for tick - borne North Asian rickettsiosis:
a Arises on 2-5 day of disease on top of trunk,
then spreads on extremities and around
joints, to a face, feet, roseolar and papular;
b Plentiful, roseolar and petechial, arises on 45 day, mainly on thorax, extremities, not
typical on face, palms and feet;
c Arises on 2-6 day, little - pointed, on intact
skin, located mainly on lateral surfaces of a
trunk, axial areas and abdomen, extremities,
is concentrated in skin plaits, face is usually
clear; frequently erythema of palms and feet
as "gloves" and "socks";
d Arises on 3-4 day on top of face, then on
trunk, forearms and thighs, then on shins,
spotty papules, leaves pigmentation and
peeling after recovery;
44. What mechanism of rash formation for
meningococcemia:
a Damage of capillaries endothelium by
microorganisms and toxins, circulating in
blood;
b sedimentation of immune complexes in
capillaries of skin;
c Drift of pathogen in lymph vessels of skin
with subsequent development of productive
- inflammatory changes;

d generalized destructive and prolypherative


vasculitis;
45. What mechanism of rash formation for
typhoid fever:
a Damage of capillaries endothelium by
microorganisms and toxins, circulating in
blood;
b sedimentation of immune complexes in
capillaries of skin;
c Drift of pathogen in lymph vessels of skin
with subsequent development of productive
- inflammatory changes;
d generalized destructive and prolypherative
vasculitis;
46. What mechanism of rash formation for
louse - borne typhus.
a Damage of capillaries endothelium by
microorganisms and toxins, circulating in
blood;
b sedimentation of immune complexes in
capillaries of skin;
c Drift of pathogen in lymph vessels of skin
with subsequent development of productive
- inflammatory changes;
d generalized destructive and prolypherative
vasculitis;
47. What abdominal symptoms are typical
for rotaviral infection;
a Spleen is enlarged since 3-4 day of disease,
soft, liver is enlarged; tendency to
constipation, ishuria, oliguria;
b Liver and spleen are enlarged since 6-7 day,
firm; stool liquid 3-4 times per day,
meteorism;
c Liver and spleen are not enlarged, abdomen
is soft, slightly painful around navel, stool is
liquid from 2 up to 10 times per day;
d liver enlarged since 3-4 day of disease,
sensitive on palpation, spleen is enlarged,
meteorism, mesenteric lymph nodes are
palpated, stool 1-2 times a day.
48. What abdominal symptoms are typical
for louse - borne typhus;
a Spleen is enlarged since 3-4 day of disease,
soft, liver is enlarged; tendency to
constipation, ishuria, oliguria;
b Liver and spleen are enlarged since 6-7 day,
firm; stool liquid 3-4 times per day,
meteorism;
c Liver and spleen are not enlarged, abdomen
is soft, slightly painful around navel, stool is
liquid from 2 up to 10 times per day;
d liver enlarged since 3-4 day of disease,
sensitive on palpation, spleen is enlarged,
meteorism, mesenteric lymph nodes are
palpated, stool 1-2 times a day.
49.What abdominal symptoms are typical
for typhoid fever;

119

a Spleen is enlarged since 3-4 day of disease,


soft, liver is enlarged; tendency to
constipation, ishuria, oliguria;
b Liver and spleen are enlarged since 6-7 day,
firm; stool liquid 3-4 times per day,
meteorism;
c Liver and spleen are not enlarged, abdomen
is soft, slightly painful around navel, stool is
liquid from 2 up to 10 times per day;
d liver enlarged since 3-4 day of disease,
sensitive on palpation, spleen is enlarged,
meteorism, mesenteric lymph nodes are
palpated, stool 1-2 times a day.
50. What symptoms are typical for initial
period of leptostyrosis:
a Acute onset with increase of body
temperature to 38C, loss of appetite,
nausea, weakness and general unwell,
sometimes dry cough. Since 2-4 day liver is
enlarged, temperature of body is
normalized;
b Acute onset with increase of body
temperature to 39,5C, muscular and joint
pains; a headache in forehead area, pain in
eyes, dry cough, tracheal pain. Liver is not
enlarged.
c Acute onset with increase of body
temperature to 39C, headache, mialgias
(mainly in leg muscles), weakness, pain in
eyes. Injection of sclera, face is hyperemic,
petechia in axillar areas. The liver is
enlarged, dark urine, oliguria.
d Acute onset with increase of body
temperature to 38C, weakness, running
nose, dry cough, pain in eyes. Conjunctiva
are hyperemic. Neck, axillary lymph nodes
and lever are enlarged. Urine is normal.
51. What symptoms are typical for initial
period of viral hepatitis A:
a Acute onset with increase of body
temperature to 38C, loss of appetite,
nausea, weakness and general unwell,
sometimes dry cough. Since 2-4 day liver is
enlarged, temperature of body is
normalized;
b Acute onset with increase of body
temperature to 39,5C, muscular and joint
pains; a headache in forehead area, pain in
eyes, dry cough, tracheal pain. Liver is not
enlarged.
c Acute onset with increase of body
temperature to 39C, headache, mialgias
(mainly in leg muscles), weakness, pain in
eyes. Injection of sclera, face is hyperemic,
petechia in axillar areas. The liver is
enlarged, dark urine, oliguria.
d Acute onset with increase of body
temperature to 38C, weakness, running

nose, dry cough, pain in eyes. Conjunctiva


are hyperemic. Neck, axillary lymph nodes
and lever are enlarged. Urine is normal.
52. What symptoms are typical for initial
period of adenoviral infection:
a Acute onset with increase of body
temperature to 38C, loss of appetite,
nausea, weakness and general unwell,
sometimes dry cough. Since 2-4 day liver is
enlarged, temperature of body is
normalized;
b Acute onset with increase of body
temperature to 39,5C, muscular and joint
pains; a headache in forehead area, pain in
eyes, dry cough, tracheal pain. Liver is not
enlarged.
c Acute onset with increase of body
temperature to 39C, headache, mialgias
(mainly in leg muscles), weakness, pain in
eyes. Injection of sclera, face is hyperemic,
petechia in axillar areas. The liver is
enlarged, dark urine, oliguria.
d Acute onset with increase of body
temperature to 38C, weakness, running
nose, dry cough, pain in eyes. Conjunctiva
are hyperemic. Neck, axillary lymph nodes
and lever are enlarged. Urine is normal.
53. What drug for specific therapy of
shigellosis:
a erythromycin;
b cyprofloxacin;
c penicillin;
d metronidazol (trychopol);
e chlorochin;
f acyclovir;
54. What drug for specific therapy of
intestinal amebiasis:
a erythromycin;
b cyprofloxacin;
c penicillin;
d metronidazol (trychopol);
e chlorochin;
f acyclovir;
55. What drug for specific therapy of
malaria:
a erythromycin;
b cyprofloxacin;
c penicillin;
d metronidazol (trychopol);
e chlorochin;
f acyclovir;
56. What symptoms are typical for initial
period of pseudo-tuberculosis:
a Acute onset with fast increase of
temperature of a body up to 39-40, strong
headache, euphoria, sleeplessness; injection
of sclera, insignificant hyperemia of the
palatum and back wall of throat, lymph

120

nodes are not enlarged, spleen is enlarged


since 3-4 day of disease;
b Acute onset with increase of temperature to
38,5-40C, edema and pain in ankle and
knee joints, pain in a throat, increase
regional lymph nodes, bright hyperemia of
the palatine with precise border, fur
coated tongue. Spleen and lever
enlargement.
c Gradual onset, from 3d day increase of
temperature up to 38C, running nose, pain
in a throat. Tonsils are enlarged, hyperemic,
without fur. Neck, axillary lymph nodes are
increased; conjunctivitis, joints are not
changed, sometimes spleen enlargement;
dry cough is quite often;
d Acute onset, after some hours temperature
reaches 40C, headache in frontal area, pain
in eyes, tracheal pain. Palatinal hyperemia.
Lymph nodes and spleen are not enlarged,
joints are normal.
57. What symptoms are typical for initial
period of louse - borne typhus:
Acute onset with fast increase of
temperature of a body up to 39-40, strong
headache, euphoria, sleeplessness; injection
of sclera, insignificant hyperemia of the
palatum and back wall of throat, lymph
nodes are not enlarged, spleen is enlarged
since 3-4 day of disease;
Acute onset with increase of temperature to
38,5-40C, edema and pain in ankle and
knee joints, pain in a throat, increase
regional lymph nodes, bright hyperemia of
the palatine with precise border, fur
coated tongue. Spleen and lever
enlargement.
Gradual onset, from 3d day increase of
temperature up to 38C, running nose, pain
in a throat. Tonsils are enlarged, hyperemic,
without fur. Neck, axillary lymph nodes are
increased; conjunctivitis, joints are not
changed, sometimes spleen enlargement;
dry cough is quite often;
Acute onset, after some hours temperature
reaches 40C, headache in frontal area, pain
in eyes, tracheal pain. Palatinal hyperemia.
Lymph nodes and spleen are not enlarged,
joints are normal.
58. What symptoms are typical for initial
period of adenoviral infection:
a Acute onset with fast increase of
temperature of a body up to 39-40,
strong headache, euphoria,
sleeplessness; injection of sclera,
insignificant hyperemia of the
palatum and back wall of throat,
lymph nodes are not enlarged,

121

spleen is enlarged since 3-4 day of


disease;
Acute onset with increase of
temperature to 38,5-40C, edema
and pain in ankle and knee joints,
pain in a throat, increase regional
lymph nodes, bright hyperemia of
the palatine with precise border, fur
coated tongue. Spleen and lever
enlargement.
Gradual onset, from 3d day increase
of temperature up to 38C, running
nose, pain in a throat. Tonsils are
enlarged, hyperemic, without fur.
Neck, axillary lymph nodes are
increased; conjunctivitis, joints are
not changed, sometimes spleen
enlargement; dry cough is quite
often;
Acute onset, after some hours
temperature reaches 40C, headache
in frontal area, pain in eyes, tracheal
pain. Palatinal hyperemia. Lymph
nodes and spleen are not enlarged,
joints are normal.

SKIN CHANGES IN SOME INFECTIOUS


DISEASES

Affected places of skin are sharply


outlined, jelly - like fluctuation of skin
during shaking. On center of edema
painless black ulcer 34 sm. with
yellow purulent edge. In outer zone
of hyperemia - vesicula, filled by dark
red contents.
b Skin is hyperemic with cyanotic
shade, edema is moderate. On
centre of edema black ulcer, 11 sm.
without purulent content. Edema and
ulcer are sharply painful.
c Moderate edema of skin, in centre of
edema zone is crater - like ulcer
under grey crusta, pus is excreted
during pressing. Highly expressed
infiltration,
lymphadenit
and
lymphangit are presented. Touching
to edges of ulcer is painful.
d Skin is markedly hyperemic, shining,
form is smoothed, a zone of affection
with precise festooned edges and a
platen on end. On center affection
hyperemia is less expressed, than on
edges.
62. What character and localization of rush
is typical for pseudo-tuberculosis:
g Occurs on 4-5 days of disease on
lateral surfaces of trunk, lateral
surfaces of shoulders and forearm
(intact face, palms, plants), rash in
roseolar and petechial;
h Rash is plentiful, roseola and papula,
sometimes eritematous (face, neck),
arises on 2-3 days of disease,
condensation in skin of joints, palms,
feet is observed, on palms and feet
quite often are eritema such "gloves"
and "socks";
i
Single elements of roseolar rash arise
on 8-10 day of disease, are located
on forward surface of abdomen and
thorax, some elements are roseola
and papula.
63. What character and localization of rush
is typical for typhoid fever:
a Occurs on 4-5 days of disease on
lateral surfaces of trunk, lateral
surfaces of shoulders and forearm
(intact face, palms, plants), rash in
roseolar and petechial;
b Rash is plentiful, roseola and papula,
sometimes eritematous (face, neck),
arises on 2-3 days of disease,
condensation in skin of joints, palms,
feet is observed, on palms and feet
quite often are eritema such "gloves"
and "socks";

59. What skin phenomena are typical for


skin form of plague:
e Affected places of skin are sharply
outlined, jelly - like fluctuation of skin
during shaking. On center of edema
painless black ulcer 34 sm. with
yellow purulent edge. In outer zone
of hyperemia - vesicula, filled by dark
red contents.
f Skin is hyperemic with cyanotic
shade, edema is moderate. On
centre of edema black ulcer, 11 sm.
without purulent content. Edema and
ulcer are sharply painful.
g Moderate edema of skin, in centre of
edema zone is crater - like ulcer
under grey crusta, pus is excreted
during pressing. Highly expressed
infiltration, lymphadenit and
lymphangit are presented. Touching
to edges of ulcer is painful.
h Skin is markedly hyperemic, shining,
form is smoothed, a zone of affection
with precise festooned edges and a
platen on end. On center affection
hyperemia is less expressed, than on
edges.
60. What skin phenomena are typical for
staphylococcal infection, carbuncle:
a Affected places of skin are sharply
outlined, jelly - like fluctuation of skin
during shaking. On center of edema
painless black ulcer 34 sm. with
yellow purulent edge. In outer zone of
hyperemia - vesicula, filled by dark red
contents.
b Skin is hyperemic with cyanotic shade,
edema is moderate. On centre of
edema black ulcer, 11 sm. without
purulent content. Edema and ulcer are
sharply painful.
c Moderate edema of skin, in centre of
edema zone is crater - like ulcer under
grey crusta, pus is excreted during
pressing. Highly expressed infiltration,
lymphadenit and lymphangit are
presented. Touching to edges of ulcer
is painful.
d Skin is markedly hyperemic, shining,
form is smoothed, a zone of affection
with precise festooned edges and a
platen on end. On center affection
hyperemia is less expressed, than on
edges.
61. What skin phenomena are typical for
skin form of anthrax:

122

Single elements of roseolar rash arise


on 8-10 day of disease, are located
on forward surface of abdomen and
thorax, some elements are roseola
and papula.
64. What character and localization of rush
is typical for louse borne typhus:
f Occurs on 4-5 days of disease on
lateral surfaces of trunk, lateral
surfaces of shoulders and forearm
(intact face, palms, plants), rash in
roseolar and petechial;
g Rash is plentiful, roseola and papula,
sometimes eritematous (face, neck),
arises on 2-3 days of disease,
condensation in skin of joints, palms,
feet is observed, on palms and feet
quite often are eritema such "gloves"
and "socks";
h Single elements of roseolar rash arise
on 8-10 day of disease, are located
on forward surface of abdomen and
thorax, some elements are roseola
and papula.

123

LABORATORY METHOD OF
DIAGNOSTICS

d Recommend assay of enzyme activity


indicating cytolysis in serum;
e Detect immunoglobulin E concentration in
serum of blood.
71. What tactic of physician in case of
negative result of IEA in patient with AIDS,
asymptomatic on examination:
a Perform clinical analysis of blood;
b Prove HIV infection diagnosis;
c *Repeat serum IEA, and in case of positive
result to perform immune blotting;
d Recommend repeated IEA assay in one ear;
e Prescribe antiretroviral therapy.
72. Which methods from mentioned below
are informative for sytomegalovirus
infection:
a Blood smear and tick drop microscopy;
b Clinical blood sample;
c *Immunoenzyme assay;
d Immune blotting;
e Latex agglutination.
73. What laboratory methods should be
applied for proving of etiology of herpesviral
infections, phase of infectious process and
respectively tactics of treatment:
a Immune blotting;
b Estimation of replicative capability of virus;
c Clinical blood analysis;
d Immunoenzyme assay;
e Reaction of Right;
f *One index is not enough, should be applied
complex, including assay of antibodies Ig M,
G, their dynamic titer, and (if necessary) in
CSF, PCR.
74. Method of specific diagnostics of
brucellosis:
a bacteriological;
b bacterioscopy;
c allergological;
d *serological;
e biological.
75. What material for specific diagnostics of
pseudotuberculosis:
a blood;
b faeces;
c sputum;
d urine;
e all mentioned.
76 What are main data of CSF in
meningococcal meningitis:
a CSF pressure elevation;
b High neutrophil pleositosis;
c Protein cell dissociation;
d Normal glucose amount;
e CSF is dimmed.
77. What laboratory methods are necessary
for proving of latent or chronic
toxoplasmosis:
a X ray examination of brain and muscles;

65. What methods are used for diagnostics


of HIV/AIDS:
a Blood smear and tick drop microscopy;
b Clinical blood analysis;
c *Immune enzyme assay;
d *Immune blotting;
e Latex agglutination.
66. What tests are used for cytomegalovirus
infection diagnostics:
a Reaction of compliment fixation;
b Absolute neutrophil count of Romanovsky
Gimza stained blood smear;
c Immune enzyme detection of specific IgM
and IgG in serum of blood;
d Cytological assay of urine sediment;
e Immunological assay (estimation of absolute
and relative amount of lymphocytes
subpopulation, immunoglobulins of M,A in
serum of blood).
67. What methods are used for detection of
replication activity of opportunistic
infections pathogens and HIV on different
stages of infectious process:
a Reaction of indirect agglutination;
b *PCR;
c Vidal reaction;
d Immunofluorescent microscopy with
monoclonal antibodies;
e Avidity of antibodies detection.
68. What defect of immunoenzyme method
in HIV/AIDS diagnostics:
a Low sensitivity;
b Low specificity;
c High cost;
d *Hypersensitivity;
e Long time performance;
69. How to treat positive result IEA in case
of absence of clinical data of HIV/AIDS on
examination:
a patient has protective antibodies level to
HIV;
b *patient may be in incubation period, acute
stage was asymtomatis or wasnt
diagnosed;
c patient in incubation period;
d patient in incubation period;
e This is acute asymptomatic stage of
infection.
70. What tactic of physician in case of
negative result of IEA in patient with AIDS on
stage of persistent generalized
lymphadenopathy:
a Clinical analysis of blood performance;
b *Repeat assay with IEA test system of
different firm;
c Exclude diagnosis of HIV infection;

124

b Parasytological examination of faeces;


c Parasytological examination of blood;
d ECG;
e Aye bottom examination;
78. What laboratory data are proving
diagnosis of viral hepatitis A:
a HBs AG in blood serum;
b High activeness ALT in blood serum;
c High level of cholesterol in blood serum;
d Anti HAV IgM in blood serum;
e Anti HCV IgM in blood serum.
79. Methods of specific laboratory
diagnostics of food poisonings:
a bacteriological of vomiting masses;
b bacteriological of faeces;
c bacteriological of blood;
d serological of blood serum.
80. What changes in blood count are typical
for shigellosis:
a leukopenia;
b moderate neutrophil leukocytosis;
c relative lymph and monocytosis;
d drum stick shift to left.
81. What methods for specific diagnostics of
ARVI:
a rinocytoscopia;
b virusological;
c serological;
d immunofluorescent.
82. What methods for specific diagnostics of
toxoplasmosis:
a Reaction of compliment fixation;
b Skin allergic test;
c Parasytoscopy of faeces;
d RSF (Sabin Feldman reaction);
e RIF (reaction of immune fluorescence).
83. What laboratory data are proving
diagnosis of viral hepatitis B:
a Anti HAV IgM in blood serum;
b HBs AG in blood serum;
c High activeness ALT in blood serum;
d High level of alkaline phosfatace in blood
serum;
e Anti HBc IgM in blood serum.
84. What methods of specific laboratory
diagnostics of shigellosis:
a bacteriological of faeces;
b bacteriological of blood;
c reaction of indirect hemmaglutination (RIH);
d reaction of immune fluorescence;
e immunoenzyme assay.
85. What methods of specific laboratory
diagnostics of cholera:
a bacteriological;
b bacteriological of blood;
c serological of blood;
d bacteriological of faeces and food.
86. What methods of laboratory diagnostics
of botulism:

a biological test on mice;


b bacteriological of blood;
c serological of blood;
d bacteriological of faeces and food.
87. What serological tests are typical for
brucellosis:
a RCF;
b reaction of agglutination of Right,
Heddelson;
c RIHA.
d RIF;
e Reaction of latex agglutination.
88. What methods for pseudotuberculosis
diagnostics:
a epidemiological;
b clinical;
c bacteriological;
d serological.
89. Material for early bacteriological
diagnostics of typhoid and paratyphoid fever
is:
faeces;
blood;
CSF;
urine;
all mentioned.
90. In what case can be false negative
result of immunoenzyme assay of HIV
infection:
a malaria;
b patients, treated with corticosteroids;
c *in pregnant;
d in homosexual persons;
e in drug abused.
91. What peculiarities of blood cell count are
typical for climax period of typhoid fever:
Leucopenia, neutropenia, eosynopenia,
relative lymphocytosis;
Normal count or insignificant leukocytosis,
tendency to neutrophil increase, drum - stick
shift to the left, monocytosis, plasmatic Turk
cells;
Leucopenia, aneosynophilia, lymphocytosis,
thrombocytopenia, ESR elevation;
Leucocytosis, neutrophenia, lympho monocytosis, atypical mononuclears.
92. What changes in BCC are typical for
influenza:
Leucocytosis, neutrophenia, lymphocytosis,
monocytosis;
Leucopenia, neutropenia, lymphocytosis;
Leucocytosis, neutropenia, monocytopenia,
lymphocytosis.
Leucocytosis, neutrophilosis, drum - stick
shift to the left.
93. What changes in BCC are typical for
infectious mononucleosis:
a Leucocytosis, neutrophenia,
lymphocytosis, monocytosis;

c
d

a
b
c
d

125

a
b
c
d

a
b
c
d
a
b
c
d
a
b
c

a
b
c

b Leucopenia, neutropenia, lymphocytosis;


c Leucocytosis, neutropenia,
monocytopenia, lymphocytosis.
d Leucocytosis, neutrophilosis, drum - stick
shift to the left.
94. What changes in BCC are typical for
acute tonsillitis:
Leucocytosis, neutrophenia, lymphocytosis,
monocytosis;
Leucopenia, neutropenia, lymphocytosis;
Leucocytosis, neutropenia, monocytopenia,
lymphocytosis.
Leucocytosis, neutrophilosis, drum - stick
shift to the left.
95. What methods of specific diagnostics of
rotaviral gastroenteritis:
a Bacteriological of faeces;
b Faeces microscopy;
c Reaction of latex agglutination;
d Rectoromanoscopy;
96. What methods of specific diagnostics of
gastrointestinal form of salmonellosis:
Bacteriological of faeces;
Faeces microscopy;
Reaction of latex agglutination;
Rectoromanoscopy;
97. What methods of specific diagnostics of
intestinal lambliosis:
Bacteriological of faeces;
Faeces microscopy;
Reaction of latex agglutination;
Rectoromanoscopy;
98. What changes in liquor are typical for
meningococcal meningitis on microscopy:
Purulent, muddy;
Transparent or twinkling;
hemorrhagical;
99. What changes in liquor are typical for
meningeal form of tick encephalitis on
microscopy:
Purulent, muddy;
Transparent or twinkling;
hemorrhagical;
100. What blood cells count is typical for
meningococcaemia:
a Neutrophil hyperleukocytosis with left shift,
high ESR;
b Moderate neutrophil leucocytosis, ESR is
moderately increased;
c Leucopenia, lymphomonocytosis, sometimes
ESR increase;
d Leucopenia, relative lymphocytosis, increase
of plasmatic cells number, ESR increase.
101. What blood sell count is typical for
influenza:
a Neutrophil hyperleukocytosis with left
shift, high ESR;
b Moderate neutrophil leucocytosis, ESR is
moderately increased;

c Leucopenia, lymphomonocytosis,
sometimes ESR increase;
d Leucopenia, relative lymphocytosis,
increase of plasmatic cells number,
ESR increase.
102. What blood sell count is typical for tick
borne encephalitis:
Neutrophil hyperleukocytosis with left shift,
high ESR;
Moderate neutrophil leucocytosis, ESR is
moderately increased;
Leucopenia, lymphomonocytosis, sometimes
ESR increase;
Leucopenia, relative lymphocytosis, increase
of plasmatic cells number, ESR increase.
103. Choose methods of specific diagnostics
of louse - borne typhus, Brills disease:
a RCF, RIA with rickettsia Provazeky;
b RCF with rickettsia Burneti;
c RCF, RIA with rickettsia Sybirica;
d IEA with borrelia Burgdorferi.
104. Choose methods of specific diagnostics
of North-Asian rickettsiosis:
a RCF, RIA with rickettsia Provazeky;
b RCF with rickettsia Burneti;
c RCF, RIA with rickettsia Sybirica;
d IEA with borrelia Burgdorferi.
105. Choose methods of specific diagnostics
of Q fever:
a RCF, RIA with rickettsia Provazeky;
b RCF with rickettsia Burneti;
c RCF, RIA with rickettsia Sybirica;
d IEA with borrelia Burgdorferi.
106. Choose the analysis of CSF, typical for
purulent meningitis:
Colorless, transparent, cytosis - 0,003 109
/l., lymphocytes 100 %, protein 0,33 /.
Sediment reaction is negative. Pressure is
250 mm of water;
Colorless, transparent, cytosis 0,2 109 /l,
lymphocytes 80 %, protein 1,0 g/l, sediment
reaction positive, pressure is 300 mm of a
water;
Colorless, transparent, cytosis 0,01 109 /l,
lymphocytes 85 %, protein 0,3 g/l, sediment
reaction negative, pressure is 300 mm of a
water;
Muddy, white yellow color, cytosis 15,0
109 /l, neutrophils 100 %, protein 6,6 g/l,
sediment reaction are sharply positive,
pressure of 350 mm of a water;
107. Choose the analysis of CSF, typical for
mentigismus:
Colorless, transparent, cytosis - 0,003 109
/l., lymphocytes 100 %, protein 0,33 /.
Sediment reaction is negative. Pressure is
250 mm of water;
Colorless, transparent, cytosis 0,2 109 /l,
lymphocytes 80 %, protein 1,0 g/l, sediment

b
c
d

126

reaction positive, pressure is 300 mm of a


water;
Colorless, transparent, cytosis 0,01 109 /l,
lymphocytes 85 %, protein 0,3 g/l, sediment
reaction negative, pressure is 300 mm of a
water;
Muddy, white yellow color, cytosis 15,0
109 /l, neutrophils 100 %, protein 6,6 g/l,
sediment reaction are sharply positive,
pressure of 350 mm of a water;
108. Choose the analysis of CSF, typical for
serous meningitis:
a Colorless, transparent, cytosis - 0,003
109 /l., lymphocytes 100 %, protein
0,33 /. Sediment reaction is
negative. Pressure is 250 mm of water;
b Colorless, transparent, cytosis 0,2 109
/l, lymphocytes 80 %, protein 1,0 g/l,
sediment reaction positive, pressure is
300 mm of a water;
c Colorless, transparent, cytosis 0,01 109
/l, lymphocytes 85 %, protein 0,3 g/l,
sediment reaction negative, pressure is
300 mm of a water;
d Muddy, white yellow color, cytosis 15,0

109 /l, neutrophils 100 %, protein 6,6


g/l, sediment reaction are sharply
positive, pressure of 350 mm of a
water;

127

It is another pare of shoes.

g Infusion of 5% glucose solution;


h *Adrenaline;
i
Glucocorticoids;
j
Cordiamine.
116. Mention appropriate remedies for
treatment of hypovolemic shock:
f Ringer-Locks solution;
g Isotonic solution of sodium chloride;
h Trisol solution;
i
10% albumin solution;
j
Rheopolyglucine.
117. Which clinical signs are typical for
pulmonary edema?
f Crepitating rails on auscultation of
lungs.
g Severe dyspnea while the patient is
at rest in the bed, forced state of the
patient in the bed
h Diffuse moist rails of different
calibers on auscultation of the lung
i
Bradycardia
j
Noise of pleura friction
118. Mention the clinical signs of acute liver
failure.
f Disorders of color sensation;
g Leucopoenia in CBC;
h Progressive decreasing of liver size;
i
Disorders of consciousness;
j
Low level of prothrombine index.
119. Choose the drug for treatment of
comatose form of malaria (due to
falciparum) caused with resistant to
chloroquine plasmodia.
f Primaquine;
g Ribavirin;
h Zydovudin;
i
Quinidine gluconate;
j
Metronidazole.
120. What correlation of colloid and saline
solutions for intensive treatment of septic
shock is optimal?
f 1:1;
g 1:2;
h 2:1;
i
1:3;
j
3:1.
121. What daily dosage of dextrans is safe
for treatment of infectious toxic shock?
f 3 liters;
g 500 ml;
h 10 ml;
i
800 ml;
j
2 liters.
122. Which complications are possible when
dextrans are overdosed?
f Necrosis of large muscles;
g Decreasing of acuteness of hearing;
h Brain edema;
i
Pulmonary edema;

URGENT STATES IN INFECTIOUS


PATHOLOGY
109. For what kind of shock increasing of
relative density of blood plasma is typical?
f infectious toxic;
g *hypovolemic;
h anaphylactic;
i
cardiogenic;
j
traumatic.
110. What preferable way of drugs
administration in case of infectious - toxic
shock development?
f per orally;
g subcutaneously;
h intramuscular;
i
*intravenously;
j
intraartreioly.
111. What one of the following solutions is
most effective for initial treatment of
infectious toxic shock?
a) Trisol;
b) 4% natrium hydrocarbonate solution;
c) Polyglucine;
d) Innunoglobuline;
e) Regydrone.
112. Choose the clinical criteria of sopor (2):
f Contact with patient is absent;
g *Contact with patient is possible only
on level of verbal-mimic reactions;
h *Directed reactions on pain
irritations;
i
Reactions of pupils on light are
absent;
j
Patients answers on questions are
monosyllabic and not always
adequate.
113. Mention the most typical clinical signs
of brain edema:
f Hemorrhagical rash on the mucous
membrane of soft palate;
g Cardiac arrhythmia;
h Psycho-motor excitation;
i
Tonoclonic cramps;
j
Tonic-and clonic cramps.
114. What level of systolic arterial pressure
can result in blocking of renal filtration?
f 105/70 mm Hg;
g 100/60 mm Hg;
h 70/50 mm Hg;
i
95/60 mm Hg;
j
90/50 mm Hg.
115. What is drug of choise for treatment of
anaphylactic shock?
f Rhiboxine;

128

Necrosis of epithelium cells of renal


tubules.
123. What speed of saline solutions infusion
you must keep during the first stage of
treatment of patient with hypovolemic
shock?
f 1 l/h;
g 3 l/h;
h 100-120 ml/min;
i
60 drops/min;
j
50 ml/min.
124. What drugs should be given for
patients with septic shock, refractory to
volume replacing treatment and
glucocorticoids?
f To continue infusion of high doses of
glucocorticoids (up to 1000-1500
mg);
g To begin intravenous infusion of
dofamine;
h Treatment with enterosorbents;
i
To begin transfusion of blood;
j
To use lymposorbtion or
plasmosorbtion.
125. Choose 2 of all laboratory signs typical
for acute respiratory failure:
f *Pa v O2 >50 mm Hg;
g Pa v CO2 > 50 mm Hg;
h *Pa v O2 40 mm Hg;
i
Pa v CO2 40 mm Hg;
j
pH of the blood 7,38.
126. Choose clinical sign that typical for
croup of II stage.
f Whistle dry rails over a lungs;
g *Inspiratory stridor;
h Comatose state;
i
Discharging of plentiful foamy
sputum;
j
Diffuse moist rails of different
calibers on auscultation of lungs.
127. Indicate the most characteristic clinical
signs of acute broncoobstruction (2).
f *Whistle dry rails over the lung;
g Dry barking cough;
h *Tympanic sound on percussion of
lungs;
i
Discharging of abundant foamy
sputum;
j
Diffuse moist rails of different
calibers on auscultation of lungs.
128. Indicate the most characteristic clinical
signs of pulmonary edema (2):
f Whistle dry rails over a lungs;
g Severe pain in the chest on inhale;
h Comatose state;
i
*Discharging of plentiful foamy
sputum;
j
*Diffuse moist rails of different
calibers on auscultation of lungs.

129. Choose appropriate clinical and


laboratory signs of acute respiratory failure
(3):
f *Dyspnea;
g *Pa v O2 40 mm Hg;
h *Diffuse cyanosis;
i
Pa v CO2 40 mm Hg
j
The work of additional muscles on
breathing
130. Which clinical and laboratory signs are
typical for acute renal failure (ARF) (2)?
f *Dryness of mucous membranes;
g *Level of creatinine in the blood
0,20 mkmol/l;
h Level of urea in the blood 0,08
mkmol/l;
i
High temperature;
j
Pain in abdomen.
131. Oliguria it is state when volume of
diuresis is:
f up to 1000 ml;
g up to 700 ml;
h *up to 500 ml;
i
up to 300 ml;
j
up to 100 ml.
132. Which volume of diuresis we can
classify as anuria?
f 500 ml;
g *300 ml;
h 200 ml;
i
100 ml;
j
50 ml.
133. These are all clinical and laboratory
signs of acute renal failure beside of:
f vomiting;
g diarrhea;
h dryness of mucous membranes;
i
*urea of the blood 0,08 mmol/l;
j
creatinine of the blood 0,27
mmol/l.;
134. Which infectious diseases can result in
acute renal failure (3)?
f *Leptospirosis;
g Brucellosis;
h *Haemorrhagical fevers;
i
Salmonellosis;
j
*Cholera.
135.Indications for hemodialysis in patients
with acute renal failure are all following
beside of (2):
f *Potassium of plasma 5,7 mmol/l;
g Urea of plasma 0.24 mkmol/l;
h *Creatinine of plasma 0,5 mkmol/l;
i
Creatinine of plasma 0,3 mkmol/l;
j
*Longevity of anuria stage more than
10 days.
136.Which drugs are effective for treatment
of acute respiratory failure (2)?
f Infusion of 10% solution of glucose;

129

g Infusion of 5% solution of glucose;


h *Glucocorticoids;
i
Prozerinum;
j
*Artificial pulmonary ventilation.
137. Which drugs can suppress formation of
foam in patients with pulmonary edema (2)?
f *Inhaling of 78% ethanol alcohol;
g Pentaminum i/m;
h Inhaling oxygen mixed with air;
i
Inhaling of ether;
j
*Inhaling of antiphomsilanum.
138. What clinical signs are typical for
diphtheric croup:
e Sudden beginning with fast
development of laryngeal stridor and
wavy course;
f Sudden development of laryngeal
stridor during eating or game,
absence of fever and catharal
symptoms;
g Gradual development of laryngeal
stridor, frequently normal body
temperature, multiple of dry rales;
h Fever, harsh voice, consequent
development of barking cough,
dysphonia, lengthened sonorous
breath, inspirator dispose, cyanosis
of lips, tachicardia.
139. What clinical signs are typical for
aspiration of foreign body:
e Sudden beginning with fast
development of laryngeal stridor and
wavy course;
f Sudden development of laryngeal
stridor during eating or game,
absence of fever and catharal
symptoms;
g Gradual development of laryngeal
stridor, frequently normal body
temperature, multiple of dry rales;
h Fever, harsh voice, consequent
development of barking cough,
dysphonia, lengthened sonorous
breath, inspirator dispose, cyanosis
of lips, tachicardia.
140. What clinical signs are typical for croup
as complication of influenza and ARVI:
e Sudden beginning with fast
development of laryngeal stridor and
wavy course;
f Sudden development of laryngeal
stridor during eating or game,
absence of fever and catharal
symptoms;
g Gradual development of laryngeal
stridor, frequently normal body
temperature, multiple of dry rales;
h Fever, harsh voice, consequent
development of barking cough,

dysphonia, lengthened sonorous


breath, inspirator dispose, cyanosis
of lips, tachicardia.
141. Chouse main pathogenetic mechanism
of toxic shock:
e dehydratation;
f Primary decrease of cardiac input;
g Pathological increase of vascular
capacity;
h Severe deficiency of steroid
hormones;
142. Chouse main pathogenetic mechanism
of cardiogenic shock:
e dehydratation;
f Primary decrease of cardiac input;
g Pathological increase of vascular
capacity;
h Severe deficiency of steroid
hormones;
143. Chouse main pathogenetic mechanism
of hypovolemic shock:
a) dehydratation;
e Primary decrease of cardiac input;
f Pathological increase of vascular
capacity;
g Severe deficiency of steroid
hormones;
144. Which of basic logical ( Kochs
postulates) proof that disease is caused by a
microorganism:
a) Microorganism must be present in
every case of disease, but absent in
healthy individuals;
b)suspected microorganism must be
isolated from affected person and grown
in culture;
c)Same disease must resalt when
isolated microorganism is introduced into
healthy host;
d)Same microorganism must be isolated
again from second affected.
e)* All mentioned

ANSWERS FOR TESTS FOR 5th YEAR


STUDENS
NA NA N
n
n
s
s
.
.
1 A 6 B 1
4 D 2

130

A NA N
n
n
s
s
.
.
D 1 A 2
9
5

A NA N
n
n
s
s
.
.
A 3 E 3
1
7

A NA N
n
n
s
s
.
.
B 4 A 5
4 D 0

A
n
s
.
D

7
2 C 6 A 1
5 C 2
8
3 A 6 A 1
6
2
9
4 C 6 B 1
7
3
0
5 E 6 B 1
8
3
1
6 B 6
1
9
3
2
7 C 7 C 1
0
3
3
8 B 7 A 1
1
3
4
9 C 7 D 1
2
3
5
1
7 A 1
0
3
3
6
1 B 7 D 1
1 D 4
3
7
1 E 7 C 1
2
5
3
8
1 B 7 B 1
3
6
3
9
1 E 7 A 1
4
7
4
0
1 B 7 B 1
5 E 8
4
1
1 C 7 A 1
6
9
4
2
1 B 8 E 1
7 C 0
4
3
1 A 8 B 1
8 D 1
4
4
1 B 8 D 1
9
2
4
5
2 C 8 A 1
0
3
4
6

0
B 1
9
1
D 1
9
2
B 1
9
3
E 1
9
4
C 1
9
5
B 1
9
6
B 1
9
7
B 1
9
8
C 1
9
9
A 2
0
0
B 2
0
1
A 2
0
2
C 2
0
3
B 2
0
4
A 2
0
5
A 2
0
6
B 2
0
7
E 2
J 0
8
B 2
0
9

3
D 2
5
4
C 2
5
5
D 2
5
6
D 2
5
7
B 2
5
8
C 2
5
9
D 2
6
0
C 2
6
1
D 2
6
2
D 2
6
3
B 2
6
4
C 2
6
5
D 2
6
6
B 2
6
7
C 2
6
8
B 2
6
9
A 2
7
0
C 2
7
1
D 2
7
2

6
C 3
1
7
B 3
1
8
D 3
1
9
B 3
2
0
B 3
2
1
C 3
2
2
A 3
2
3
C 3
2
4
B 3
2
5
C 3
2
6
C 3
2
7
A 3
2
8
A 3
2
9
A 3
3
0
B 3
3
1
B 3
3
2
C 3
3
3
C 3
3
4
D 3
3
5

9
E 3
8
0
B 3
C 8
1
A 3
B 8
D 2
C 3
8
3
A 3
8
4
B 3
8
5
C 3
8
6
A 3
8
7
D 3
8
8
A 3
8
9
B 3
9
0
B 3
9
1
D 3
9
2
B 3
9
3
C 3
9
4
A 3
9
5
D 3
9
6
A 3
9
7
B 3
9
8

2
A 4
4
3
C 4
4
4
A 4
4
5
B 4
4
6
D 4
4
7
B 4
4
8
B 4
4
9
C 4
5
0
B 4
5
1
B 4
5
2
A 4
5
3
E 4
5
4
A 4
5
5
A 4
5
6
B 4
5
7
B 4
5
8
A 4
5
9
B 4
6
0
E 4
6
1

5
C 5
D 0
6
A 5
0
7
D 5
0
8
B 5
0
9
C 5
1
0
B 5
F 1
1
B 5
1
2
C 5
1
3
A 5
E 1
4
D 5
1
5
B 5
1
6
A 5
1
7
B 5
1
8
D 5
1
9
E 5
2
0
D 5
2
1
A 5
2
2
C 5
2
3
B 5
2
4

2 B 8 B 1
1
4 E 4
7
2 A 8 B 1
2
5
4
8
2 C 8 B 1
3
6
4
9
2 C 8
1
4
7
5
0
2 A 8
1
5
8
5
1
2 C 8 D 1
6
9
5
2
2 B 9 B 1
7
0
5
3
2 D 9 E 1
8
1
5
4
2 A 9 D 1
9
2
5
5
3
9 C 1
0
3
5
6
3 B 9 A 1
1
4
5
7
3 D 9 C 1
2
5
5
8
3 C 9 A 1
3
6
5
9
3 A 9 D 1
4
7
6
0
3 B 9 B 1
5
8 D 6
1
3 B 9 B 1
6
9 C 6
2
3 A 1 A 1
7
0 B 6
0
3
3 C 1 D 1
8
0
6
1
4
3 B 1 C 1
9
0
6
2
5
4 A 1 E 1
0
0
6

C
D
B
A
C
B
C
D
A
C

A
B
C
A
C
B
A
C

131

A 2
1
0
C 2
1
1
E 2
1
2
B 2
1
3
B 2
1
4
C 2
D 1
5
C 2
1
6
A 2
1
7
B 2
1
8
D 2
1
9
B 2
2
0
C 2
2
1
C 2
2
2
E 2
2
3
C 2
2
4
A 2
2
5
C 2
2
6
A 2
2
7
B 2
2
8
D 2
2

B 2
7
3
A 2
7
4
C 2
7
5
A 2
7
6
B 2
7
7
B 2
7
8
C 2
7
9
C 2
8
0
D 2
8
1
B 2
8
2
D 2
8
3
E 2
8
4
C 2
8
5
B 2
8
6
D 2
8
7
B 2
8
8
C 2
8
9
B 2
9
0
C 2
9
1
D 2
9

B 3
3
6
A 3
3
7
B 3
3
8
E 3
3
9
D 3
4
0
E 3
4
1
A 3
4
2
E 3
4
3
B 3
4
4
C 3
4
5
C 3
4
6
D 3
4
7
A 3
4
8
D 3
4
9
C 3
5
0
B 3
5
1
C 3
5
2
C 3
5
3
C 3
5
4
A 3
5

C 3
9
9
A 4
0
0
A 4
0
1
B 4
0
2
D 4
0
3
B 4
0
4
A 4
0
5
D 4
0
6
B 4
0
7
B 4
0
8
D 4
0
9
C 4
1
0
B 4
1
1
B 4
1
2
C 4
1
3
A 4
1
4
D 4
1
5
C 4
1
6
D 4
1
7
B 4
1

A 4
6
2
A 4
6
3
B 4
6
4
C 4
E 6
5
A 4
6
6
B 4
6
7
A 4
6
8
B 4
6
9
B 4
7
0
C 4
7
1
A 4
7
2
A 4
7
3
A 4
7
4
A 4
7
5
A 4
7
6
B 4
7
7
C 4
7
8
B 4
7
9
C 4
8
0
D 4
8

A 5
2
5
D 5
2
6
C 5
2
7
B 5
2
8
A 5
2
9
C 5
3
0
C 5
3
1
D 5
3
2
B 5
3
3
A 5
3
4
B 5
3
5
B 5
3
6
C 5
3
7
A 5
3
8
B 5
3
9
C 5
4
0
E 5
4
1
D 5
4
2
B 5
4
3
B 5
4

D
B
C
E
D
C
D
E
A
E
E

B
A
B
B
C
D
A
B

3
4 B 1
1
0
4
4 D 1
2
0
5
4 B 1
3
0
6
4 C 1
4
0
7
4 A 1
5
0
8
4 C 1
6
0
9
4 C 1
7
1
0
4 C 1
8
1
1
4 A 1
9
1
2
5 A 1
0
1
3
5 A 1
1
1
4
5 B 1
2
1
5
5 C 1
3
1
6
5 B 1
4
1
7
5 D 1
5
1
8
5 C 1
6
1
9
5 A 1
7
2
0
5 A 1
8
2
1
5 C 1
9
2
2

6
C 1
6
7
D 1
6
8
B 1
C 6
9
E 1
7
0
C 1
E 7
1
A 1
B 7
2
B 1
D 7
3
A 1
C 7
4
A 1
B 7
5
E 1
7
6
D 1
E 7
7
E 1
7
8
B 1
7
9
C 1
8
0
A 1
8
1
A 1
D 8
2
A 1
B 8
E 3
D 1
8
4
D 1
8
5

9
A 2
3
0
C 2
3
1
B 2
3
2
B 2
3
3
C 2
3
4
A 2
3
5
A 2
3
6
B 2
3
7
B 2
3
8
C 2
3
9
B 2
4
0
C 2
4
1
C 2
4
2
E 2
4
3
A 2
4
4
B 2
4
5
A 2
4
6
A 2
4
7
A 2
4
8

2
D 2
9
3
B 2
9
4
F 2
9
5
A 2
9
6
D 2
9
7
B 2
9
8
D 2
9
9
A 3
0
0
C 3
0
1
D 3
0
2
C 3
0
3
A 3
0
4
C 3
0
5
D 3
0
6
B 3
0
7
B 3
0
8
A 3
0
9
C 3
1
0
A 3
1
1

5
C 3
5
6
C 3
5
7
B 3
5
8
A 3
5
9
A 3
6
0
A 3
6
1
B 3
6
2
B 3
6
3
E 3
6
4
E 3
F 6
5
D 3
E 6
6
C 3
6
7
E 3
6
8
A 3
6
9
B 3
7
0
D 3
7
1
C 3
7
2
C 3
7
3
A 3
7
4

8
A 4
1
9
C 4
2
0
C 4
2
1
B 4
2
2
D 4
2
3
C 4
2
4
A 4
2
5
C 4
2
6
C 4
2
7
B 4
2
8
D 4
2
9
C 4
3
0
B 4
3
1
B 4
3
2
B 4
3
3
C 4
3
4
A 4
3
5
C 4
3
6
A 4
3
7

1
A 4
8
2
C 4
8
3
B 4
8
4
B 4
D 8
5
C 4
8
6
C 4
8
7
D 4
E 8
8
B 4
8
9
C 4
9
0
B 4
9
1
D 1
9
2
C 4
9
3
B 4
9
4
C 4
9
5
C 4
9
6
C 4
9
7
B 4
9
8
C 4
9
9
C 5
0
0

4
B 5
E 4
5
C 5
4
6
C 5
4
7
B 5
4
8
B 5
C 4
9
C 5
5
0
D 5
5
1
C 5
5
2
B 5
E 5
3
C 5
5
4
B 5
5
5
C 5
5
6
5
5
7
5
5
8
A 5
5
9
A

6 B 1
0
2
3
6 D 1
1
2
4
6 A 1
2
2
5
6 C 1
3
2
6

B
C
C
D
A

B 1
E 8
6
C 1
8
7
C 1
8
8
A 1
8
9

A 2
4
9
B 2
5
0
C 2
5
1
C 2
5
2

C 3
1
2
B 3
1
3
B 3
1
4
B 3
1
5

B 3
7
5
C 3
7
6
D 3
7
7
D 3
E 7
8

B 4
3
8
C 4
3
9
D 4
4
0
C 4
4
1

A 5
0
1
A 5
0
2
A 5
E 0
3
C 5
E 0
4

A
A
A
B

ANSWERS FOR CLINICAL CASES AND


TESTS FOR 6th YEAR STUDENS

B
28 Diagnosis: viral hepatitis
Differential diagnosis: hepatitis B, C,
D, leptospirosis.
29 Diagnisis: tropical malaria, malarial
coma.
Differential diagnosis: sepsis, typhoid
fever,
influenza,
leptospirosis,
hemorrhagic fevers.
30 Diagnosis: influenza.
Differential diagnosis: parainfluenza,
adenoviral
infection,
measles,
leptospirosis, paratyphoid A.
31 Diagnosis malaria tertiana.
Differential diagnosis: influenza,
recurrent typhus.
32 Diagnosis: Shigellosis.
Differential
diagnosis: amebiazis,
salmonellosis,
echerichiosis,
intestinal iersiniosis.
33 Diagnosis: hemorrhagic fever with
renal syndrome.
Differential diagnosis: leptospirosis.
34 Diagnosis: Leptospirsis.
Differential diagnosis: viral hepatitis,
hemorrhagic
fever
with
renal
syndrome, pseudotuberculosis.
35 Diagnosis: opistorchosis.
Differential diagnosis: typhoid and
paratyphoid A, B; viral hepatitis C, B,
acute cholecistitis.
36 Diagnosis: diphtheria of larynx;
Differential diagnosis: parainfluenza.
37 Diagnosis:
rabies.
Hydrophobic
phase.
Differential
diagnosis:
tetanus,
arboviral encephalitis, atropine or
strychnine
poisoning,
delirium
tremens.
38 Diagnosis: polyomielitis.

E
C
D
A
C
E
E
E
C

C
C
C

132

Differential diagnosis: botulism,


arboviral
encephalitis,
lymphocytar
meningitis;
ECHO and Kocksaki diseases.
39 Diagnosis: plague, pulmonary form.
Differential diagnosis: pulmonary
form
of
antrax;
pneumococcal
pneumonia, influenza, complicated
with edema of lungs.
40 Diagnosis: Epstine Burr infectious
mononucleosis.
Differential
diagnosis:
cytomegaloviral infection, felinosis,
acute tonsillitis, localized form of
diphtheria, acute HIV infection,
leucosis.
41 Diagnosis: enteroviral disease.
Differential
diagnosis:
rotaviral
infection,
intestinal
iersiniosis,
echerichiosis, salmonellosis.
42 Diagnosis: tularemia, eye bubonic
form.
Differential diagnosis: plague,
sodocu,
felinosis,
purulent
lymphadenitis.
43 Diagnosis: arboviral tick- borne
encephalitis.
Differential diagnosis: polyomielitis,
rabies,
meningococcal
meningoencephalitis.
44 Diagnosis: serum disease.
Differential diagnosis: measles,
iersiniosis.
45 Diagnosis: tularemia, eye bubonic
form.
Differential diagnosis: plague,
sodocu,
felinosis,
purulent
lymphadenitis, infectious mononucleosis.
46 Diagnosis: anaphylactic shock

Differential diagnosis: toxic infectious


shock, acute poisoning.
47 Diagnosis: acute brucellosis.
Differential diagnosis: typhoid fever,
tuberculosis,
lymphogranulematosis,
reumatic fever, iersiniosis.
48 Diagnosis: measles.
Differential diagnosis: rubella,
enteroviral exantema, serum disease,
infectious mononucleosis.
49 Diagnosis: Brills disease (endemic
louse born typhus).
Differential diagnosis: influenza,
arboviral encephalitis, meningococcal
infection.
50 Diagnosis: brucellosis.
Differential diagnosis: tuberculosis,
sepsis,
lymphogranulematosis,
pseudotuberculosis,
reumatic
fever,
malaria.
51 Diagnosis: plague, bubonic form.
Differential diagnosis: tularemia,
sodocu, purulent lymphadenitis.
52 Diagnosis: infectious mononucleosis.
Differential diagnosis:
cytomegaloviral
infection,
felinosis,
acute tonsillitis, localized form of
diphtheria, acute HIV infection,
leucosis, viral hepatitis A.
53 Diagnosis: tropical malaria
Differential diagnosis: sepsis, typhoid
fever,
influenza,
leptospirosis,
hemorrhagic fevers.
54 : rabies, hydrophobic form.
Differential diagnosis: tetanus, tick
born encephalitis, delirium tremens.

133

S-ar putea să vă placă și